Steroids for Acute COPD—But for How Long?

Article Type
Changed
Tue, 12/13/2016 - 12:08
Display Headline
Steroids for Acute COPD—But for How Long?
Not only was a shorter course of glucocorticoid therapy as effective as a 14-day regimen, but there was no difference in the time to next exacerbation.

PRACTICE CHANGER

Prescribe a five-day regimen of glucocorticoid therapy for acute exacerbations of chronic obstructive pulmonary disease (COPD); the shorter course of treatment appears to be as effective as a 14-day regimen.1

Strength of ­recommendation

B: Based on a single well-­designed randomized controlled trial (RCT).1

ILLUSTRATIVE CASE

A 55-year-old man with COPD presents to the emergency department (ED) with progressive shortness of breath, cough, and sputum production in the past four days. He is diagnosed with a COPD exacerbation, treated with corticosteroids, and admitted to the hospital. His inpatient treatment includes antibiotics, inhaled albuterol and ipratropium, supplemental oxygen, and oral corticosteroids.

How many days should he take oral steroids?

 

 

 

Severe exacerbations of COPD are independently associated with mortality,2 regardless of baseline severity. Guidelines and systematic reviews highlight the importance of using oral glucocorticoids in the management of acute COPD exacerbations, as the drugs have been found to shorten recovery time and length of hospital stay, improve lung function, and reduce the risk for early relapse and treatment failure.3-5 What is not clear is how long the course of oral steroids should be.

What we know (and don’t know) about duration

Data supporting a 14-day course of steroids versus a longer (eight-week) duration come from the Systemic Corticosteroids in COPD Exacerbations trial.6 Global Initiative for Chronic Obstructive Lung Disease (GOLD) criteria suggest a 10- to 14-day regimen (30 to 40 mg/d) but acknowledge that there is a lack of data from clinical and observational studies to support this recommendation.3 A recent Cochrane review compared a short course of treatment (three to seven days) with a longer regimen (10 to 15 days) and found that the evidence to support a clinical practice change was inconclusive.5

The study detailed in this PURL—a double-blind RCT comparing five-day with 14-day oral steroid treatment in patients ­hospitalized for acute COPD exacerbation—had more definitive results.1

Continue reading for the study summary...

 

 

STUDY SUMMARY

Shorter and longer regimens produce equal results

Leuppi et al1 used noninferiority methodology to compare a five- and a 14-day course of prednisone 40 mg/d to treat patients with COPD exacerbations. A patient was considered to have an exacerbation if he or she had a change from baseline in two or more of the following: dyspnea, cough, sputum quantity, or purulence.

Participants were patients who presented to the EDs of five Swiss teaching hospitals between March 2006 and February 2011. To be eligible, individuals had to be 40 or older and have at least 20 pack-years of smoking. Exclusion criteria included asthma, mild obstruction (FEV1/FVC > 70%), pneumonia, an estimated survival of less than six months, pregnancy, and lactation.

All the participants (N = 311) received 40 mg methylprednisolone intravenously on day 1, followed by prednisone 40 mg orally on days 2 through 5. The researchers then randomly divided participants into two groups: One group continued to take prednisone 40 mg/d and the other group received a matching placebo for an additional nine days. Participants in both groups also received antibiotics for seven days, twice-daily inhaled steroids, daily tiotropium, and nebulized albuterol, as needed; additional oral glucocorticoids could be administered, as well, at the discretion of the treating physicians.

The primary outcome was the time to the next COPD exacerbation, up to 180 days. Noninferiority between the groups was defined as no more than a 15% absolute increase in exacerbations. The dropout rate was 5.7%, evenly divided between groups. Intention to treat and per-protocol analyses were conducted, and hazard ratios (HRs) were calculated using the Kaplan-Meier method and Cox proportional hazards models.

The time to next COPD exacerbation did not differ between the study groups: 56 days for those on the five-day steroid regimen versus 57 days for those on the
14-day regimen in the intention-to-treat analysis (HR, 0.95). Sensitivity analyses adjusting for baseline characteristics provided similar results, as did the per-protocol analysis.

Secondary outcomes (overall survival; need for mechanical ventilation; need for additional corticosteroids; and clinical performance measures, such as dyspnea score and quality of life) also did not differ between groups. Nor were there differences in hyperglycemia, worsening hypertension, infection, or other adverse effects typically associated with glucocorticoid use. The active treatment group took more than 400 mg more prednisone than the placebo group (mean, 793 mg vs 379 mg, respectively).

 

 

WHAT’S NEW?

Now we know: five days is enough

While randomized trials have found that glucocorticoids improve COPD symptoms, the optimal treatment dose and duration were not known. Indeed, current guidelines recommend treatment for more than five days.3 This trial clearly demonstrated that 40-mg prednisone for five days is at least as good as a 14-day treatment course. Furthermore, it is un­necessary to taper the short-course therapy, which simplifies the regimen.

CAVEATS

Will the results apply to those less severely ill?

More than 80% of patients with acute COPD exacerbations can be managed in an outpatient setting.3 However, participants in this trial were hospitalized for a median of 8.5 days, and most had severe or very severe COPD—and thus, were not fully representative of COPD patients typically seen in an outpatient practice. Yet patients with less severe disease should be at least as likely to ­respond to short-course steroids as those whose COPD is more ­severe.

It is important to note that participants in this study all received optimal guideline-based therapies during hospitalization, which may be difficult to achieve for some patients treated in an outpatient setting. Finally, treatment adherence observed during the hospitalization period in this trial is unlikely to be replicated in the outpatient setting.

CHALLENGES TO IMPLEMENTATION

Identifying patients who need steroids for a longer duration

For patients with new COPD exacerbations or those successfully treated using short-course therapy in the past, a five-day regimen may be appropriate. For those in whom prior attempts at short-course treatment have failed, however, a 14-day course of treatment may be more advisable. That said, no guidelines are available to help us determine which patients previously treated with a longer regimen will find the shorter course of treatment unsuccessful.

Continue for references...

 

 

REFERENCES

1. Leuppi JD, Schuetz P, Bingisser R, et al. Short-term vs conventional glucocorticoid therapy in acute exacerbations of chronic obstructive pulmonary disease: the REDUCE randomized clinical trial. JAMA. 2013;309:2223-2231.

2. Soler-Cataluna JJ, Martinez-Garcia MA, Roman Sanchez P, et al. Severe acute exacerbations and mortality in patients with chronic obstructive pulmonary disease. Thorax. 2005;60:925-931.

3. Global Initiative for Chronic Obstructive Lung Disease, Inc. The global strategy for diagnosis, management, and prevention of chronic obstructive pulmonary disease. www.goldcopd.org. Accessed January 9, 2014.

4. Quon BS, Gan WQ, Sin DD. Contemporary management of acute exacerbations of COPD: a systematic review and metaanalysis. Chest. 2008;133:756-766.

5. Walters JA, Wang W, Morley C, et al. Different durations of corticosteroid therapy for exacerbations of chronic obstructive pulmonary disease. Cochrane Database Syst Rev. 2011; (10):CD006897.

6. Niewoehner DE, Erbland ML, Deupree RH, et al. Effect of systemic glucocorticoids on exacerbations of chronic obstructive pulmonary disease. Department of Veterans Affairs Cooperative Study Group. N Engl J Med. 1999;340:1941-1947.

ACKNOWLEDGEMENT

The PURLs Surveillance System is supported in part by Grant Number UL 1RR 024999 from the National Center for Research Resources, a Clinical Translational Science Award to the University of Chicago. The content is solely the responsibility of the authors and does not necessarily represent the official views of the National Center for Research Resources or the National Institutes of Health.

Copyright © 2014. The Family Physicians Inquiries Network. All rights reserved.

Reprinted with permission from the Family Physicians Inquiries Network and The Journal of Family Practice. 2014;63(1):29-30, 32.

Author and Disclosure Information

Gary N. Asher, MD, MPH, Anne Mounsey, MD

Gary N. Asher and Anne Mounsey are with the Department of Family Medicine, University of North Carolina at Chapel Hill.

Issue
Clinician Reviews - 24(2)
Publications
Topics
Page Number
16,19
Legacy Keywords
purls, steroids, COPD, acute COPD, glucocorticoid, exacerbation, chronic obstructive pulmonary disease, cough, shortness of breath, oral steroids, duration of treatment, duration
Sections
Author and Disclosure Information

Gary N. Asher, MD, MPH, Anne Mounsey, MD

Gary N. Asher and Anne Mounsey are with the Department of Family Medicine, University of North Carolina at Chapel Hill.

Author and Disclosure Information

Gary N. Asher, MD, MPH, Anne Mounsey, MD

Gary N. Asher and Anne Mounsey are with the Department of Family Medicine, University of North Carolina at Chapel Hill.

Not only was a shorter course of glucocorticoid therapy as effective as a 14-day regimen, but there was no difference in the time to next exacerbation.
Not only was a shorter course of glucocorticoid therapy as effective as a 14-day regimen, but there was no difference in the time to next exacerbation.

PRACTICE CHANGER

Prescribe a five-day regimen of glucocorticoid therapy for acute exacerbations of chronic obstructive pulmonary disease (COPD); the shorter course of treatment appears to be as effective as a 14-day regimen.1

Strength of ­recommendation

B: Based on a single well-­designed randomized controlled trial (RCT).1

ILLUSTRATIVE CASE

A 55-year-old man with COPD presents to the emergency department (ED) with progressive shortness of breath, cough, and sputum production in the past four days. He is diagnosed with a COPD exacerbation, treated with corticosteroids, and admitted to the hospital. His inpatient treatment includes antibiotics, inhaled albuterol and ipratropium, supplemental oxygen, and oral corticosteroids.

How many days should he take oral steroids?

 

 

 

Severe exacerbations of COPD are independently associated with mortality,2 regardless of baseline severity. Guidelines and systematic reviews highlight the importance of using oral glucocorticoids in the management of acute COPD exacerbations, as the drugs have been found to shorten recovery time and length of hospital stay, improve lung function, and reduce the risk for early relapse and treatment failure.3-5 What is not clear is how long the course of oral steroids should be.

What we know (and don’t know) about duration

Data supporting a 14-day course of steroids versus a longer (eight-week) duration come from the Systemic Corticosteroids in COPD Exacerbations trial.6 Global Initiative for Chronic Obstructive Lung Disease (GOLD) criteria suggest a 10- to 14-day regimen (30 to 40 mg/d) but acknowledge that there is a lack of data from clinical and observational studies to support this recommendation.3 A recent Cochrane review compared a short course of treatment (three to seven days) with a longer regimen (10 to 15 days) and found that the evidence to support a clinical practice change was inconclusive.5

The study detailed in this PURL—a double-blind RCT comparing five-day with 14-day oral steroid treatment in patients ­hospitalized for acute COPD exacerbation—had more definitive results.1

Continue reading for the study summary...

 

 

STUDY SUMMARY

Shorter and longer regimens produce equal results

Leuppi et al1 used noninferiority methodology to compare a five- and a 14-day course of prednisone 40 mg/d to treat patients with COPD exacerbations. A patient was considered to have an exacerbation if he or she had a change from baseline in two or more of the following: dyspnea, cough, sputum quantity, or purulence.

Participants were patients who presented to the EDs of five Swiss teaching hospitals between March 2006 and February 2011. To be eligible, individuals had to be 40 or older and have at least 20 pack-years of smoking. Exclusion criteria included asthma, mild obstruction (FEV1/FVC > 70%), pneumonia, an estimated survival of less than six months, pregnancy, and lactation.

All the participants (N = 311) received 40 mg methylprednisolone intravenously on day 1, followed by prednisone 40 mg orally on days 2 through 5. The researchers then randomly divided participants into two groups: One group continued to take prednisone 40 mg/d and the other group received a matching placebo for an additional nine days. Participants in both groups also received antibiotics for seven days, twice-daily inhaled steroids, daily tiotropium, and nebulized albuterol, as needed; additional oral glucocorticoids could be administered, as well, at the discretion of the treating physicians.

The primary outcome was the time to the next COPD exacerbation, up to 180 days. Noninferiority between the groups was defined as no more than a 15% absolute increase in exacerbations. The dropout rate was 5.7%, evenly divided between groups. Intention to treat and per-protocol analyses were conducted, and hazard ratios (HRs) were calculated using the Kaplan-Meier method and Cox proportional hazards models.

The time to next COPD exacerbation did not differ between the study groups: 56 days for those on the five-day steroid regimen versus 57 days for those on the
14-day regimen in the intention-to-treat analysis (HR, 0.95). Sensitivity analyses adjusting for baseline characteristics provided similar results, as did the per-protocol analysis.

Secondary outcomes (overall survival; need for mechanical ventilation; need for additional corticosteroids; and clinical performance measures, such as dyspnea score and quality of life) also did not differ between groups. Nor were there differences in hyperglycemia, worsening hypertension, infection, or other adverse effects typically associated with glucocorticoid use. The active treatment group took more than 400 mg more prednisone than the placebo group (mean, 793 mg vs 379 mg, respectively).

 

 

WHAT’S NEW?

Now we know: five days is enough

While randomized trials have found that glucocorticoids improve COPD symptoms, the optimal treatment dose and duration were not known. Indeed, current guidelines recommend treatment for more than five days.3 This trial clearly demonstrated that 40-mg prednisone for five days is at least as good as a 14-day treatment course. Furthermore, it is un­necessary to taper the short-course therapy, which simplifies the regimen.

CAVEATS

Will the results apply to those less severely ill?

More than 80% of patients with acute COPD exacerbations can be managed in an outpatient setting.3 However, participants in this trial were hospitalized for a median of 8.5 days, and most had severe or very severe COPD—and thus, were not fully representative of COPD patients typically seen in an outpatient practice. Yet patients with less severe disease should be at least as likely to ­respond to short-course steroids as those whose COPD is more ­severe.

It is important to note that participants in this study all received optimal guideline-based therapies during hospitalization, which may be difficult to achieve for some patients treated in an outpatient setting. Finally, treatment adherence observed during the hospitalization period in this trial is unlikely to be replicated in the outpatient setting.

CHALLENGES TO IMPLEMENTATION

Identifying patients who need steroids for a longer duration

For patients with new COPD exacerbations or those successfully treated using short-course therapy in the past, a five-day regimen may be appropriate. For those in whom prior attempts at short-course treatment have failed, however, a 14-day course of treatment may be more advisable. That said, no guidelines are available to help us determine which patients previously treated with a longer regimen will find the shorter course of treatment unsuccessful.

Continue for references...

 

 

REFERENCES

1. Leuppi JD, Schuetz P, Bingisser R, et al. Short-term vs conventional glucocorticoid therapy in acute exacerbations of chronic obstructive pulmonary disease: the REDUCE randomized clinical trial. JAMA. 2013;309:2223-2231.

2. Soler-Cataluna JJ, Martinez-Garcia MA, Roman Sanchez P, et al. Severe acute exacerbations and mortality in patients with chronic obstructive pulmonary disease. Thorax. 2005;60:925-931.

3. Global Initiative for Chronic Obstructive Lung Disease, Inc. The global strategy for diagnosis, management, and prevention of chronic obstructive pulmonary disease. www.goldcopd.org. Accessed January 9, 2014.

4. Quon BS, Gan WQ, Sin DD. Contemporary management of acute exacerbations of COPD: a systematic review and metaanalysis. Chest. 2008;133:756-766.

5. Walters JA, Wang W, Morley C, et al. Different durations of corticosteroid therapy for exacerbations of chronic obstructive pulmonary disease. Cochrane Database Syst Rev. 2011; (10):CD006897.

6. Niewoehner DE, Erbland ML, Deupree RH, et al. Effect of systemic glucocorticoids on exacerbations of chronic obstructive pulmonary disease. Department of Veterans Affairs Cooperative Study Group. N Engl J Med. 1999;340:1941-1947.

ACKNOWLEDGEMENT

The PURLs Surveillance System is supported in part by Grant Number UL 1RR 024999 from the National Center for Research Resources, a Clinical Translational Science Award to the University of Chicago. The content is solely the responsibility of the authors and does not necessarily represent the official views of the National Center for Research Resources or the National Institutes of Health.

Copyright © 2014. The Family Physicians Inquiries Network. All rights reserved.

Reprinted with permission from the Family Physicians Inquiries Network and The Journal of Family Practice. 2014;63(1):29-30, 32.

PRACTICE CHANGER

Prescribe a five-day regimen of glucocorticoid therapy for acute exacerbations of chronic obstructive pulmonary disease (COPD); the shorter course of treatment appears to be as effective as a 14-day regimen.1

Strength of ­recommendation

B: Based on a single well-­designed randomized controlled trial (RCT).1

ILLUSTRATIVE CASE

A 55-year-old man with COPD presents to the emergency department (ED) with progressive shortness of breath, cough, and sputum production in the past four days. He is diagnosed with a COPD exacerbation, treated with corticosteroids, and admitted to the hospital. His inpatient treatment includes antibiotics, inhaled albuterol and ipratropium, supplemental oxygen, and oral corticosteroids.

How many days should he take oral steroids?

 

 

 

Severe exacerbations of COPD are independently associated with mortality,2 regardless of baseline severity. Guidelines and systematic reviews highlight the importance of using oral glucocorticoids in the management of acute COPD exacerbations, as the drugs have been found to shorten recovery time and length of hospital stay, improve lung function, and reduce the risk for early relapse and treatment failure.3-5 What is not clear is how long the course of oral steroids should be.

What we know (and don’t know) about duration

Data supporting a 14-day course of steroids versus a longer (eight-week) duration come from the Systemic Corticosteroids in COPD Exacerbations trial.6 Global Initiative for Chronic Obstructive Lung Disease (GOLD) criteria suggest a 10- to 14-day regimen (30 to 40 mg/d) but acknowledge that there is a lack of data from clinical and observational studies to support this recommendation.3 A recent Cochrane review compared a short course of treatment (three to seven days) with a longer regimen (10 to 15 days) and found that the evidence to support a clinical practice change was inconclusive.5

The study detailed in this PURL—a double-blind RCT comparing five-day with 14-day oral steroid treatment in patients ­hospitalized for acute COPD exacerbation—had more definitive results.1

Continue reading for the study summary...

 

 

STUDY SUMMARY

Shorter and longer regimens produce equal results

Leuppi et al1 used noninferiority methodology to compare a five- and a 14-day course of prednisone 40 mg/d to treat patients with COPD exacerbations. A patient was considered to have an exacerbation if he or she had a change from baseline in two or more of the following: dyspnea, cough, sputum quantity, or purulence.

Participants were patients who presented to the EDs of five Swiss teaching hospitals between March 2006 and February 2011. To be eligible, individuals had to be 40 or older and have at least 20 pack-years of smoking. Exclusion criteria included asthma, mild obstruction (FEV1/FVC > 70%), pneumonia, an estimated survival of less than six months, pregnancy, and lactation.

All the participants (N = 311) received 40 mg methylprednisolone intravenously on day 1, followed by prednisone 40 mg orally on days 2 through 5. The researchers then randomly divided participants into two groups: One group continued to take prednisone 40 mg/d and the other group received a matching placebo for an additional nine days. Participants in both groups also received antibiotics for seven days, twice-daily inhaled steroids, daily tiotropium, and nebulized albuterol, as needed; additional oral glucocorticoids could be administered, as well, at the discretion of the treating physicians.

The primary outcome was the time to the next COPD exacerbation, up to 180 days. Noninferiority between the groups was defined as no more than a 15% absolute increase in exacerbations. The dropout rate was 5.7%, evenly divided between groups. Intention to treat and per-protocol analyses were conducted, and hazard ratios (HRs) were calculated using the Kaplan-Meier method and Cox proportional hazards models.

The time to next COPD exacerbation did not differ between the study groups: 56 days for those on the five-day steroid regimen versus 57 days for those on the
14-day regimen in the intention-to-treat analysis (HR, 0.95). Sensitivity analyses adjusting for baseline characteristics provided similar results, as did the per-protocol analysis.

Secondary outcomes (overall survival; need for mechanical ventilation; need for additional corticosteroids; and clinical performance measures, such as dyspnea score and quality of life) also did not differ between groups. Nor were there differences in hyperglycemia, worsening hypertension, infection, or other adverse effects typically associated with glucocorticoid use. The active treatment group took more than 400 mg more prednisone than the placebo group (mean, 793 mg vs 379 mg, respectively).

 

 

WHAT’S NEW?

Now we know: five days is enough

While randomized trials have found that glucocorticoids improve COPD symptoms, the optimal treatment dose and duration were not known. Indeed, current guidelines recommend treatment for more than five days.3 This trial clearly demonstrated that 40-mg prednisone for five days is at least as good as a 14-day treatment course. Furthermore, it is un­necessary to taper the short-course therapy, which simplifies the regimen.

CAVEATS

Will the results apply to those less severely ill?

More than 80% of patients with acute COPD exacerbations can be managed in an outpatient setting.3 However, participants in this trial were hospitalized for a median of 8.5 days, and most had severe or very severe COPD—and thus, were not fully representative of COPD patients typically seen in an outpatient practice. Yet patients with less severe disease should be at least as likely to ­respond to short-course steroids as those whose COPD is more ­severe.

It is important to note that participants in this study all received optimal guideline-based therapies during hospitalization, which may be difficult to achieve for some patients treated in an outpatient setting. Finally, treatment adherence observed during the hospitalization period in this trial is unlikely to be replicated in the outpatient setting.

CHALLENGES TO IMPLEMENTATION

Identifying patients who need steroids for a longer duration

For patients with new COPD exacerbations or those successfully treated using short-course therapy in the past, a five-day regimen may be appropriate. For those in whom prior attempts at short-course treatment have failed, however, a 14-day course of treatment may be more advisable. That said, no guidelines are available to help us determine which patients previously treated with a longer regimen will find the shorter course of treatment unsuccessful.

Continue for references...

 

 

REFERENCES

1. Leuppi JD, Schuetz P, Bingisser R, et al. Short-term vs conventional glucocorticoid therapy in acute exacerbations of chronic obstructive pulmonary disease: the REDUCE randomized clinical trial. JAMA. 2013;309:2223-2231.

2. Soler-Cataluna JJ, Martinez-Garcia MA, Roman Sanchez P, et al. Severe acute exacerbations and mortality in patients with chronic obstructive pulmonary disease. Thorax. 2005;60:925-931.

3. Global Initiative for Chronic Obstructive Lung Disease, Inc. The global strategy for diagnosis, management, and prevention of chronic obstructive pulmonary disease. www.goldcopd.org. Accessed January 9, 2014.

4. Quon BS, Gan WQ, Sin DD. Contemporary management of acute exacerbations of COPD: a systematic review and metaanalysis. Chest. 2008;133:756-766.

5. Walters JA, Wang W, Morley C, et al. Different durations of corticosteroid therapy for exacerbations of chronic obstructive pulmonary disease. Cochrane Database Syst Rev. 2011; (10):CD006897.

6. Niewoehner DE, Erbland ML, Deupree RH, et al. Effect of systemic glucocorticoids on exacerbations of chronic obstructive pulmonary disease. Department of Veterans Affairs Cooperative Study Group. N Engl J Med. 1999;340:1941-1947.

ACKNOWLEDGEMENT

The PURLs Surveillance System is supported in part by Grant Number UL 1RR 024999 from the National Center for Research Resources, a Clinical Translational Science Award to the University of Chicago. The content is solely the responsibility of the authors and does not necessarily represent the official views of the National Center for Research Resources or the National Institutes of Health.

Copyright © 2014. The Family Physicians Inquiries Network. All rights reserved.

Reprinted with permission from the Family Physicians Inquiries Network and The Journal of Family Practice. 2014;63(1):29-30, 32.

Issue
Clinician Reviews - 24(2)
Issue
Clinician Reviews - 24(2)
Page Number
16,19
Page Number
16,19
Publications
Publications
Topics
Article Type
Display Headline
Steroids for Acute COPD—But for How Long?
Display Headline
Steroids for Acute COPD—But for How Long?
Legacy Keywords
purls, steroids, COPD, acute COPD, glucocorticoid, exacerbation, chronic obstructive pulmonary disease, cough, shortness of breath, oral steroids, duration of treatment, duration
Legacy Keywords
purls, steroids, COPD, acute COPD, glucocorticoid, exacerbation, chronic obstructive pulmonary disease, cough, shortness of breath, oral steroids, duration of treatment, duration
Sections
Article Source

PURLs Copyright

Inside the Article

Treating migraine: The case for aspirin

Article Type
Changed
Tue, 07/14/2020 - 09:53
Display Headline
Treating migraine: The case for aspirin
Practice changer

Recommend aspirin 975 mg (3 adult tablets) as a viable first-line treatment for acute migraine. Consider prescribing metoclopramide 10 mg to be taken with aspirin to markedly decrease associated nausea and help achieve maximum symptom relief.1

Strength of recommendation

A: Based on a Cochrane meta-analysis of 13 good quality, randomized controlled trials (RCTs).

Kirthi V, Derry S, Moore RA. Aspirin with or without an antiemetic for acute migraine headaches in adults. Cochrane Database Syst Rev. 2013;(4):CD008041.

 

Illustrative case

During a routine physical, a 37-year-old patient asks you what she should take for her occasional migraines. She describes a unilateral headache with associated nausea, vomiting, phonophobia, and photophobia. What medication should you recommend?

Migraine headache affects more than 37 million Americans.2 Women are 3 times more likely than men to suffer from migraine, with the highest prevalence among those between the ages of 30 and 50 years.3,4 More than 50% of patients report that episodes cause severe impairment, resulting in an average loss of 4 to 6 workdays each year due to migraine.5,6

Do you recommend this low-cost option?

Although many patients try over-the-counter headache remedies for migraine, when they do seek medical care for this condition, most (67%) turn to their primary care providers.7 But despite a 2010 Cochrane review showing aspirin’s efficacy for acute migraine,8 our experience—based on discussions with physicians at numerous residency programs—suggests that family physicians are not likely to recommend it.

Further evidence of the underuse of aspirin for migraine comes from a 2013 review of national surveillance studies,5 which found that in 2009, triptans accounted for nearly 80% of antimigraine analgesics prescribed during office visits.5 Thus, when the Cochrane reviewers issued this update of the earlier meta-analysis, we welcomed the opportunity to feature a practice changer that might not be getting the “traction” it deserves.

STUDY SUMMARY: Multiple RCTs highlight aspirin's efficacy

The 2013 Cochrane reviewers used the same 13 good quality, double-blind RCTs involving 4222 participants as the earlier review; no new studies that warranted inclusion were found. A total of 5261 episodes of migraine of moderate to severe intensity were treated with either aspirin alone or aspirin plus the antiemetic metoclopramide.1

Five studies had placebo controls, 4 had active controls (sumatriptan, zolmitriptan, ibuprofen, acetaminophen plus codeine, and ergotamine plus caffeine among them), and 4 had both active and placebo controls. Primary outcomes were pain-free status at 2 hours and headache relief (defined as a reduction in pain from moderate or severe to none or mild without the use of rescue medication) at 2 hours. Sustained headache relief at 24 hours was a secondary outcome.

Patients self-assessed their headache pain, using either a 4-point categorical scale (none, mild, moderate, or severe) or a 100 mm visual analog scale. On the analog scale, <30 mm was considered mild or no pain; ≥30 mm was considered moderate or severe.

Study participants were 18 to 65 years of age (the mean age range was 37-44), and their symptoms met International Headache Society criteria for migraine with or without aura.9 All participants had migraine symptoms for ≥12 months, with between one and 6 attacks of moderate to severe intensity per month prior to the study period.

In 6 studies (n=2027), investigators compared either 900 or 1000 mg aspirin alone with placebo. For both primary outcomes, aspirin alone was superior to placebo, with a number needed to treat (NNT) of 8.1 for 2-hour pain-free status and 4.9 for 2-hour headache relief. In 3 studies (n=1142), aspirin was superior to placebo for 24-hour headache relief, with an NNT of 6.6. Aspirin plus metoclopramide was also better than placebo for primary and secondary outcomes, with an NNT of 8.8 for 2-hour pain-free status, 3.3 for 2-hour headache relief, and 6.2 for 24-hour headache relief. Based on subgroup analysis, aspirin plus metoclopramide was more effective than aspirin alone for 2-hour headache relief (P=.0131), but equivalent for 2-hour pain-free status and 24-hour headache relief. The addition of metoclopramide to aspirin significantly reduced nausea (P<.00006) and vomiting (P=.002).

In 2 studies (n=726), aspirin alone was equivalent to sumatriptan 50 mg for reaching pain-free and headache relief status at 2 hours. Two additional studies (n=523) compared aspirin plus metoclopramide with sumatriptan 100 mg and found them to be equal for 2-hour headache relief, but the aspirin combination was inferior to the triptan for pain-free status at 2 hours (n=528). Data were insufficient to compare the efficacy of aspirin with zolmitriptan, ibuprofen, or acetaminophen plus codeine.

There were no reports of gastrointestinal bleed or other serious adverse events attributable to aspirin therapy. Most adverse effects were mild or moderate disturbances of the digestive and nervous systems, with a number needed to harm of 34 (95% confidence interval, 18-340) for aspirin (with or without metoclopramide) vs placebo.

 

 

WHAT'S NEW?: A reminder of aspirin's efficacy in treating migraine

The addition of metoclopramide to aspirin reduces nausea and vomiting, but offers little, if any, benefit for headache/pain relief.The update of this meta-analysis confirms that high-dose aspirin (900-1000 mg) is an effective treatment for migraine headache in adults between the ages of 18 and 65 years. The addition of metoclopramide reduces nausea and vomiting, but offers little if any benefit for headache/pain relief.

 

CAVEATS: Lack of comparison with other treatments

Data were insufficient to compare the efficacy of aspirin with zolmitriptan, other nonsteroidal anti-inflammatory drugs alone, or acetaminophen plus codeine. Aspirin should be used with caution in patients with chronic renal disease and/or a history of peptic ulcer disease.

CHALLENGES TO IMPLEMENTATION: Patients want a prescription

Patients often expect a prescription when they visit a physician with complaints of migraine headache and may feel shortchanged if they’re told to take an aspirin. Providing a prescription for the antiemetic metoclopramide, as well as a brief explanation of the evidence indicating that aspirin is effective for migraine, may adequately address such expectations.

Acknowledgement

The PURLs Surveillance System was supported in part by Grant Number UL1RR024999 from the National Center for Research Resources, a Clinical Translational Science Award to the University of Chicago. The content is solely the responsibility of the authors and does not necessarily represent the official views of the National Center for Research Resources or the National Institutes of Health.

Files
References

1. Kirthi V, Derry S, Moore RA. Aspirin with or without an antiemetic for acute migraine headaches in adults. Cochrane Database Syst Rev. 2013;(4):CD008041.

2. Migraine. National Headache Foundation Web site. Available at: http://www.headaches.org/education/Headache_Topic_Sheets/Migraine. Accessed January 10, 2014.

3. Lipton RB, Stewart WF, Diamond S, et. al. Prevalence and burden of migraine in the United States: data from the American Migraine Study II. Headache. 2001;41:646-657.

4. Victor TW, Hu X, Campbell JC, et al. Migraine prevalence by age and sex in the United States: a life-span study. Cephalalgia. 2010;9:1065-1072.

5. Smitherman TA, Burch R, Sheikh H, et al. The prevalence, impact, and treatment of migraine and severe headaches in the United States: a review of statistics from national surveillance studies. Headache. 2013;53:427-436.

6. Hu XH, Markson LE, Lipton RB, et al. Burden of migraine in the United States: disability and economic costs. Arch Intern Med.1999;159:813-818.

7. Gibbs TS, Fleischer AB Jr, Feldman SR, et al. Health care utilization in patients with migraine: demographics and patterns of care in the ambulatory setting. Headache. 2003;43:330-335.

8. Kirthi V, Derry S, Moore RA, et al. Aspirin with or without an antiemetic for acute migraine headaches in adults. Cochrane Database Syst Rev. 2010;(4):CD008041.

9. The international classification of headache disorders. 2nd ed. Cephalalgia. 2004;24(suppl 1):S9-S160.

Article PDF
Author and Disclosure Information

Vickie F. Ingledue, MD
Anne Mounsey, MD

Department of Family Medicine, University of North Carolina at Chapel Hill

PURLs EDITOR
James Stevermer, MD, MSPH
Department of Family Medicine, University of Missouri-Columbia

Issue
The Journal of Family Practice - 63(2)
Publications
Topics
Page Number
94-96
Legacy Keywords
Vickie F. Ingledue; MD; Anne Mounsey; MD; migraine; aspirin; headache; sumatriptan; metoclopramide; triptan; ibuprofen; zolmitriptan; acetaminophen
Sections
Files
Files
Author and Disclosure Information

Vickie F. Ingledue, MD
Anne Mounsey, MD

Department of Family Medicine, University of North Carolina at Chapel Hill

PURLs EDITOR
James Stevermer, MD, MSPH
Department of Family Medicine, University of Missouri-Columbia

Author and Disclosure Information

Vickie F. Ingledue, MD
Anne Mounsey, MD

Department of Family Medicine, University of North Carolina at Chapel Hill

PURLs EDITOR
James Stevermer, MD, MSPH
Department of Family Medicine, University of Missouri-Columbia

Article PDF
Article PDF
Practice changer

Recommend aspirin 975 mg (3 adult tablets) as a viable first-line treatment for acute migraine. Consider prescribing metoclopramide 10 mg to be taken with aspirin to markedly decrease associated nausea and help achieve maximum symptom relief.1

Strength of recommendation

A: Based on a Cochrane meta-analysis of 13 good quality, randomized controlled trials (RCTs).

Kirthi V, Derry S, Moore RA. Aspirin with or without an antiemetic for acute migraine headaches in adults. Cochrane Database Syst Rev. 2013;(4):CD008041.

 

Illustrative case

During a routine physical, a 37-year-old patient asks you what she should take for her occasional migraines. She describes a unilateral headache with associated nausea, vomiting, phonophobia, and photophobia. What medication should you recommend?

Migraine headache affects more than 37 million Americans.2 Women are 3 times more likely than men to suffer from migraine, with the highest prevalence among those between the ages of 30 and 50 years.3,4 More than 50% of patients report that episodes cause severe impairment, resulting in an average loss of 4 to 6 workdays each year due to migraine.5,6

Do you recommend this low-cost option?

Although many patients try over-the-counter headache remedies for migraine, when they do seek medical care for this condition, most (67%) turn to their primary care providers.7 But despite a 2010 Cochrane review showing aspirin’s efficacy for acute migraine,8 our experience—based on discussions with physicians at numerous residency programs—suggests that family physicians are not likely to recommend it.

Further evidence of the underuse of aspirin for migraine comes from a 2013 review of national surveillance studies,5 which found that in 2009, triptans accounted for nearly 80% of antimigraine analgesics prescribed during office visits.5 Thus, when the Cochrane reviewers issued this update of the earlier meta-analysis, we welcomed the opportunity to feature a practice changer that might not be getting the “traction” it deserves.

STUDY SUMMARY: Multiple RCTs highlight aspirin's efficacy

The 2013 Cochrane reviewers used the same 13 good quality, double-blind RCTs involving 4222 participants as the earlier review; no new studies that warranted inclusion were found. A total of 5261 episodes of migraine of moderate to severe intensity were treated with either aspirin alone or aspirin plus the antiemetic metoclopramide.1

Five studies had placebo controls, 4 had active controls (sumatriptan, zolmitriptan, ibuprofen, acetaminophen plus codeine, and ergotamine plus caffeine among them), and 4 had both active and placebo controls. Primary outcomes were pain-free status at 2 hours and headache relief (defined as a reduction in pain from moderate or severe to none or mild without the use of rescue medication) at 2 hours. Sustained headache relief at 24 hours was a secondary outcome.

Patients self-assessed their headache pain, using either a 4-point categorical scale (none, mild, moderate, or severe) or a 100 mm visual analog scale. On the analog scale, <30 mm was considered mild or no pain; ≥30 mm was considered moderate or severe.

Study participants were 18 to 65 years of age (the mean age range was 37-44), and their symptoms met International Headache Society criteria for migraine with or without aura.9 All participants had migraine symptoms for ≥12 months, with between one and 6 attacks of moderate to severe intensity per month prior to the study period.

In 6 studies (n=2027), investigators compared either 900 or 1000 mg aspirin alone with placebo. For both primary outcomes, aspirin alone was superior to placebo, with a number needed to treat (NNT) of 8.1 for 2-hour pain-free status and 4.9 for 2-hour headache relief. In 3 studies (n=1142), aspirin was superior to placebo for 24-hour headache relief, with an NNT of 6.6. Aspirin plus metoclopramide was also better than placebo for primary and secondary outcomes, with an NNT of 8.8 for 2-hour pain-free status, 3.3 for 2-hour headache relief, and 6.2 for 24-hour headache relief. Based on subgroup analysis, aspirin plus metoclopramide was more effective than aspirin alone for 2-hour headache relief (P=.0131), but equivalent for 2-hour pain-free status and 24-hour headache relief. The addition of metoclopramide to aspirin significantly reduced nausea (P<.00006) and vomiting (P=.002).

In 2 studies (n=726), aspirin alone was equivalent to sumatriptan 50 mg for reaching pain-free and headache relief status at 2 hours. Two additional studies (n=523) compared aspirin plus metoclopramide with sumatriptan 100 mg and found them to be equal for 2-hour headache relief, but the aspirin combination was inferior to the triptan for pain-free status at 2 hours (n=528). Data were insufficient to compare the efficacy of aspirin with zolmitriptan, ibuprofen, or acetaminophen plus codeine.

There were no reports of gastrointestinal bleed or other serious adverse events attributable to aspirin therapy. Most adverse effects were mild or moderate disturbances of the digestive and nervous systems, with a number needed to harm of 34 (95% confidence interval, 18-340) for aspirin (with or without metoclopramide) vs placebo.

 

 

WHAT'S NEW?: A reminder of aspirin's efficacy in treating migraine

The addition of metoclopramide to aspirin reduces nausea and vomiting, but offers little, if any, benefit for headache/pain relief.The update of this meta-analysis confirms that high-dose aspirin (900-1000 mg) is an effective treatment for migraine headache in adults between the ages of 18 and 65 years. The addition of metoclopramide reduces nausea and vomiting, but offers little if any benefit for headache/pain relief.

 

CAVEATS: Lack of comparison with other treatments

Data were insufficient to compare the efficacy of aspirin with zolmitriptan, other nonsteroidal anti-inflammatory drugs alone, or acetaminophen plus codeine. Aspirin should be used with caution in patients with chronic renal disease and/or a history of peptic ulcer disease.

CHALLENGES TO IMPLEMENTATION: Patients want a prescription

Patients often expect a prescription when they visit a physician with complaints of migraine headache and may feel shortchanged if they’re told to take an aspirin. Providing a prescription for the antiemetic metoclopramide, as well as a brief explanation of the evidence indicating that aspirin is effective for migraine, may adequately address such expectations.

Acknowledgement

The PURLs Surveillance System was supported in part by Grant Number UL1RR024999 from the National Center for Research Resources, a Clinical Translational Science Award to the University of Chicago. The content is solely the responsibility of the authors and does not necessarily represent the official views of the National Center for Research Resources or the National Institutes of Health.

Practice changer

Recommend aspirin 975 mg (3 adult tablets) as a viable first-line treatment for acute migraine. Consider prescribing metoclopramide 10 mg to be taken with aspirin to markedly decrease associated nausea and help achieve maximum symptom relief.1

Strength of recommendation

A: Based on a Cochrane meta-analysis of 13 good quality, randomized controlled trials (RCTs).

Kirthi V, Derry S, Moore RA. Aspirin with or without an antiemetic for acute migraine headaches in adults. Cochrane Database Syst Rev. 2013;(4):CD008041.

 

Illustrative case

During a routine physical, a 37-year-old patient asks you what she should take for her occasional migraines. She describes a unilateral headache with associated nausea, vomiting, phonophobia, and photophobia. What medication should you recommend?

Migraine headache affects more than 37 million Americans.2 Women are 3 times more likely than men to suffer from migraine, with the highest prevalence among those between the ages of 30 and 50 years.3,4 More than 50% of patients report that episodes cause severe impairment, resulting in an average loss of 4 to 6 workdays each year due to migraine.5,6

Do you recommend this low-cost option?

Although many patients try over-the-counter headache remedies for migraine, when they do seek medical care for this condition, most (67%) turn to their primary care providers.7 But despite a 2010 Cochrane review showing aspirin’s efficacy for acute migraine,8 our experience—based on discussions with physicians at numerous residency programs—suggests that family physicians are not likely to recommend it.

Further evidence of the underuse of aspirin for migraine comes from a 2013 review of national surveillance studies,5 which found that in 2009, triptans accounted for nearly 80% of antimigraine analgesics prescribed during office visits.5 Thus, when the Cochrane reviewers issued this update of the earlier meta-analysis, we welcomed the opportunity to feature a practice changer that might not be getting the “traction” it deserves.

STUDY SUMMARY: Multiple RCTs highlight aspirin's efficacy

The 2013 Cochrane reviewers used the same 13 good quality, double-blind RCTs involving 4222 participants as the earlier review; no new studies that warranted inclusion were found. A total of 5261 episodes of migraine of moderate to severe intensity were treated with either aspirin alone or aspirin plus the antiemetic metoclopramide.1

Five studies had placebo controls, 4 had active controls (sumatriptan, zolmitriptan, ibuprofen, acetaminophen plus codeine, and ergotamine plus caffeine among them), and 4 had both active and placebo controls. Primary outcomes were pain-free status at 2 hours and headache relief (defined as a reduction in pain from moderate or severe to none or mild without the use of rescue medication) at 2 hours. Sustained headache relief at 24 hours was a secondary outcome.

Patients self-assessed their headache pain, using either a 4-point categorical scale (none, mild, moderate, or severe) or a 100 mm visual analog scale. On the analog scale, <30 mm was considered mild or no pain; ≥30 mm was considered moderate or severe.

Study participants were 18 to 65 years of age (the mean age range was 37-44), and their symptoms met International Headache Society criteria for migraine with or without aura.9 All participants had migraine symptoms for ≥12 months, with between one and 6 attacks of moderate to severe intensity per month prior to the study period.

In 6 studies (n=2027), investigators compared either 900 or 1000 mg aspirin alone with placebo. For both primary outcomes, aspirin alone was superior to placebo, with a number needed to treat (NNT) of 8.1 for 2-hour pain-free status and 4.9 for 2-hour headache relief. In 3 studies (n=1142), aspirin was superior to placebo for 24-hour headache relief, with an NNT of 6.6. Aspirin plus metoclopramide was also better than placebo for primary and secondary outcomes, with an NNT of 8.8 for 2-hour pain-free status, 3.3 for 2-hour headache relief, and 6.2 for 24-hour headache relief. Based on subgroup analysis, aspirin plus metoclopramide was more effective than aspirin alone for 2-hour headache relief (P=.0131), but equivalent for 2-hour pain-free status and 24-hour headache relief. The addition of metoclopramide to aspirin significantly reduced nausea (P<.00006) and vomiting (P=.002).

In 2 studies (n=726), aspirin alone was equivalent to sumatriptan 50 mg for reaching pain-free and headache relief status at 2 hours. Two additional studies (n=523) compared aspirin plus metoclopramide with sumatriptan 100 mg and found them to be equal for 2-hour headache relief, but the aspirin combination was inferior to the triptan for pain-free status at 2 hours (n=528). Data were insufficient to compare the efficacy of aspirin with zolmitriptan, ibuprofen, or acetaminophen plus codeine.

There were no reports of gastrointestinal bleed or other serious adverse events attributable to aspirin therapy. Most adverse effects were mild or moderate disturbances of the digestive and nervous systems, with a number needed to harm of 34 (95% confidence interval, 18-340) for aspirin (with or without metoclopramide) vs placebo.

 

 

WHAT'S NEW?: A reminder of aspirin's efficacy in treating migraine

The addition of metoclopramide to aspirin reduces nausea and vomiting, but offers little, if any, benefit for headache/pain relief.The update of this meta-analysis confirms that high-dose aspirin (900-1000 mg) is an effective treatment for migraine headache in adults between the ages of 18 and 65 years. The addition of metoclopramide reduces nausea and vomiting, but offers little if any benefit for headache/pain relief.

 

CAVEATS: Lack of comparison with other treatments

Data were insufficient to compare the efficacy of aspirin with zolmitriptan, other nonsteroidal anti-inflammatory drugs alone, or acetaminophen plus codeine. Aspirin should be used with caution in patients with chronic renal disease and/or a history of peptic ulcer disease.

CHALLENGES TO IMPLEMENTATION: Patients want a prescription

Patients often expect a prescription when they visit a physician with complaints of migraine headache and may feel shortchanged if they’re told to take an aspirin. Providing a prescription for the antiemetic metoclopramide, as well as a brief explanation of the evidence indicating that aspirin is effective for migraine, may adequately address such expectations.

Acknowledgement

The PURLs Surveillance System was supported in part by Grant Number UL1RR024999 from the National Center for Research Resources, a Clinical Translational Science Award to the University of Chicago. The content is solely the responsibility of the authors and does not necessarily represent the official views of the National Center for Research Resources or the National Institutes of Health.

References

1. Kirthi V, Derry S, Moore RA. Aspirin with or without an antiemetic for acute migraine headaches in adults. Cochrane Database Syst Rev. 2013;(4):CD008041.

2. Migraine. National Headache Foundation Web site. Available at: http://www.headaches.org/education/Headache_Topic_Sheets/Migraine. Accessed January 10, 2014.

3. Lipton RB, Stewart WF, Diamond S, et. al. Prevalence and burden of migraine in the United States: data from the American Migraine Study II. Headache. 2001;41:646-657.

4. Victor TW, Hu X, Campbell JC, et al. Migraine prevalence by age and sex in the United States: a life-span study. Cephalalgia. 2010;9:1065-1072.

5. Smitherman TA, Burch R, Sheikh H, et al. The prevalence, impact, and treatment of migraine and severe headaches in the United States: a review of statistics from national surveillance studies. Headache. 2013;53:427-436.

6. Hu XH, Markson LE, Lipton RB, et al. Burden of migraine in the United States: disability and economic costs. Arch Intern Med.1999;159:813-818.

7. Gibbs TS, Fleischer AB Jr, Feldman SR, et al. Health care utilization in patients with migraine: demographics and patterns of care in the ambulatory setting. Headache. 2003;43:330-335.

8. Kirthi V, Derry S, Moore RA, et al. Aspirin with or without an antiemetic for acute migraine headaches in adults. Cochrane Database Syst Rev. 2010;(4):CD008041.

9. The international classification of headache disorders. 2nd ed. Cephalalgia. 2004;24(suppl 1):S9-S160.

References

1. Kirthi V, Derry S, Moore RA. Aspirin with or without an antiemetic for acute migraine headaches in adults. Cochrane Database Syst Rev. 2013;(4):CD008041.

2. Migraine. National Headache Foundation Web site. Available at: http://www.headaches.org/education/Headache_Topic_Sheets/Migraine. Accessed January 10, 2014.

3. Lipton RB, Stewart WF, Diamond S, et. al. Prevalence and burden of migraine in the United States: data from the American Migraine Study II. Headache. 2001;41:646-657.

4. Victor TW, Hu X, Campbell JC, et al. Migraine prevalence by age and sex in the United States: a life-span study. Cephalalgia. 2010;9:1065-1072.

5. Smitherman TA, Burch R, Sheikh H, et al. The prevalence, impact, and treatment of migraine and severe headaches in the United States: a review of statistics from national surveillance studies. Headache. 2013;53:427-436.

6. Hu XH, Markson LE, Lipton RB, et al. Burden of migraine in the United States: disability and economic costs. Arch Intern Med.1999;159:813-818.

7. Gibbs TS, Fleischer AB Jr, Feldman SR, et al. Health care utilization in patients with migraine: demographics and patterns of care in the ambulatory setting. Headache. 2003;43:330-335.

8. Kirthi V, Derry S, Moore RA, et al. Aspirin with or without an antiemetic for acute migraine headaches in adults. Cochrane Database Syst Rev. 2010;(4):CD008041.

9. The international classification of headache disorders. 2nd ed. Cephalalgia. 2004;24(suppl 1):S9-S160.

Issue
The Journal of Family Practice - 63(2)
Issue
The Journal of Family Practice - 63(2)
Page Number
94-96
Page Number
94-96
Publications
Publications
Topics
Article Type
Display Headline
Treating migraine: The case for aspirin
Display Headline
Treating migraine: The case for aspirin
Legacy Keywords
Vickie F. Ingledue; MD; Anne Mounsey; MD; migraine; aspirin; headache; sumatriptan; metoclopramide; triptan; ibuprofen; zolmitriptan; acetaminophen
Legacy Keywords
Vickie F. Ingledue; MD; Anne Mounsey; MD; migraine; aspirin; headache; sumatriptan; metoclopramide; triptan; ibuprofen; zolmitriptan; acetaminophen
Sections
PURLs Copyright

Copyright © 2014 Family Physicians Inquiries Network. All rights reserved.

Disallow All Ads
Alternative CME
Use ProPublica
Hide sidebar & use full width
render the right sidebar.
Conference Recap Checkbox
Not Conference Recap
Clinical Edge
Display the Slideshow in this Article
Article PDF Media
Media Files

Steroids for acute COPD—but for how long?

Article Type
Changed
Tue, 07/14/2020 - 09:48
Display Headline
Steroids for acute COPD—but for how long?
Practice changer

Prescribe a 5-day regimen of glucocorticoid therapy for acute chronic obstructive pulmonary disease (COPD) exacerbations; the shorter course of treatment appears to be as effective as a 14-day regiment.1

Strength of recommendation

B: Based on a single well-design randomized controlled trial (RCT).

Leuppi JD, Schuetz P, Bingisser R, et al. Short-term vs conventional glucocorticoid therapy in acute exacerbations of chronic obstructive pulmonary disease: the REDUCE randomized clinical trial. JAMA. 2013;309:2223-2231.

 

Illustrative case

A 55-year-old man with COPD presents to the emergency department (ED) because of progressive shortness of breath, cough, and sputum production over the past 4 days. He is diagnosed with a COPD exacerbation, treated with corticosteroids, and admitted to the hospital. His inpatient treatment includes antibiotics, inhaled albuterol and ipratropium, supplemental oxygen, and oral corticosteroids.

How many days should he take oral steroids?

Severe exacerbations of COPD are independently associated with mortality,2 regardless of baseline severity. Guidelines and systematic reviews highlight the importance of using oral glucocorticoids in the management of acute COPD exacerbations, as the drugs have been found to shorten recovery time and length of hospital stay, improve lung function, and reduce the risk of early relapse and treatment failure.3-5 What is not clear is how long the course of oral steroids should be.

What we know (and don’t know) about duration

Data supporting a 14-day course of steroids vs a longer (8-week) duration comes from the Systemic Corticosteroids in COPD Exacerbations trial.6 Global Initiative for Chronic Obstructive Lung Disease (GOLD) criteria suggest a 10- to-14-day regimen (30-40 mg/d), but acknowledge that there is a lack of data from clinical and observational studies to support this recommendation.3 A recent Cochrane review compared a short course of treatment (3-7 days) with a longer regimen (10-15 days) and found that the evidence to support a clinical practice change was inconclusive.5

The study detailed in this PURL—a double-blind RCT comparing 5-day with 14-day oral steroid treatment in patients hospitalized for acute COPD exacerbation—had more definitive results.1

STUDY SUMMARY: Shorter and longer regimens produce equal results

Leuppi et al1 used noninferiority methodology to compare a 5- vs a 14-day course of prednisone (40 mg/d) to treat patients with COPD exacerbations. A patient was considered to have a COPD exacerbation if he or she had a change from baseline in ≥2 of the following: dyspnea, cough, sputum quantity, or purulence.

Participants were patients who came to the EDs of 5 Swiss teaching hospitals between March 2006 and February 2011. To be eligible, individuals had to be 40 years or older and have ≥20 pack-years of smoking. Exclusion criteria included asthma, mild obstruction (forced expiratory volume in one second/forced vital capacity >70%), pneumonia, an estimated survival <6 months, pregnancy, and lactation.

All the participants (N=311) received 40 mg methylprednisolone intravenously on Day 1, followed by prednisone 40 mg orally on Days 2 through 5. The researchers then randomly divided participants into 2 groups: One group continued to take prednisone 40 mg/d and the other group received a matching placebo for an additional 9 days. Participants in both groups also received antibiotics for 7 days, twice daily inhaled steroids, daily tiotropium, and nebulized albuterol, as needed; additional oral glucocorticoids could be administered, as well, at the discretion of the treating physicians.

This trial clearly demonstrated that 40 mg prednisone for 5 days is at least as good as a 14-day regimen. What's more, it is not necessary to taper the short-course therapy, which simplifies treatment. The primary outcome was the time to the next COPD exacerbation, up to 180 days. Noninferiority between the groups was defined as no more than a 15% absolute increase in exacerbations. The dropout rate was 5.7%, evenly divided between groups. Intention to treat and per-protocol analyses were conducted, and hazard ratios (HRs) were calculated using the Kaplan-Meier method and Cox proportional hazards models.

The time to next COPD exacerbation did not differ between the study groups: 56 days for those on the 5-day steroid regimen vs 57 days for those on the 14-day regimen in the intention to treat analysis (HR=0.95; 90% confidence interval, 0.70-1.29; P=.006). Sensitivity analyses adjusting for baseline characteristics provided similar results, as did the per-protocol analysis.

Secondary outcomes (overall survival; need for mechanical ventilation; need for additional corticosteroids; and clinical performance measures, such as dyspnea score and quality of life) also did not differ between groups. Nor were there differences in hyperglycemia, worsening hypertension, infection, or other adverse effects typically associated with glucocorticoid use. The active treatment group took >400 mg more prednisone than the placebo group (mean, 793 mg vs 379 mg; P<.001).

 

 

 

WHAT'S NEW?: Now we know: 5 days is enough

While randomized trials have found that glucocorticoids improve COPD symptoms, the optimal treatment dose and duration were not known. Indeed, current guidelines recommend treatment for >5 days.3 This trial clearly demonstrated that 40 mg prednisone for 5 days is at least as good as a 14-day treatment course. Furthermore, it is unnecessary to taper the short-course therapy, which simplifies the regimen.

CAVEATS: Will the results apply to those less severely ill?

More than 80% of patients with acute COPD exacerbations can be managed in an outpatient setting.3 However, participants in this trial were hospitalized for a median of 8.5 days, and most had severe or very severe COPD—and thus, were not fully representative of COPD patients typically seen in an outpatient practice. Yet patients with less severe disease should be at least as likely to respond to short-course steroids as those whose COPD is more severe.

It is important to note that participants in this study all received optimal guideline-based therapies during hospitalization, which may be difficult to achieve for some patients treated in an outpatient setting. Finally, treatment adherence observed during the hospitalization period in this trial is unlikely to be replicated in the outpatient setting.

CHALLENGES TO IMPLEMENTATION: Identifying patients who need steroids for a longer duration

For patients with new COPD exacerbations or those successfully treated using short-course therapy in the past, a 5-day regimen may be appropriate. For those who have failed prior attempts at short-course treatment, however, a 14-day course of treatment may be more advisable. That said, no guidelines are available to help us determine which of those who were previously treated with a longer regimen may fail on the shorter course of treatment.

Acknowledgement

The PURLs Surveillance System is supported in part by Grant Number UL 1RR 024999 from the National Center for Research Resources, a Clinical Translational Science Award to the University of Chicago. The content is solely the responsibility of the authors and does not necessarily represent the official views of the National Center for Research Resources or the National Institutes of Health.

Files
References

1. Leuppi JD, Schuetz P, Bingisser R, et al. Short-term vs conventional glucocorticoid therapy in acute exacerbations of chronic obstructive pulmonary disease: the REDUCE randomized clinical trial. JAMA. 2013;309:2223-2231.

2. Soler-Cataluna JJ, Martinez-Garcia MA, Roman Sanchez P, et al. Severe acute exacerbations and mortality in patients with chronic obstructive pulmonary disease. Thorax. 2005;60:925-931.

3. Global Initiative for Chronic Obstructive Lung Disease, Inc. The global strategy for diagnosis, management, and prevention of chronic obstructive pulmonary disease. Available at: http://www.goldcopd.org/guidelines-global-strategy-for-diagnosismanagement.html. Accessed December 13, 2013.

4. Quon BS, Gan WQ, Sin DD. Contemporary management of acute exacerbations of COPD: a systematic review and metaanalysis. Chest. 2008;133:756-766.

5. Walters JA, Wang W, Morley C,et al. Different durations of corticosteroid therapy for exacerbations of chronic obstructive pulmonary disease. Cochrane Database Syst Rev. 2011;(10):CD006897.

6. Niewoehner DE, Erbland ML, Deupree RH, et al. Effect of systemic glucocorticoids on exacerbations of chronic obstructive pulmonary disease. Department of Veterans Affairs Cooperative Study Group. N Engl J Med. 1999;340:1941-1947.

Article PDF
Author and Disclosure Information

Gary N. Asher, MD, MPH
Anne Mounsey, MD

Department of Family Medicine, University of North Carolina at Chapel Hill

PURLs EDITOR
James Stevermer, MD, MSPH
Department of Family Community Medicine, University of Missouri - Columbia

Issue
The Journal of Family Practice - 63(1)
Publications
Topics
Page Number
29-30,32
Legacy Keywords
Gary N. Asher; MD; MPH; Anne Mounsey; MD; steroids; COPD; chronic obstructive pulmonary disease; glucocorticoid therapy; COPD exacerbations
Sections
Files
Files
Author and Disclosure Information

Gary N. Asher, MD, MPH
Anne Mounsey, MD

Department of Family Medicine, University of North Carolina at Chapel Hill

PURLs EDITOR
James Stevermer, MD, MSPH
Department of Family Community Medicine, University of Missouri - Columbia

Author and Disclosure Information

Gary N. Asher, MD, MPH
Anne Mounsey, MD

Department of Family Medicine, University of North Carolina at Chapel Hill

PURLs EDITOR
James Stevermer, MD, MSPH
Department of Family Community Medicine, University of Missouri - Columbia

Article PDF
Article PDF
Practice changer

Prescribe a 5-day regimen of glucocorticoid therapy for acute chronic obstructive pulmonary disease (COPD) exacerbations; the shorter course of treatment appears to be as effective as a 14-day regiment.1

Strength of recommendation

B: Based on a single well-design randomized controlled trial (RCT).

Leuppi JD, Schuetz P, Bingisser R, et al. Short-term vs conventional glucocorticoid therapy in acute exacerbations of chronic obstructive pulmonary disease: the REDUCE randomized clinical trial. JAMA. 2013;309:2223-2231.

 

Illustrative case

A 55-year-old man with COPD presents to the emergency department (ED) because of progressive shortness of breath, cough, and sputum production over the past 4 days. He is diagnosed with a COPD exacerbation, treated with corticosteroids, and admitted to the hospital. His inpatient treatment includes antibiotics, inhaled albuterol and ipratropium, supplemental oxygen, and oral corticosteroids.

How many days should he take oral steroids?

Severe exacerbations of COPD are independently associated with mortality,2 regardless of baseline severity. Guidelines and systematic reviews highlight the importance of using oral glucocorticoids in the management of acute COPD exacerbations, as the drugs have been found to shorten recovery time and length of hospital stay, improve lung function, and reduce the risk of early relapse and treatment failure.3-5 What is not clear is how long the course of oral steroids should be.

What we know (and don’t know) about duration

Data supporting a 14-day course of steroids vs a longer (8-week) duration comes from the Systemic Corticosteroids in COPD Exacerbations trial.6 Global Initiative for Chronic Obstructive Lung Disease (GOLD) criteria suggest a 10- to-14-day regimen (30-40 mg/d), but acknowledge that there is a lack of data from clinical and observational studies to support this recommendation.3 A recent Cochrane review compared a short course of treatment (3-7 days) with a longer regimen (10-15 days) and found that the evidence to support a clinical practice change was inconclusive.5

The study detailed in this PURL—a double-blind RCT comparing 5-day with 14-day oral steroid treatment in patients hospitalized for acute COPD exacerbation—had more definitive results.1

STUDY SUMMARY: Shorter and longer regimens produce equal results

Leuppi et al1 used noninferiority methodology to compare a 5- vs a 14-day course of prednisone (40 mg/d) to treat patients with COPD exacerbations. A patient was considered to have a COPD exacerbation if he or she had a change from baseline in ≥2 of the following: dyspnea, cough, sputum quantity, or purulence.

Participants were patients who came to the EDs of 5 Swiss teaching hospitals between March 2006 and February 2011. To be eligible, individuals had to be 40 years or older and have ≥20 pack-years of smoking. Exclusion criteria included asthma, mild obstruction (forced expiratory volume in one second/forced vital capacity >70%), pneumonia, an estimated survival <6 months, pregnancy, and lactation.

All the participants (N=311) received 40 mg methylprednisolone intravenously on Day 1, followed by prednisone 40 mg orally on Days 2 through 5. The researchers then randomly divided participants into 2 groups: One group continued to take prednisone 40 mg/d and the other group received a matching placebo for an additional 9 days. Participants in both groups also received antibiotics for 7 days, twice daily inhaled steroids, daily tiotropium, and nebulized albuterol, as needed; additional oral glucocorticoids could be administered, as well, at the discretion of the treating physicians.

This trial clearly demonstrated that 40 mg prednisone for 5 days is at least as good as a 14-day regimen. What's more, it is not necessary to taper the short-course therapy, which simplifies treatment. The primary outcome was the time to the next COPD exacerbation, up to 180 days. Noninferiority between the groups was defined as no more than a 15% absolute increase in exacerbations. The dropout rate was 5.7%, evenly divided between groups. Intention to treat and per-protocol analyses were conducted, and hazard ratios (HRs) were calculated using the Kaplan-Meier method and Cox proportional hazards models.

The time to next COPD exacerbation did not differ between the study groups: 56 days for those on the 5-day steroid regimen vs 57 days for those on the 14-day regimen in the intention to treat analysis (HR=0.95; 90% confidence interval, 0.70-1.29; P=.006). Sensitivity analyses adjusting for baseline characteristics provided similar results, as did the per-protocol analysis.

Secondary outcomes (overall survival; need for mechanical ventilation; need for additional corticosteroids; and clinical performance measures, such as dyspnea score and quality of life) also did not differ between groups. Nor were there differences in hyperglycemia, worsening hypertension, infection, or other adverse effects typically associated with glucocorticoid use. The active treatment group took >400 mg more prednisone than the placebo group (mean, 793 mg vs 379 mg; P<.001).

 

 

 

WHAT'S NEW?: Now we know: 5 days is enough

While randomized trials have found that glucocorticoids improve COPD symptoms, the optimal treatment dose and duration were not known. Indeed, current guidelines recommend treatment for >5 days.3 This trial clearly demonstrated that 40 mg prednisone for 5 days is at least as good as a 14-day treatment course. Furthermore, it is unnecessary to taper the short-course therapy, which simplifies the regimen.

CAVEATS: Will the results apply to those less severely ill?

More than 80% of patients with acute COPD exacerbations can be managed in an outpatient setting.3 However, participants in this trial were hospitalized for a median of 8.5 days, and most had severe or very severe COPD—and thus, were not fully representative of COPD patients typically seen in an outpatient practice. Yet patients with less severe disease should be at least as likely to respond to short-course steroids as those whose COPD is more severe.

It is important to note that participants in this study all received optimal guideline-based therapies during hospitalization, which may be difficult to achieve for some patients treated in an outpatient setting. Finally, treatment adherence observed during the hospitalization period in this trial is unlikely to be replicated in the outpatient setting.

CHALLENGES TO IMPLEMENTATION: Identifying patients who need steroids for a longer duration

For patients with new COPD exacerbations or those successfully treated using short-course therapy in the past, a 5-day regimen may be appropriate. For those who have failed prior attempts at short-course treatment, however, a 14-day course of treatment may be more advisable. That said, no guidelines are available to help us determine which of those who were previously treated with a longer regimen may fail on the shorter course of treatment.

Acknowledgement

The PURLs Surveillance System is supported in part by Grant Number UL 1RR 024999 from the National Center for Research Resources, a Clinical Translational Science Award to the University of Chicago. The content is solely the responsibility of the authors and does not necessarily represent the official views of the National Center for Research Resources or the National Institutes of Health.

Practice changer

Prescribe a 5-day regimen of glucocorticoid therapy for acute chronic obstructive pulmonary disease (COPD) exacerbations; the shorter course of treatment appears to be as effective as a 14-day regiment.1

Strength of recommendation

B: Based on a single well-design randomized controlled trial (RCT).

Leuppi JD, Schuetz P, Bingisser R, et al. Short-term vs conventional glucocorticoid therapy in acute exacerbations of chronic obstructive pulmonary disease: the REDUCE randomized clinical trial. JAMA. 2013;309:2223-2231.

 

Illustrative case

A 55-year-old man with COPD presents to the emergency department (ED) because of progressive shortness of breath, cough, and sputum production over the past 4 days. He is diagnosed with a COPD exacerbation, treated with corticosteroids, and admitted to the hospital. His inpatient treatment includes antibiotics, inhaled albuterol and ipratropium, supplemental oxygen, and oral corticosteroids.

How many days should he take oral steroids?

Severe exacerbations of COPD are independently associated with mortality,2 regardless of baseline severity. Guidelines and systematic reviews highlight the importance of using oral glucocorticoids in the management of acute COPD exacerbations, as the drugs have been found to shorten recovery time and length of hospital stay, improve lung function, and reduce the risk of early relapse and treatment failure.3-5 What is not clear is how long the course of oral steroids should be.

What we know (and don’t know) about duration

Data supporting a 14-day course of steroids vs a longer (8-week) duration comes from the Systemic Corticosteroids in COPD Exacerbations trial.6 Global Initiative for Chronic Obstructive Lung Disease (GOLD) criteria suggest a 10- to-14-day regimen (30-40 mg/d), but acknowledge that there is a lack of data from clinical and observational studies to support this recommendation.3 A recent Cochrane review compared a short course of treatment (3-7 days) with a longer regimen (10-15 days) and found that the evidence to support a clinical practice change was inconclusive.5

The study detailed in this PURL—a double-blind RCT comparing 5-day with 14-day oral steroid treatment in patients hospitalized for acute COPD exacerbation—had more definitive results.1

STUDY SUMMARY: Shorter and longer regimens produce equal results

Leuppi et al1 used noninferiority methodology to compare a 5- vs a 14-day course of prednisone (40 mg/d) to treat patients with COPD exacerbations. A patient was considered to have a COPD exacerbation if he or she had a change from baseline in ≥2 of the following: dyspnea, cough, sputum quantity, or purulence.

Participants were patients who came to the EDs of 5 Swiss teaching hospitals between March 2006 and February 2011. To be eligible, individuals had to be 40 years or older and have ≥20 pack-years of smoking. Exclusion criteria included asthma, mild obstruction (forced expiratory volume in one second/forced vital capacity >70%), pneumonia, an estimated survival <6 months, pregnancy, and lactation.

All the participants (N=311) received 40 mg methylprednisolone intravenously on Day 1, followed by prednisone 40 mg orally on Days 2 through 5. The researchers then randomly divided participants into 2 groups: One group continued to take prednisone 40 mg/d and the other group received a matching placebo for an additional 9 days. Participants in both groups also received antibiotics for 7 days, twice daily inhaled steroids, daily tiotropium, and nebulized albuterol, as needed; additional oral glucocorticoids could be administered, as well, at the discretion of the treating physicians.

This trial clearly demonstrated that 40 mg prednisone for 5 days is at least as good as a 14-day regimen. What's more, it is not necessary to taper the short-course therapy, which simplifies treatment. The primary outcome was the time to the next COPD exacerbation, up to 180 days. Noninferiority between the groups was defined as no more than a 15% absolute increase in exacerbations. The dropout rate was 5.7%, evenly divided between groups. Intention to treat and per-protocol analyses were conducted, and hazard ratios (HRs) were calculated using the Kaplan-Meier method and Cox proportional hazards models.

The time to next COPD exacerbation did not differ between the study groups: 56 days for those on the 5-day steroid regimen vs 57 days for those on the 14-day regimen in the intention to treat analysis (HR=0.95; 90% confidence interval, 0.70-1.29; P=.006). Sensitivity analyses adjusting for baseline characteristics provided similar results, as did the per-protocol analysis.

Secondary outcomes (overall survival; need for mechanical ventilation; need for additional corticosteroids; and clinical performance measures, such as dyspnea score and quality of life) also did not differ between groups. Nor were there differences in hyperglycemia, worsening hypertension, infection, or other adverse effects typically associated with glucocorticoid use. The active treatment group took >400 mg more prednisone than the placebo group (mean, 793 mg vs 379 mg; P<.001).

 

 

 

WHAT'S NEW?: Now we know: 5 days is enough

While randomized trials have found that glucocorticoids improve COPD symptoms, the optimal treatment dose and duration were not known. Indeed, current guidelines recommend treatment for >5 days.3 This trial clearly demonstrated that 40 mg prednisone for 5 days is at least as good as a 14-day treatment course. Furthermore, it is unnecessary to taper the short-course therapy, which simplifies the regimen.

CAVEATS: Will the results apply to those less severely ill?

More than 80% of patients with acute COPD exacerbations can be managed in an outpatient setting.3 However, participants in this trial were hospitalized for a median of 8.5 days, and most had severe or very severe COPD—and thus, were not fully representative of COPD patients typically seen in an outpatient practice. Yet patients with less severe disease should be at least as likely to respond to short-course steroids as those whose COPD is more severe.

It is important to note that participants in this study all received optimal guideline-based therapies during hospitalization, which may be difficult to achieve for some patients treated in an outpatient setting. Finally, treatment adherence observed during the hospitalization period in this trial is unlikely to be replicated in the outpatient setting.

CHALLENGES TO IMPLEMENTATION: Identifying patients who need steroids for a longer duration

For patients with new COPD exacerbations or those successfully treated using short-course therapy in the past, a 5-day regimen may be appropriate. For those who have failed prior attempts at short-course treatment, however, a 14-day course of treatment may be more advisable. That said, no guidelines are available to help us determine which of those who were previously treated with a longer regimen may fail on the shorter course of treatment.

Acknowledgement

The PURLs Surveillance System is supported in part by Grant Number UL 1RR 024999 from the National Center for Research Resources, a Clinical Translational Science Award to the University of Chicago. The content is solely the responsibility of the authors and does not necessarily represent the official views of the National Center for Research Resources or the National Institutes of Health.

References

1. Leuppi JD, Schuetz P, Bingisser R, et al. Short-term vs conventional glucocorticoid therapy in acute exacerbations of chronic obstructive pulmonary disease: the REDUCE randomized clinical trial. JAMA. 2013;309:2223-2231.

2. Soler-Cataluna JJ, Martinez-Garcia MA, Roman Sanchez P, et al. Severe acute exacerbations and mortality in patients with chronic obstructive pulmonary disease. Thorax. 2005;60:925-931.

3. Global Initiative for Chronic Obstructive Lung Disease, Inc. The global strategy for diagnosis, management, and prevention of chronic obstructive pulmonary disease. Available at: http://www.goldcopd.org/guidelines-global-strategy-for-diagnosismanagement.html. Accessed December 13, 2013.

4. Quon BS, Gan WQ, Sin DD. Contemporary management of acute exacerbations of COPD: a systematic review and metaanalysis. Chest. 2008;133:756-766.

5. Walters JA, Wang W, Morley C,et al. Different durations of corticosteroid therapy for exacerbations of chronic obstructive pulmonary disease. Cochrane Database Syst Rev. 2011;(10):CD006897.

6. Niewoehner DE, Erbland ML, Deupree RH, et al. Effect of systemic glucocorticoids on exacerbations of chronic obstructive pulmonary disease. Department of Veterans Affairs Cooperative Study Group. N Engl J Med. 1999;340:1941-1947.

References

1. Leuppi JD, Schuetz P, Bingisser R, et al. Short-term vs conventional glucocorticoid therapy in acute exacerbations of chronic obstructive pulmonary disease: the REDUCE randomized clinical trial. JAMA. 2013;309:2223-2231.

2. Soler-Cataluna JJ, Martinez-Garcia MA, Roman Sanchez P, et al. Severe acute exacerbations and mortality in patients with chronic obstructive pulmonary disease. Thorax. 2005;60:925-931.

3. Global Initiative for Chronic Obstructive Lung Disease, Inc. The global strategy for diagnosis, management, and prevention of chronic obstructive pulmonary disease. Available at: http://www.goldcopd.org/guidelines-global-strategy-for-diagnosismanagement.html. Accessed December 13, 2013.

4. Quon BS, Gan WQ, Sin DD. Contemporary management of acute exacerbations of COPD: a systematic review and metaanalysis. Chest. 2008;133:756-766.

5. Walters JA, Wang W, Morley C,et al. Different durations of corticosteroid therapy for exacerbations of chronic obstructive pulmonary disease. Cochrane Database Syst Rev. 2011;(10):CD006897.

6. Niewoehner DE, Erbland ML, Deupree RH, et al. Effect of systemic glucocorticoids on exacerbations of chronic obstructive pulmonary disease. Department of Veterans Affairs Cooperative Study Group. N Engl J Med. 1999;340:1941-1947.

Issue
The Journal of Family Practice - 63(1)
Issue
The Journal of Family Practice - 63(1)
Page Number
29-30,32
Page Number
29-30,32
Publications
Publications
Topics
Article Type
Display Headline
Steroids for acute COPD—but for how long?
Display Headline
Steroids for acute COPD—but for how long?
Legacy Keywords
Gary N. Asher; MD; MPH; Anne Mounsey; MD; steroids; COPD; chronic obstructive pulmonary disease; glucocorticoid therapy; COPD exacerbations
Legacy Keywords
Gary N. Asher; MD; MPH; Anne Mounsey; MD; steroids; COPD; chronic obstructive pulmonary disease; glucocorticoid therapy; COPD exacerbations
Sections
PURLs Copyright

Copyright © 2014 Family Physicians Inquiries Network. All rights reserved.

Disallow All Ads
Alternative CME
Use ProPublica
Hide sidebar & use full width
render the right sidebar.
Conference Recap Checkbox
Not Conference Recap
Clinical Edge
Display the Slideshow in this Article
Article PDF Media
Media Files

Mediterranean Diet: Higher Fat But Lower Risk

Article Type
Changed
Tue, 05/03/2022 - 15:51
Display Headline
Mediterranean Diet: Higher Fat But Lower Risk
For patients at high risk for cardiovascular disease, this diet may be the best bet.

PRACTICE CHANGER

Counsel patients at high risk for cardiovascular disease and stroke to follow a Mediterranean diet, which is associated with a 30% risk reduction.1

STRENGTH OF RECOMMENDATION

B: Based on one well-designed randomized controlled trial.1

ILLUSTRATIVE CASE

A 62-year-old patient with diabetes, obesity, and a family history of early-onset coronary artery disease is motivated to make significant lifestyle changes. You recommend moderate aerobic exercise (30 min five d/wk) but wonder whether a low-fat or a Mediterranean diet would be more effective in reducing her risk.

Cardiovascular disease (CVD), including heart disease and stroke, is the leading cause of mortality in the United States. CVD accounts for one in every three deaths,2 and stroke is a leading cause of long-term disability.2 The direct cost of treating CVD is estimated at $312.6 billion annually.2

Many modifiable risk factors contribute to CVD, including smoking, sedentary lifestyle, obesity, alcohol consumption, and poorly controlled chronic disease, as well as an unhealthy diet. A recent report from the American Heart Association suggests that 13% of deaths from CVD can be attributed to poor diet.2

Focus counseling on at-risk patients

Primary care providers (PCPs) often struggle to effectively counsel patients on behavioral change strategies, facing many barriers. Chief among them are the lack of time, training, and confidence in their counseling techniques, as well as a lack of patient motivation and readiness to change.3 In recognition of these barriers, the US Preventive Services Task Force recently recommended that PCPs focus behavioral counseling efforts on patients at high risk for heart disease.4

Large observational studies have found an association between trans fat and increased risk for CVD, as well as decreased risk for CVD in patients adhering to a Mediterranean diet.5-11 This type of diet typically includes a high intake of olive oil, fruit, nuts, vegetables, and cereals; moderate intake of fish and poultry; and low intake of dairy products, red meat, processed meats, and sweets. It also includes wine in moderation, consumed with meals.

Data on the physiologic properties of olive oil, including its antioxidant, vasodilating, and antiplatelet effects—as well as its effects on LDL cholesterol that may inhibit atherogenesis—support the link between a Mediterranean diet and a decreased risk for CVD found in the observational studies.12,13 Until recently, however, no randomized controlled trial (RCT) had compared the effect of a Mediterranean diet with that of a low-fat diet for primary prevention of CVD.

STUDY SUMMARY

Mediterranean diet significantly lowers risk

Prevencion con Dieta Mediterranea (PREDIMED) was a large RCT (N = 7,447) comparing two variations of a Mediterranean diet with a low-fat diet for primary prevention of CVD. This Spanish study enrolled men ages 55 to 80 and women ages 60 to 80 who were at high risk for CVD. The risk was based on either a diagnosis of type 2 diabetes or the presence of ≥ 3 major risk factors, including smoking, hypertension, elevated LDL cholesterol, low HDL cholesterol, overweight or obesity, and a family history of early heart disease.

Participants were randomly assigned to one of three dietary groups: One group followed a Mediterranean diet supplemented with ≥ 4 Tb of extra virgin olive oil per day; a second group was put on a Mediterranean diet supplemented by 30 g (about 1/3 cup) of mixed nuts daily; a third group (the controls) was advised to follow a low-fat diet. The majority of baseline characteristics and medications taken throughout the study were similar among all three groups.

Those in both Mediterranean diet groups were followed for a median of 4.8 years, during which they received quarterly dietary classes and individual and group counseling. The controls received baseline training, plus a leaflet about low-fat diets annually. In year 3, however, the researchers began giving the control group the same level of counseling as those in the Mediterranean diet groups to avoid confounding ­results.

Adherence to the diets was determined by a self-reported 14-item dietary screening questionnaire, plus urinary hydroxytyrosol and serum alpha-linoleic acid levels to assess for olive oil and mixed nut compliance. Self-reporting5 and biometric data indicated good compliance with the Mediterranean diets, and there was no difference found in levels of exercise among the groups.

After five years, those in the Mediterranean diet groups had consumed significantly more olive oil, nuts, vegetables, fruits, wine, legumes, seafood, and sofrito sauce (a popular tomato-based sauce) than the control group. Participants in the low-fat diet group had decreased their fat intake by 2%, while those in the Mediterranean groups had increased fat intake (by 2.03% for the olive oil group and 2.1% for the nut group). Overall, 37% of energy intake by those in the low-fat diet group came from fat (exceeding the < 30% of calories derived from fat intake that defines a low-fat diet), compared with 39% fat intake for those in both Mediterranean diet groups.

 

 

The primary outcome was a composite of MI, stroke, and death from cardiovascular causes, and there were clinically meaningful and statistically significant differences between the Mediterranean diet groups and the controls. The primary outcome rate for the supplemental olive oil group was 3.8%; 3.4% for the extra nuts group; and 4.4% for the controls. This represents a 30% reduction in risk for combined stroke, MI, and death due to cardiovascular causes for the Mediterranean diet groups (hazard ratio [HR], 0.7 and number needed to treat [NNT], 148 for the olive oil group; HR, 0.7 and NNT, 100 for the group consuming extra nuts). Similar benefits were found in the multivariable adjusted analyses. The results correspond to three fewer events (stroke, MI, or ­cardiovascular death) per 1,000 person-years for this high-risk population.

The only individual outcome that showed a significant decrease was stroke, with an NNT of 125 in both Mediterranean diet groups. Outcomes for the controls were similar before and after they began receiving quarterly counseling.

WHAT’S NEW?

Mediterranean diet is better than a lower-fat regimen

This study indicates that a Mediterranean diet, with increased intake of either olive oil or mixed nuts, is more protective against CVD than a recommended low-fat diet. It also shows that advising patients at high risk to follow a Mediterranean diet, providing dietary counseling, and monitoring them for adherence, rather than simply recommending a low-fat diet, can significantly decrease the risk for stroke.

Rates of CVD are higher in the US than in Spain, so implementing a Mediterranean diet on a large scale in this country has the potential to produce a greater response than that seen in this study.

CAVEATS

Would a true low-fat diet be a better comparison?

Although the control group’s diet was meant to be low fat, the participants did not achieve this, possibly due to the relatively low level of dietary education and personalized counseling at the start of the study. Their inability to reach the < 30% fat target could also reflect the difficulty patients have, in general, in decreasing fat content in their diet, which may mean the diet they maintained was a more realistic comparison.

This study used one brand of olive oil and a particular mixture of nuts (walnuts, hazelnuts, and almonds); it is possible that variations on either of these could affect the benefits of the diet.

CHALLENGES TO IMPLEMENTATION

Fitting a Mediterranean diet into an American lifestyle

The typical US diet is significantly different from that of most Spaniards. Americans may find it difficult to add either ≥ 4 Tb of olive oil or 30 g (1/3 cup) of nuts daily, for example, due to both cost and availability. Limited access to both individual and group counseling could be a barrier, as well.

On the other hand, this practice changer has the potential to simplify dietary counseling by allowing clinicians to focus on just one type of diet, for which there are many resources available both online and in print. We believe it makes sense to recommend a Mediterranean diet, while continuing to recommend increased exercise, smoking cessation, and improved control of chronic disease to lower patients’ risk for poor outcomes from CVD.

REFERENCES

1. Estruch R, Ros F, Salas-Salvado J, et al. Primary prevention of cardiovascular disease with a Mediterranean diet. N Engl J Med. 2013;368: 1279-1290.

2. Go AS, Mozaffarian D, Roger VL, et al; American Heart Association Statistics Committee and Stroke Statistics Subcommittee. Heart disease and stroke statistics—2013 update: a report from the American Heart Association. Circulation. 2013;127:e6-e245.

3. Kushner RF. Barriers to providing nutrition counseling by physicians: a survey of primary care practitioners. Prev Med. 1995;24:546-552.

4. USPSTF. Behavioral counseling to promote a healthful diet and physical activity for cardiovascular disease prevention in adults. www.uspreventiveservicestaskforce.org/uspstf/usp sphys.htm. Accessed December 18, 2013.

5. Hu FB, Stampfer MJ, Manson JE, et al. Dietary fat intake and the risk of coronary heart disease in women. N Engl J Med. 1997;337:1491-1499.

6. Oomen C, Ocké MC, Feskens JM, et al. Association between trans fatty acid intake and 10-year risk of coronary heart disease in the Zutphen Elderly Study: a prospective population-based study. Lancet. 2001;357:746-751.

7. de Lorgeril M, Salen P, Martin JL, et al. Mediterranean diet, traditional risk factors and the rate of cardiovascular complications after myocardial infarction. Final report of the Lyon Diet Heart Study. Circulation. 1999;99:779-785.

8. Knoops KT, de Groot LC, Kromhout D, et al. Mediterranean diet, lifestyle factors, and 10-year mortality in elderly European men and women: the HALE project. JAMA. 2004;292: 1433-1439.

 

 

9. Kris-Etherton P, Eckel RH, Howard BV, et al; Nutrition Committee Population Science Committee and Clinical Science Committee of the American Heart Association. Lyon Diet Heart Study. Benefits of a Mediterranean-style, National Education Program/AHA Step 1 Dietary Pattern on cardiovascular disease. Circulation. 2001;103:1823-1825.

10. Panagiotakos DB, Chrysohoou C, Pitsavos C, et al. The association of Mediterranean diet with lower risk of acute coronary syndromes, in hypertensive subjects. Int J Cardiol. 2002; 82:141-147.

11. Panagiotakos DB, Pitsavos C, Chrysohoou C, et al. The role of traditional Mediterranean-type of diet and lifestyle, in the development of acute coronary syndromes: preliminary results from CARDIO 2000 study. Centr Eur J Public Health. 2002;10:11-15.

12. Esposito K, Marfella R, Ciotola M, et al. Effect of a Mediterranean-style diet on endothelial dysfunction and markers of vascular inflammation in the metabolic syndrome: a randomized trial. JAMA. 2004;292:1440-1446.

13. Vincent-Baudry S, Defoort C, Gerber M, et al. The Medi-RIVAGE study: reduction of cardiovascular disease risk factors after a 3-mo intervention with a Mediterranean-type diet or a low-fat diet. Am J Clin Nutr. 2005;82: 964-971.

Acknowledgement

The PURLs Surveillance System is supported in part by Grant Number UL 1RR 024999 from the National Center for Research Resources, a Clinical Translational Science Award to the University of Chicago. The content is solely the responsibility of the authors and does not necessarily represent the official views of the National Center for Research Resources or the National Institutes of Health.

Copyright © 2013. The Family Physicians Inquiries Network. All rights reserved.

Reprinted with permission from the Family Physicians Inquiries Network and The Journal of Family Practice. 2013;62(12):745-746, 748.

Author and Disclosure Information

Mark Gwynne, DO, Anne Mounsey, MD

Issue
Clinician Reviews - 24(1)
Publications
Topics
Page Number
18-20
Legacy Keywords
PURLs, practice changer, CVD, cardiovascular disease, Mediterranean diet, best diet, low-fat, fat, diet
Sections
Author and Disclosure Information

Mark Gwynne, DO, Anne Mounsey, MD

Author and Disclosure Information

Mark Gwynne, DO, Anne Mounsey, MD

For patients at high risk for cardiovascular disease, this diet may be the best bet.
For patients at high risk for cardiovascular disease, this diet may be the best bet.

PRACTICE CHANGER

Counsel patients at high risk for cardiovascular disease and stroke to follow a Mediterranean diet, which is associated with a 30% risk reduction.1

STRENGTH OF RECOMMENDATION

B: Based on one well-designed randomized controlled trial.1

ILLUSTRATIVE CASE

A 62-year-old patient with diabetes, obesity, and a family history of early-onset coronary artery disease is motivated to make significant lifestyle changes. You recommend moderate aerobic exercise (30 min five d/wk) but wonder whether a low-fat or a Mediterranean diet would be more effective in reducing her risk.

Cardiovascular disease (CVD), including heart disease and stroke, is the leading cause of mortality in the United States. CVD accounts for one in every three deaths,2 and stroke is a leading cause of long-term disability.2 The direct cost of treating CVD is estimated at $312.6 billion annually.2

Many modifiable risk factors contribute to CVD, including smoking, sedentary lifestyle, obesity, alcohol consumption, and poorly controlled chronic disease, as well as an unhealthy diet. A recent report from the American Heart Association suggests that 13% of deaths from CVD can be attributed to poor diet.2

Focus counseling on at-risk patients

Primary care providers (PCPs) often struggle to effectively counsel patients on behavioral change strategies, facing many barriers. Chief among them are the lack of time, training, and confidence in their counseling techniques, as well as a lack of patient motivation and readiness to change.3 In recognition of these barriers, the US Preventive Services Task Force recently recommended that PCPs focus behavioral counseling efforts on patients at high risk for heart disease.4

Large observational studies have found an association between trans fat and increased risk for CVD, as well as decreased risk for CVD in patients adhering to a Mediterranean diet.5-11 This type of diet typically includes a high intake of olive oil, fruit, nuts, vegetables, and cereals; moderate intake of fish and poultry; and low intake of dairy products, red meat, processed meats, and sweets. It also includes wine in moderation, consumed with meals.

Data on the physiologic properties of olive oil, including its antioxidant, vasodilating, and antiplatelet effects—as well as its effects on LDL cholesterol that may inhibit atherogenesis—support the link between a Mediterranean diet and a decreased risk for CVD found in the observational studies.12,13 Until recently, however, no randomized controlled trial (RCT) had compared the effect of a Mediterranean diet with that of a low-fat diet for primary prevention of CVD.

STUDY SUMMARY

Mediterranean diet significantly lowers risk

Prevencion con Dieta Mediterranea (PREDIMED) was a large RCT (N = 7,447) comparing two variations of a Mediterranean diet with a low-fat diet for primary prevention of CVD. This Spanish study enrolled men ages 55 to 80 and women ages 60 to 80 who were at high risk for CVD. The risk was based on either a diagnosis of type 2 diabetes or the presence of ≥ 3 major risk factors, including smoking, hypertension, elevated LDL cholesterol, low HDL cholesterol, overweight or obesity, and a family history of early heart disease.

Participants were randomly assigned to one of three dietary groups: One group followed a Mediterranean diet supplemented with ≥ 4 Tb of extra virgin olive oil per day; a second group was put on a Mediterranean diet supplemented by 30 g (about 1/3 cup) of mixed nuts daily; a third group (the controls) was advised to follow a low-fat diet. The majority of baseline characteristics and medications taken throughout the study were similar among all three groups.

Those in both Mediterranean diet groups were followed for a median of 4.8 years, during which they received quarterly dietary classes and individual and group counseling. The controls received baseline training, plus a leaflet about low-fat diets annually. In year 3, however, the researchers began giving the control group the same level of counseling as those in the Mediterranean diet groups to avoid confounding ­results.

Adherence to the diets was determined by a self-reported 14-item dietary screening questionnaire, plus urinary hydroxytyrosol and serum alpha-linoleic acid levels to assess for olive oil and mixed nut compliance. Self-reporting5 and biometric data indicated good compliance with the Mediterranean diets, and there was no difference found in levels of exercise among the groups.

After five years, those in the Mediterranean diet groups had consumed significantly more olive oil, nuts, vegetables, fruits, wine, legumes, seafood, and sofrito sauce (a popular tomato-based sauce) than the control group. Participants in the low-fat diet group had decreased their fat intake by 2%, while those in the Mediterranean groups had increased fat intake (by 2.03% for the olive oil group and 2.1% for the nut group). Overall, 37% of energy intake by those in the low-fat diet group came from fat (exceeding the < 30% of calories derived from fat intake that defines a low-fat diet), compared with 39% fat intake for those in both Mediterranean diet groups.

 

 

The primary outcome was a composite of MI, stroke, and death from cardiovascular causes, and there were clinically meaningful and statistically significant differences between the Mediterranean diet groups and the controls. The primary outcome rate for the supplemental olive oil group was 3.8%; 3.4% for the extra nuts group; and 4.4% for the controls. This represents a 30% reduction in risk for combined stroke, MI, and death due to cardiovascular causes for the Mediterranean diet groups (hazard ratio [HR], 0.7 and number needed to treat [NNT], 148 for the olive oil group; HR, 0.7 and NNT, 100 for the group consuming extra nuts). Similar benefits were found in the multivariable adjusted analyses. The results correspond to three fewer events (stroke, MI, or ­cardiovascular death) per 1,000 person-years for this high-risk population.

The only individual outcome that showed a significant decrease was stroke, with an NNT of 125 in both Mediterranean diet groups. Outcomes for the controls were similar before and after they began receiving quarterly counseling.

WHAT’S NEW?

Mediterranean diet is better than a lower-fat regimen

This study indicates that a Mediterranean diet, with increased intake of either olive oil or mixed nuts, is more protective against CVD than a recommended low-fat diet. It also shows that advising patients at high risk to follow a Mediterranean diet, providing dietary counseling, and monitoring them for adherence, rather than simply recommending a low-fat diet, can significantly decrease the risk for stroke.

Rates of CVD are higher in the US than in Spain, so implementing a Mediterranean diet on a large scale in this country has the potential to produce a greater response than that seen in this study.

CAVEATS

Would a true low-fat diet be a better comparison?

Although the control group’s diet was meant to be low fat, the participants did not achieve this, possibly due to the relatively low level of dietary education and personalized counseling at the start of the study. Their inability to reach the < 30% fat target could also reflect the difficulty patients have, in general, in decreasing fat content in their diet, which may mean the diet they maintained was a more realistic comparison.

This study used one brand of olive oil and a particular mixture of nuts (walnuts, hazelnuts, and almonds); it is possible that variations on either of these could affect the benefits of the diet.

CHALLENGES TO IMPLEMENTATION

Fitting a Mediterranean diet into an American lifestyle

The typical US diet is significantly different from that of most Spaniards. Americans may find it difficult to add either ≥ 4 Tb of olive oil or 30 g (1/3 cup) of nuts daily, for example, due to both cost and availability. Limited access to both individual and group counseling could be a barrier, as well.

On the other hand, this practice changer has the potential to simplify dietary counseling by allowing clinicians to focus on just one type of diet, for which there are many resources available both online and in print. We believe it makes sense to recommend a Mediterranean diet, while continuing to recommend increased exercise, smoking cessation, and improved control of chronic disease to lower patients’ risk for poor outcomes from CVD.

REFERENCES

1. Estruch R, Ros F, Salas-Salvado J, et al. Primary prevention of cardiovascular disease with a Mediterranean diet. N Engl J Med. 2013;368: 1279-1290.

2. Go AS, Mozaffarian D, Roger VL, et al; American Heart Association Statistics Committee and Stroke Statistics Subcommittee. Heart disease and stroke statistics—2013 update: a report from the American Heart Association. Circulation. 2013;127:e6-e245.

3. Kushner RF. Barriers to providing nutrition counseling by physicians: a survey of primary care practitioners. Prev Med. 1995;24:546-552.

4. USPSTF. Behavioral counseling to promote a healthful diet and physical activity for cardiovascular disease prevention in adults. www.uspreventiveservicestaskforce.org/uspstf/usp sphys.htm. Accessed December 18, 2013.

5. Hu FB, Stampfer MJ, Manson JE, et al. Dietary fat intake and the risk of coronary heart disease in women. N Engl J Med. 1997;337:1491-1499.

6. Oomen C, Ocké MC, Feskens JM, et al. Association between trans fatty acid intake and 10-year risk of coronary heart disease in the Zutphen Elderly Study: a prospective population-based study. Lancet. 2001;357:746-751.

7. de Lorgeril M, Salen P, Martin JL, et al. Mediterranean diet, traditional risk factors and the rate of cardiovascular complications after myocardial infarction. Final report of the Lyon Diet Heart Study. Circulation. 1999;99:779-785.

8. Knoops KT, de Groot LC, Kromhout D, et al. Mediterranean diet, lifestyle factors, and 10-year mortality in elderly European men and women: the HALE project. JAMA. 2004;292: 1433-1439.

 

 

9. Kris-Etherton P, Eckel RH, Howard BV, et al; Nutrition Committee Population Science Committee and Clinical Science Committee of the American Heart Association. Lyon Diet Heart Study. Benefits of a Mediterranean-style, National Education Program/AHA Step 1 Dietary Pattern on cardiovascular disease. Circulation. 2001;103:1823-1825.

10. Panagiotakos DB, Chrysohoou C, Pitsavos C, et al. The association of Mediterranean diet with lower risk of acute coronary syndromes, in hypertensive subjects. Int J Cardiol. 2002; 82:141-147.

11. Panagiotakos DB, Pitsavos C, Chrysohoou C, et al. The role of traditional Mediterranean-type of diet and lifestyle, in the development of acute coronary syndromes: preliminary results from CARDIO 2000 study. Centr Eur J Public Health. 2002;10:11-15.

12. Esposito K, Marfella R, Ciotola M, et al. Effect of a Mediterranean-style diet on endothelial dysfunction and markers of vascular inflammation in the metabolic syndrome: a randomized trial. JAMA. 2004;292:1440-1446.

13. Vincent-Baudry S, Defoort C, Gerber M, et al. The Medi-RIVAGE study: reduction of cardiovascular disease risk factors after a 3-mo intervention with a Mediterranean-type diet or a low-fat diet. Am J Clin Nutr. 2005;82: 964-971.

Acknowledgement

The PURLs Surveillance System is supported in part by Grant Number UL 1RR 024999 from the National Center for Research Resources, a Clinical Translational Science Award to the University of Chicago. The content is solely the responsibility of the authors and does not necessarily represent the official views of the National Center for Research Resources or the National Institutes of Health.

Copyright © 2013. The Family Physicians Inquiries Network. All rights reserved.

Reprinted with permission from the Family Physicians Inquiries Network and The Journal of Family Practice. 2013;62(12):745-746, 748.

PRACTICE CHANGER

Counsel patients at high risk for cardiovascular disease and stroke to follow a Mediterranean diet, which is associated with a 30% risk reduction.1

STRENGTH OF RECOMMENDATION

B: Based on one well-designed randomized controlled trial.1

ILLUSTRATIVE CASE

A 62-year-old patient with diabetes, obesity, and a family history of early-onset coronary artery disease is motivated to make significant lifestyle changes. You recommend moderate aerobic exercise (30 min five d/wk) but wonder whether a low-fat or a Mediterranean diet would be more effective in reducing her risk.

Cardiovascular disease (CVD), including heart disease and stroke, is the leading cause of mortality in the United States. CVD accounts for one in every three deaths,2 and stroke is a leading cause of long-term disability.2 The direct cost of treating CVD is estimated at $312.6 billion annually.2

Many modifiable risk factors contribute to CVD, including smoking, sedentary lifestyle, obesity, alcohol consumption, and poorly controlled chronic disease, as well as an unhealthy diet. A recent report from the American Heart Association suggests that 13% of deaths from CVD can be attributed to poor diet.2

Focus counseling on at-risk patients

Primary care providers (PCPs) often struggle to effectively counsel patients on behavioral change strategies, facing many barriers. Chief among them are the lack of time, training, and confidence in their counseling techniques, as well as a lack of patient motivation and readiness to change.3 In recognition of these barriers, the US Preventive Services Task Force recently recommended that PCPs focus behavioral counseling efforts on patients at high risk for heart disease.4

Large observational studies have found an association between trans fat and increased risk for CVD, as well as decreased risk for CVD in patients adhering to a Mediterranean diet.5-11 This type of diet typically includes a high intake of olive oil, fruit, nuts, vegetables, and cereals; moderate intake of fish and poultry; and low intake of dairy products, red meat, processed meats, and sweets. It also includes wine in moderation, consumed with meals.

Data on the physiologic properties of olive oil, including its antioxidant, vasodilating, and antiplatelet effects—as well as its effects on LDL cholesterol that may inhibit atherogenesis—support the link between a Mediterranean diet and a decreased risk for CVD found in the observational studies.12,13 Until recently, however, no randomized controlled trial (RCT) had compared the effect of a Mediterranean diet with that of a low-fat diet for primary prevention of CVD.

STUDY SUMMARY

Mediterranean diet significantly lowers risk

Prevencion con Dieta Mediterranea (PREDIMED) was a large RCT (N = 7,447) comparing two variations of a Mediterranean diet with a low-fat diet for primary prevention of CVD. This Spanish study enrolled men ages 55 to 80 and women ages 60 to 80 who were at high risk for CVD. The risk was based on either a diagnosis of type 2 diabetes or the presence of ≥ 3 major risk factors, including smoking, hypertension, elevated LDL cholesterol, low HDL cholesterol, overweight or obesity, and a family history of early heart disease.

Participants were randomly assigned to one of three dietary groups: One group followed a Mediterranean diet supplemented with ≥ 4 Tb of extra virgin olive oil per day; a second group was put on a Mediterranean diet supplemented by 30 g (about 1/3 cup) of mixed nuts daily; a third group (the controls) was advised to follow a low-fat diet. The majority of baseline characteristics and medications taken throughout the study were similar among all three groups.

Those in both Mediterranean diet groups were followed for a median of 4.8 years, during which they received quarterly dietary classes and individual and group counseling. The controls received baseline training, plus a leaflet about low-fat diets annually. In year 3, however, the researchers began giving the control group the same level of counseling as those in the Mediterranean diet groups to avoid confounding ­results.

Adherence to the diets was determined by a self-reported 14-item dietary screening questionnaire, plus urinary hydroxytyrosol and serum alpha-linoleic acid levels to assess for olive oil and mixed nut compliance. Self-reporting5 and biometric data indicated good compliance with the Mediterranean diets, and there was no difference found in levels of exercise among the groups.

After five years, those in the Mediterranean diet groups had consumed significantly more olive oil, nuts, vegetables, fruits, wine, legumes, seafood, and sofrito sauce (a popular tomato-based sauce) than the control group. Participants in the low-fat diet group had decreased their fat intake by 2%, while those in the Mediterranean groups had increased fat intake (by 2.03% for the olive oil group and 2.1% for the nut group). Overall, 37% of energy intake by those in the low-fat diet group came from fat (exceeding the < 30% of calories derived from fat intake that defines a low-fat diet), compared with 39% fat intake for those in both Mediterranean diet groups.

 

 

The primary outcome was a composite of MI, stroke, and death from cardiovascular causes, and there were clinically meaningful and statistically significant differences between the Mediterranean diet groups and the controls. The primary outcome rate for the supplemental olive oil group was 3.8%; 3.4% for the extra nuts group; and 4.4% for the controls. This represents a 30% reduction in risk for combined stroke, MI, and death due to cardiovascular causes for the Mediterranean diet groups (hazard ratio [HR], 0.7 and number needed to treat [NNT], 148 for the olive oil group; HR, 0.7 and NNT, 100 for the group consuming extra nuts). Similar benefits were found in the multivariable adjusted analyses. The results correspond to three fewer events (stroke, MI, or ­cardiovascular death) per 1,000 person-years for this high-risk population.

The only individual outcome that showed a significant decrease was stroke, with an NNT of 125 in both Mediterranean diet groups. Outcomes for the controls were similar before and after they began receiving quarterly counseling.

WHAT’S NEW?

Mediterranean diet is better than a lower-fat regimen

This study indicates that a Mediterranean diet, with increased intake of either olive oil or mixed nuts, is more protective against CVD than a recommended low-fat diet. It also shows that advising patients at high risk to follow a Mediterranean diet, providing dietary counseling, and monitoring them for adherence, rather than simply recommending a low-fat diet, can significantly decrease the risk for stroke.

Rates of CVD are higher in the US than in Spain, so implementing a Mediterranean diet on a large scale in this country has the potential to produce a greater response than that seen in this study.

CAVEATS

Would a true low-fat diet be a better comparison?

Although the control group’s diet was meant to be low fat, the participants did not achieve this, possibly due to the relatively low level of dietary education and personalized counseling at the start of the study. Their inability to reach the < 30% fat target could also reflect the difficulty patients have, in general, in decreasing fat content in their diet, which may mean the diet they maintained was a more realistic comparison.

This study used one brand of olive oil and a particular mixture of nuts (walnuts, hazelnuts, and almonds); it is possible that variations on either of these could affect the benefits of the diet.

CHALLENGES TO IMPLEMENTATION

Fitting a Mediterranean diet into an American lifestyle

The typical US diet is significantly different from that of most Spaniards. Americans may find it difficult to add either ≥ 4 Tb of olive oil or 30 g (1/3 cup) of nuts daily, for example, due to both cost and availability. Limited access to both individual and group counseling could be a barrier, as well.

On the other hand, this practice changer has the potential to simplify dietary counseling by allowing clinicians to focus on just one type of diet, for which there are many resources available both online and in print. We believe it makes sense to recommend a Mediterranean diet, while continuing to recommend increased exercise, smoking cessation, and improved control of chronic disease to lower patients’ risk for poor outcomes from CVD.

REFERENCES

1. Estruch R, Ros F, Salas-Salvado J, et al. Primary prevention of cardiovascular disease with a Mediterranean diet. N Engl J Med. 2013;368: 1279-1290.

2. Go AS, Mozaffarian D, Roger VL, et al; American Heart Association Statistics Committee and Stroke Statistics Subcommittee. Heart disease and stroke statistics—2013 update: a report from the American Heart Association. Circulation. 2013;127:e6-e245.

3. Kushner RF. Barriers to providing nutrition counseling by physicians: a survey of primary care practitioners. Prev Med. 1995;24:546-552.

4. USPSTF. Behavioral counseling to promote a healthful diet and physical activity for cardiovascular disease prevention in adults. www.uspreventiveservicestaskforce.org/uspstf/usp sphys.htm. Accessed December 18, 2013.

5. Hu FB, Stampfer MJ, Manson JE, et al. Dietary fat intake and the risk of coronary heart disease in women. N Engl J Med. 1997;337:1491-1499.

6. Oomen C, Ocké MC, Feskens JM, et al. Association between trans fatty acid intake and 10-year risk of coronary heart disease in the Zutphen Elderly Study: a prospective population-based study. Lancet. 2001;357:746-751.

7. de Lorgeril M, Salen P, Martin JL, et al. Mediterranean diet, traditional risk factors and the rate of cardiovascular complications after myocardial infarction. Final report of the Lyon Diet Heart Study. Circulation. 1999;99:779-785.

8. Knoops KT, de Groot LC, Kromhout D, et al. Mediterranean diet, lifestyle factors, and 10-year mortality in elderly European men and women: the HALE project. JAMA. 2004;292: 1433-1439.

 

 

9. Kris-Etherton P, Eckel RH, Howard BV, et al; Nutrition Committee Population Science Committee and Clinical Science Committee of the American Heart Association. Lyon Diet Heart Study. Benefits of a Mediterranean-style, National Education Program/AHA Step 1 Dietary Pattern on cardiovascular disease. Circulation. 2001;103:1823-1825.

10. Panagiotakos DB, Chrysohoou C, Pitsavos C, et al. The association of Mediterranean diet with lower risk of acute coronary syndromes, in hypertensive subjects. Int J Cardiol. 2002; 82:141-147.

11. Panagiotakos DB, Pitsavos C, Chrysohoou C, et al. The role of traditional Mediterranean-type of diet and lifestyle, in the development of acute coronary syndromes: preliminary results from CARDIO 2000 study. Centr Eur J Public Health. 2002;10:11-15.

12. Esposito K, Marfella R, Ciotola M, et al. Effect of a Mediterranean-style diet on endothelial dysfunction and markers of vascular inflammation in the metabolic syndrome: a randomized trial. JAMA. 2004;292:1440-1446.

13. Vincent-Baudry S, Defoort C, Gerber M, et al. The Medi-RIVAGE study: reduction of cardiovascular disease risk factors after a 3-mo intervention with a Mediterranean-type diet or a low-fat diet. Am J Clin Nutr. 2005;82: 964-971.

Acknowledgement

The PURLs Surveillance System is supported in part by Grant Number UL 1RR 024999 from the National Center for Research Resources, a Clinical Translational Science Award to the University of Chicago. The content is solely the responsibility of the authors and does not necessarily represent the official views of the National Center for Research Resources or the National Institutes of Health.

Copyright © 2013. The Family Physicians Inquiries Network. All rights reserved.

Reprinted with permission from the Family Physicians Inquiries Network and The Journal of Family Practice. 2013;62(12):745-746, 748.

Issue
Clinician Reviews - 24(1)
Issue
Clinician Reviews - 24(1)
Page Number
18-20
Page Number
18-20
Publications
Publications
Topics
Article Type
Display Headline
Mediterranean Diet: Higher Fat But Lower Risk
Display Headline
Mediterranean Diet: Higher Fat But Lower Risk
Legacy Keywords
PURLs, practice changer, CVD, cardiovascular disease, Mediterranean diet, best diet, low-fat, fat, diet
Legacy Keywords
PURLs, practice changer, CVD, cardiovascular disease, Mediterranean diet, best diet, low-fat, fat, diet
Sections
Article Source

PURLs Copyright

Inside the Article

Is Self-swabbing for STIs a Good Idea?

Article Type
Changed
Thu, 03/28/2019 - 15:55
Display Headline
Is Self-swabbing for STIs a Good Idea?
It is. There is no down side to self-collection, this study suggests.

PRACTICE CHANGER

Ask women who are at risk for sexually transmitted infections (STIs) to self-swab for chlamydia and gonorrhea testing; self-collection of vulvovaginal swabs with nucleic acid amplification testing (NAAT) has excellent sensitivity in women with and without symptoms.1,2

STRENGTH OF RECOMMENDATION

B: Based on a prospective diagnostic cohort study.1,2

ILLUSTRATIVE CASE

An 18-year-old patient requests testing for STIs. She has no symptoms. What is the best way to collect samples for chlamydia and gonorrhea testing?

Despite public health efforts, chlamydia and gonorrhea remain significant health problems, with more than 1.4 million cases of chlamydia and 321,849 cases of gonorrhea reported in the United States in 2011.3 Both can have devastating effects on reproduction, even in women who are ­asymptomatic.

Annual testing recommended for at-risk women

According to the CDC, most reported cases of chlamydia (70%) and gonorrhea (62%) occur in men and women between the ages of 15 and 24.3 Both the CDC and the US Preventive Services Task Force recommend annual chlamydia screening for all sexually active women younger than 25 and for older women who have certain risk factors (eg, having multiple sex partners or living in communities with a high burden of disease).4,5 Annual gonorrhea screening is also recommended for sexually active women with risk factors.4,5

How best to test? A number of unknowns

NAAT is the most sensitive test for detection of chlamydia and gonorrhea, but other questions about how best to screen for STIs remain.1,6 It has not been clear whether self-collected vulvovaginal swabs are equivalent to clinician-collected urethral or endocervical swabs for the detection of gonorrhea, or whether NAAT testing of the self-collected swabs or culture of the clinician-collected swabs is a more sensitive test for gonorrhea.

While some studies have found self-collected vulvovaginal samples to be as sensitive as clinician-collected endocervical samples for the diagnosis of chlamydia and gonorrhea, samples are still often collected by clinicians.7,8 Collecting endocervical swabs is uncomfortable for patients and time consuming for clinicians, and evidence suggests that patients prefer noninvasive sampling.9

STUDY SUMMARY
Self-collected samples are highly sensitive

This study was designed to compare the sensitivity and specificity of self-collected vulvovaginal swabs with that of clinician-collected swabs for chlamydia and gonorrhea, both in asymptomatic women and women with symptoms of an STI. Test methods were also assessed for gonorrhea, comparing detection rates of self-swabs tested with NAAT and the culture of clinician-collected urethral and endocervical samples.

The researchers evaluated a total of 3,973 women, ages 16 to 59, who sought care at a single sexual health center in the United Kingdom. The average age was 25; 37% of the participants reported a prior STI, and 42% had at least one symptom suggestive of an STI. Women were excluded from the study if they had taken an antibiotic in the preceding 28 days or were unable/unwilling to take a vulvovaginal swab or undergo clinician examination and sample collection.

The women performed vulvovaginal swabs for NAAT prior to a speculum exam; endocervical swab for both NAAT and culture and a urethral swab for culture were collected by the clinician. All the swabs sent for NAAT were tested for chlamydia and gonorrhea, and cultures were performed to detect gonorrhea.

Chlamydia: Vulvovaginal swabs have higher detection rates

Of the 3,867 participants with complete results, 10.2% were infected with chlamydia. Self-collected vulvovaginal swabs were significantly more sensitive than endocervical swabs (97% vs 88%) and had equal specificity (99.9% vs 100%). In women with symptoms of an STI, the sensitivity was 97% vs 88%; in those with no symptoms, the sensitivity was 97% vs 89%.

Gonorrhea: Self-collection, NAAT yield better results

Gonorrhea was found in 2.5% of the 3,859 women with complete results for testing of this STI. Self-collected swabs and clinician-collected swabs analyzed by NAAT both had excellent sensitivity (99% and 96%, respectively). But self-collected samples that underwent NAAT were significantly more sensitive than clinician-collected urethral and endocervical samples that were cultured (99% vs 81%). The number needed to test by self-collection for NAAT (compared with clinician-collected culture) to detect one additional case of gonorrhea was 5.

In women with symptoms suggestive of infection, the NAAT assays—both clinician- and self-collected—were equivalent and were more sensitive than gonorrhea culture. In asymptomatic women, 1.8% of whom had gonorrhea, the vulvovaginal swab sent for NAAT was more sensitive than culture (98% vs 78%) and was equivalent to the endocervical swab for NAAT (90%).

The bottom line: Self-collected vulvovaginal swabs are the sample of choice for both chlamydia and gonorrhea testing in women, regardless of whether they have symptoms. When a clinical examination is needed, either the clinician or the patient can collect a vulvovaginal swab.

 

 

WHAT’S NEW
Endocervical samples, ­cultures have lower ­detection rates

In this study, endocervical samples collected by the clinician rather than self-collected vulvovaginal samples would have missed 9% (one in 11) of chlamydial infections in women with symptoms of an STI. Vulvovaginal swabs and endocervical swabs have equal sensitivity for the diagnosis of gonorrhea when NAAT is used, but culture would have missed one in five gonorrhea infections (in women with and without symptoms).

CAVEATS
NAAT is costly and does not test for drug sensitivity

Although NAAT has replaced cell culture methodology as the gold standard for gonorrhea and chlamydia diagnosis, it is potentially costly if not readily available in your practice setting. What’s more, NAAT does not allow testing for antibiotic sensitivity, which is particularly relevant with increasing resistance of gonorrhea to multiple antibiotics. In addition, it’s unclear whether these results would apply to all NAAT assays or just the one used in this study.

These studies examine sensitivity and specificity of gonorrhea and chlamydia testing in a high-risk population: women who were seeking care in a sexual health center. Your patient population may be at lower risk, which will lower the prevalence of STIs and lower the positive predictive value of NAAT. A positive NAAT test for an STI should be followed by a confirmation NAAT in low-risk populations.

CHALLENGES TO IMPLEMENTATION
Reconsidering the way we practice

Most family practice providers are accustomed to performing a full examination on patients with a suspected STI, and changing the flow of the office visit may be difficult. And to implement this practice changer properly, it would be necessary to provide patient instruction in self-collection technique.

Also, making this change could be costly if you do not have this particular NAAT available. Once implemented, however, self-collection with NAAT will likely save time and be more comfortable for your patients. It will also provide a higher sensitivity in detecting chlamydia infections and equal sensitivity in detecting gonorrhea, compared with clinician-collected NAAT testing.

REFERENCES

1. Schoeman SA, Stewart CM, Booth RA, et al. Assessment of best single sample for finding chlamydia in women with and without symptoms: a diagnostic test study. BMJ. 2012;345:e8013.

2. Stewart CM, Schoeman SA, Booth RA, et al. Assessment of self taken swabs versus clinician taken swab for cultures for diagnosing gonorrhea in women: single centre, diagnostic accuracy study. BMJ. 2012;345: e8107.

3. CDC. Fact sheet: STD trends in the United States—2011 national data for chlamydia, gonorrhea, and syphilis (2012). www.cdc.gov/std/stats11/trends-2011.pdf. Accessed November 15, 2013.

4. CDC. Fact sheet: Incidence, prevalence, and cost of sexually transmitted infections in the United States (2013). www.cdc.gov/std/stats/STI-Estimates-Fact-Sheet-Feb-2013.pdf. Accessed November 15, 2013.

5. United States Preventive Services Task Force.  USPSTF Recommendations for STI screening (2008). www.uspreventiveservicestaskforce.org/uspstf08/methods/stinfections.htm. Accessed November 15, 2013.

6. CDC. 2010 STD treatment guidelines. www.cdc.gov/std/treatment/2010/default.htm. Accessed November 15, 2013.

7. Cook RL, Hutchison SL, Østergaard L, et al. Systematic review: non-invasive testing for Chlamydia trachomatis and Neisseria gonorrhoeae. Ann Intern Med. 2005;142:914-925.

8. Moss S, Mallinson H. The contribution of APTIMA Combo 2 assay to the diagnosis of gonorrhoea in genitourinary medicine setting. Int J STD AIDS. 2007;18:551-554.

9. Chernesky MA, Hook EW 3rd, Martin DH, et al. Women find it easy and prefer to collect their own vaginal swabs to diagnose ­Chlamydia trachomatis or Neisseria gonorrhaea infections. Sex Transm Dis. 2005;32: 729-733.

ACKNOWLEDGEMENT

The PURLs Surveillance System is supported in part by Grant Number UL 1RR 024999 from the National Center for Research Resources, a Clinical Translational Science Award to the University of Chicago. The content is solely the responsibility of the authors and does not necessarily represent the official views of the National Center for Research Resources or the National Institutes of Health.

Copyright © 2013. The Family Physicians Inquiries Network. All rights reserved.

Reprinted with permission from the Family Physicians Inquiries Network and The Journal of Family Practice. 2013;62(11):651-653.

Author and Disclosure Information

Cristy Page, MD, MPH, Anne Mounsey, MD, Kate Rowland, MD, MS

Issue
Clinician Reviews - 23(12)
Publications
Topics
Page Number
28,32-33
Legacy Keywords
purls, sti, self swab, swabbing, sexually transmitted, chlamydia, gonorrhea, vulvovaginal, NAAT, urethral, endocervical
Sections
Author and Disclosure Information

Cristy Page, MD, MPH, Anne Mounsey, MD, Kate Rowland, MD, MS

Author and Disclosure Information

Cristy Page, MD, MPH, Anne Mounsey, MD, Kate Rowland, MD, MS

It is. There is no down side to self-collection, this study suggests.
It is. There is no down side to self-collection, this study suggests.

PRACTICE CHANGER

Ask women who are at risk for sexually transmitted infections (STIs) to self-swab for chlamydia and gonorrhea testing; self-collection of vulvovaginal swabs with nucleic acid amplification testing (NAAT) has excellent sensitivity in women with and without symptoms.1,2

STRENGTH OF RECOMMENDATION

B: Based on a prospective diagnostic cohort study.1,2

ILLUSTRATIVE CASE

An 18-year-old patient requests testing for STIs. She has no symptoms. What is the best way to collect samples for chlamydia and gonorrhea testing?

Despite public health efforts, chlamydia and gonorrhea remain significant health problems, with more than 1.4 million cases of chlamydia and 321,849 cases of gonorrhea reported in the United States in 2011.3 Both can have devastating effects on reproduction, even in women who are ­asymptomatic.

Annual testing recommended for at-risk women

According to the CDC, most reported cases of chlamydia (70%) and gonorrhea (62%) occur in men and women between the ages of 15 and 24.3 Both the CDC and the US Preventive Services Task Force recommend annual chlamydia screening for all sexually active women younger than 25 and for older women who have certain risk factors (eg, having multiple sex partners or living in communities with a high burden of disease).4,5 Annual gonorrhea screening is also recommended for sexually active women with risk factors.4,5

How best to test? A number of unknowns

NAAT is the most sensitive test for detection of chlamydia and gonorrhea, but other questions about how best to screen for STIs remain.1,6 It has not been clear whether self-collected vulvovaginal swabs are equivalent to clinician-collected urethral or endocervical swabs for the detection of gonorrhea, or whether NAAT testing of the self-collected swabs or culture of the clinician-collected swabs is a more sensitive test for gonorrhea.

While some studies have found self-collected vulvovaginal samples to be as sensitive as clinician-collected endocervical samples for the diagnosis of chlamydia and gonorrhea, samples are still often collected by clinicians.7,8 Collecting endocervical swabs is uncomfortable for patients and time consuming for clinicians, and evidence suggests that patients prefer noninvasive sampling.9

STUDY SUMMARY
Self-collected samples are highly sensitive

This study was designed to compare the sensitivity and specificity of self-collected vulvovaginal swabs with that of clinician-collected swabs for chlamydia and gonorrhea, both in asymptomatic women and women with symptoms of an STI. Test methods were also assessed for gonorrhea, comparing detection rates of self-swabs tested with NAAT and the culture of clinician-collected urethral and endocervical samples.

The researchers evaluated a total of 3,973 women, ages 16 to 59, who sought care at a single sexual health center in the United Kingdom. The average age was 25; 37% of the participants reported a prior STI, and 42% had at least one symptom suggestive of an STI. Women were excluded from the study if they had taken an antibiotic in the preceding 28 days or were unable/unwilling to take a vulvovaginal swab or undergo clinician examination and sample collection.

The women performed vulvovaginal swabs for NAAT prior to a speculum exam; endocervical swab for both NAAT and culture and a urethral swab for culture were collected by the clinician. All the swabs sent for NAAT were tested for chlamydia and gonorrhea, and cultures were performed to detect gonorrhea.

Chlamydia: Vulvovaginal swabs have higher detection rates

Of the 3,867 participants with complete results, 10.2% were infected with chlamydia. Self-collected vulvovaginal swabs were significantly more sensitive than endocervical swabs (97% vs 88%) and had equal specificity (99.9% vs 100%). In women with symptoms of an STI, the sensitivity was 97% vs 88%; in those with no symptoms, the sensitivity was 97% vs 89%.

Gonorrhea: Self-collection, NAAT yield better results

Gonorrhea was found in 2.5% of the 3,859 women with complete results for testing of this STI. Self-collected swabs and clinician-collected swabs analyzed by NAAT both had excellent sensitivity (99% and 96%, respectively). But self-collected samples that underwent NAAT were significantly more sensitive than clinician-collected urethral and endocervical samples that were cultured (99% vs 81%). The number needed to test by self-collection for NAAT (compared with clinician-collected culture) to detect one additional case of gonorrhea was 5.

In women with symptoms suggestive of infection, the NAAT assays—both clinician- and self-collected—were equivalent and were more sensitive than gonorrhea culture. In asymptomatic women, 1.8% of whom had gonorrhea, the vulvovaginal swab sent for NAAT was more sensitive than culture (98% vs 78%) and was equivalent to the endocervical swab for NAAT (90%).

The bottom line: Self-collected vulvovaginal swabs are the sample of choice for both chlamydia and gonorrhea testing in women, regardless of whether they have symptoms. When a clinical examination is needed, either the clinician or the patient can collect a vulvovaginal swab.

 

 

WHAT’S NEW
Endocervical samples, ­cultures have lower ­detection rates

In this study, endocervical samples collected by the clinician rather than self-collected vulvovaginal samples would have missed 9% (one in 11) of chlamydial infections in women with symptoms of an STI. Vulvovaginal swabs and endocervical swabs have equal sensitivity for the diagnosis of gonorrhea when NAAT is used, but culture would have missed one in five gonorrhea infections (in women with and without symptoms).

CAVEATS
NAAT is costly and does not test for drug sensitivity

Although NAAT has replaced cell culture methodology as the gold standard for gonorrhea and chlamydia diagnosis, it is potentially costly if not readily available in your practice setting. What’s more, NAAT does not allow testing for antibiotic sensitivity, which is particularly relevant with increasing resistance of gonorrhea to multiple antibiotics. In addition, it’s unclear whether these results would apply to all NAAT assays or just the one used in this study.

These studies examine sensitivity and specificity of gonorrhea and chlamydia testing in a high-risk population: women who were seeking care in a sexual health center. Your patient population may be at lower risk, which will lower the prevalence of STIs and lower the positive predictive value of NAAT. A positive NAAT test for an STI should be followed by a confirmation NAAT in low-risk populations.

CHALLENGES TO IMPLEMENTATION
Reconsidering the way we practice

Most family practice providers are accustomed to performing a full examination on patients with a suspected STI, and changing the flow of the office visit may be difficult. And to implement this practice changer properly, it would be necessary to provide patient instruction in self-collection technique.

Also, making this change could be costly if you do not have this particular NAAT available. Once implemented, however, self-collection with NAAT will likely save time and be more comfortable for your patients. It will also provide a higher sensitivity in detecting chlamydia infections and equal sensitivity in detecting gonorrhea, compared with clinician-collected NAAT testing.

REFERENCES

1. Schoeman SA, Stewart CM, Booth RA, et al. Assessment of best single sample for finding chlamydia in women with and without symptoms: a diagnostic test study. BMJ. 2012;345:e8013.

2. Stewart CM, Schoeman SA, Booth RA, et al. Assessment of self taken swabs versus clinician taken swab for cultures for diagnosing gonorrhea in women: single centre, diagnostic accuracy study. BMJ. 2012;345: e8107.

3. CDC. Fact sheet: STD trends in the United States—2011 national data for chlamydia, gonorrhea, and syphilis (2012). www.cdc.gov/std/stats11/trends-2011.pdf. Accessed November 15, 2013.

4. CDC. Fact sheet: Incidence, prevalence, and cost of sexually transmitted infections in the United States (2013). www.cdc.gov/std/stats/STI-Estimates-Fact-Sheet-Feb-2013.pdf. Accessed November 15, 2013.

5. United States Preventive Services Task Force.  USPSTF Recommendations for STI screening (2008). www.uspreventiveservicestaskforce.org/uspstf08/methods/stinfections.htm. Accessed November 15, 2013.

6. CDC. 2010 STD treatment guidelines. www.cdc.gov/std/treatment/2010/default.htm. Accessed November 15, 2013.

7. Cook RL, Hutchison SL, Østergaard L, et al. Systematic review: non-invasive testing for Chlamydia trachomatis and Neisseria gonorrhoeae. Ann Intern Med. 2005;142:914-925.

8. Moss S, Mallinson H. The contribution of APTIMA Combo 2 assay to the diagnosis of gonorrhoea in genitourinary medicine setting. Int J STD AIDS. 2007;18:551-554.

9. Chernesky MA, Hook EW 3rd, Martin DH, et al. Women find it easy and prefer to collect their own vaginal swabs to diagnose ­Chlamydia trachomatis or Neisseria gonorrhaea infections. Sex Transm Dis. 2005;32: 729-733.

ACKNOWLEDGEMENT

The PURLs Surveillance System is supported in part by Grant Number UL 1RR 024999 from the National Center for Research Resources, a Clinical Translational Science Award to the University of Chicago. The content is solely the responsibility of the authors and does not necessarily represent the official views of the National Center for Research Resources or the National Institutes of Health.

Copyright © 2013. The Family Physicians Inquiries Network. All rights reserved.

Reprinted with permission from the Family Physicians Inquiries Network and The Journal of Family Practice. 2013;62(11):651-653.

PRACTICE CHANGER

Ask women who are at risk for sexually transmitted infections (STIs) to self-swab for chlamydia and gonorrhea testing; self-collection of vulvovaginal swabs with nucleic acid amplification testing (NAAT) has excellent sensitivity in women with and without symptoms.1,2

STRENGTH OF RECOMMENDATION

B: Based on a prospective diagnostic cohort study.1,2

ILLUSTRATIVE CASE

An 18-year-old patient requests testing for STIs. She has no symptoms. What is the best way to collect samples for chlamydia and gonorrhea testing?

Despite public health efforts, chlamydia and gonorrhea remain significant health problems, with more than 1.4 million cases of chlamydia and 321,849 cases of gonorrhea reported in the United States in 2011.3 Both can have devastating effects on reproduction, even in women who are ­asymptomatic.

Annual testing recommended for at-risk women

According to the CDC, most reported cases of chlamydia (70%) and gonorrhea (62%) occur in men and women between the ages of 15 and 24.3 Both the CDC and the US Preventive Services Task Force recommend annual chlamydia screening for all sexually active women younger than 25 and for older women who have certain risk factors (eg, having multiple sex partners or living in communities with a high burden of disease).4,5 Annual gonorrhea screening is also recommended for sexually active women with risk factors.4,5

How best to test? A number of unknowns

NAAT is the most sensitive test for detection of chlamydia and gonorrhea, but other questions about how best to screen for STIs remain.1,6 It has not been clear whether self-collected vulvovaginal swabs are equivalent to clinician-collected urethral or endocervical swabs for the detection of gonorrhea, or whether NAAT testing of the self-collected swabs or culture of the clinician-collected swabs is a more sensitive test for gonorrhea.

While some studies have found self-collected vulvovaginal samples to be as sensitive as clinician-collected endocervical samples for the diagnosis of chlamydia and gonorrhea, samples are still often collected by clinicians.7,8 Collecting endocervical swabs is uncomfortable for patients and time consuming for clinicians, and evidence suggests that patients prefer noninvasive sampling.9

STUDY SUMMARY
Self-collected samples are highly sensitive

This study was designed to compare the sensitivity and specificity of self-collected vulvovaginal swabs with that of clinician-collected swabs for chlamydia and gonorrhea, both in asymptomatic women and women with symptoms of an STI. Test methods were also assessed for gonorrhea, comparing detection rates of self-swabs tested with NAAT and the culture of clinician-collected urethral and endocervical samples.

The researchers evaluated a total of 3,973 women, ages 16 to 59, who sought care at a single sexual health center in the United Kingdom. The average age was 25; 37% of the participants reported a prior STI, and 42% had at least one symptom suggestive of an STI. Women were excluded from the study if they had taken an antibiotic in the preceding 28 days or were unable/unwilling to take a vulvovaginal swab or undergo clinician examination and sample collection.

The women performed vulvovaginal swabs for NAAT prior to a speculum exam; endocervical swab for both NAAT and culture and a urethral swab for culture were collected by the clinician. All the swabs sent for NAAT were tested for chlamydia and gonorrhea, and cultures were performed to detect gonorrhea.

Chlamydia: Vulvovaginal swabs have higher detection rates

Of the 3,867 participants with complete results, 10.2% were infected with chlamydia. Self-collected vulvovaginal swabs were significantly more sensitive than endocervical swabs (97% vs 88%) and had equal specificity (99.9% vs 100%). In women with symptoms of an STI, the sensitivity was 97% vs 88%; in those with no symptoms, the sensitivity was 97% vs 89%.

Gonorrhea: Self-collection, NAAT yield better results

Gonorrhea was found in 2.5% of the 3,859 women with complete results for testing of this STI. Self-collected swabs and clinician-collected swabs analyzed by NAAT both had excellent sensitivity (99% and 96%, respectively). But self-collected samples that underwent NAAT were significantly more sensitive than clinician-collected urethral and endocervical samples that were cultured (99% vs 81%). The number needed to test by self-collection for NAAT (compared with clinician-collected culture) to detect one additional case of gonorrhea was 5.

In women with symptoms suggestive of infection, the NAAT assays—both clinician- and self-collected—were equivalent and were more sensitive than gonorrhea culture. In asymptomatic women, 1.8% of whom had gonorrhea, the vulvovaginal swab sent for NAAT was more sensitive than culture (98% vs 78%) and was equivalent to the endocervical swab for NAAT (90%).

The bottom line: Self-collected vulvovaginal swabs are the sample of choice for both chlamydia and gonorrhea testing in women, regardless of whether they have symptoms. When a clinical examination is needed, either the clinician or the patient can collect a vulvovaginal swab.

 

 

WHAT’S NEW
Endocervical samples, ­cultures have lower ­detection rates

In this study, endocervical samples collected by the clinician rather than self-collected vulvovaginal samples would have missed 9% (one in 11) of chlamydial infections in women with symptoms of an STI. Vulvovaginal swabs and endocervical swabs have equal sensitivity for the diagnosis of gonorrhea when NAAT is used, but culture would have missed one in five gonorrhea infections (in women with and without symptoms).

CAVEATS
NAAT is costly and does not test for drug sensitivity

Although NAAT has replaced cell culture methodology as the gold standard for gonorrhea and chlamydia diagnosis, it is potentially costly if not readily available in your practice setting. What’s more, NAAT does not allow testing for antibiotic sensitivity, which is particularly relevant with increasing resistance of gonorrhea to multiple antibiotics. In addition, it’s unclear whether these results would apply to all NAAT assays or just the one used in this study.

These studies examine sensitivity and specificity of gonorrhea and chlamydia testing in a high-risk population: women who were seeking care in a sexual health center. Your patient population may be at lower risk, which will lower the prevalence of STIs and lower the positive predictive value of NAAT. A positive NAAT test for an STI should be followed by a confirmation NAAT in low-risk populations.

CHALLENGES TO IMPLEMENTATION
Reconsidering the way we practice

Most family practice providers are accustomed to performing a full examination on patients with a suspected STI, and changing the flow of the office visit may be difficult. And to implement this practice changer properly, it would be necessary to provide patient instruction in self-collection technique.

Also, making this change could be costly if you do not have this particular NAAT available. Once implemented, however, self-collection with NAAT will likely save time and be more comfortable for your patients. It will also provide a higher sensitivity in detecting chlamydia infections and equal sensitivity in detecting gonorrhea, compared with clinician-collected NAAT testing.

REFERENCES

1. Schoeman SA, Stewart CM, Booth RA, et al. Assessment of best single sample for finding chlamydia in women with and without symptoms: a diagnostic test study. BMJ. 2012;345:e8013.

2. Stewart CM, Schoeman SA, Booth RA, et al. Assessment of self taken swabs versus clinician taken swab for cultures for diagnosing gonorrhea in women: single centre, diagnostic accuracy study. BMJ. 2012;345: e8107.

3. CDC. Fact sheet: STD trends in the United States—2011 national data for chlamydia, gonorrhea, and syphilis (2012). www.cdc.gov/std/stats11/trends-2011.pdf. Accessed November 15, 2013.

4. CDC. Fact sheet: Incidence, prevalence, and cost of sexually transmitted infections in the United States (2013). www.cdc.gov/std/stats/STI-Estimates-Fact-Sheet-Feb-2013.pdf. Accessed November 15, 2013.

5. United States Preventive Services Task Force.  USPSTF Recommendations for STI screening (2008). www.uspreventiveservicestaskforce.org/uspstf08/methods/stinfections.htm. Accessed November 15, 2013.

6. CDC. 2010 STD treatment guidelines. www.cdc.gov/std/treatment/2010/default.htm. Accessed November 15, 2013.

7. Cook RL, Hutchison SL, Østergaard L, et al. Systematic review: non-invasive testing for Chlamydia trachomatis and Neisseria gonorrhoeae. Ann Intern Med. 2005;142:914-925.

8. Moss S, Mallinson H. The contribution of APTIMA Combo 2 assay to the diagnosis of gonorrhoea in genitourinary medicine setting. Int J STD AIDS. 2007;18:551-554.

9. Chernesky MA, Hook EW 3rd, Martin DH, et al. Women find it easy and prefer to collect their own vaginal swabs to diagnose ­Chlamydia trachomatis or Neisseria gonorrhaea infections. Sex Transm Dis. 2005;32: 729-733.

ACKNOWLEDGEMENT

The PURLs Surveillance System is supported in part by Grant Number UL 1RR 024999 from the National Center for Research Resources, a Clinical Translational Science Award to the University of Chicago. The content is solely the responsibility of the authors and does not necessarily represent the official views of the National Center for Research Resources or the National Institutes of Health.

Copyright © 2013. The Family Physicians Inquiries Network. All rights reserved.

Reprinted with permission from the Family Physicians Inquiries Network and The Journal of Family Practice. 2013;62(11):651-653.

Issue
Clinician Reviews - 23(12)
Issue
Clinician Reviews - 23(12)
Page Number
28,32-33
Page Number
28,32-33
Publications
Publications
Topics
Article Type
Display Headline
Is Self-swabbing for STIs a Good Idea?
Display Headline
Is Self-swabbing for STIs a Good Idea?
Legacy Keywords
purls, sti, self swab, swabbing, sexually transmitted, chlamydia, gonorrhea, vulvovaginal, NAAT, urethral, endocervical
Legacy Keywords
purls, sti, self swab, swabbing, sexually transmitted, chlamydia, gonorrhea, vulvovaginal, NAAT, urethral, endocervical
Sections
Article Source

PURLs Copyright

Inside the Article

Mediterranean diet: Higher fat but lower risk

Article Type
Changed
Tue, 07/14/2020 - 15:25
Display Headline
Mediterranean diet: Higher fat but lower risk
Practice changer

Counsel patients at high risk for cardiovascular disease and stroke to follow a Mediterranean diet, which is associated with a 30% risk reduction.1

Strength of recommendation

A: Based on one well-design randomized controlled trial (RCT).

Estruch R, Ros F, Salas-Salvado J, et al. Primary prevention of cardiovascular disease with a Mediterranean diet. N Engl J Med. 2013;368:1279-1290.

 

Illustrative case

A 62-year-old patient with diabetes, obesity, and a family history of early onset coronary artery disease is motivated to make significant lifestyle changes. You recommend moderate aerobic exercise for 30 minutes 5 times a week, but wonder whether a low-fat diet or a Mediterranean diet would be more effective in lowering her risk.

Cardiovascular disease (CVD), including heart disease and stroke, is the leading cause of mortality in the United States. CVD accounts for one in every 3 deaths,2 and stroke is a leading cause of long-term disability.2 The direct cost of treating CVD is estimated at $312.6 billion annually.2

Many modifiable risk factors contribute to CVD, including smoking, sedentary lifestyle, obesity, alcohol consumption, and poorly controlled chronic disease, as well as an unhealthy diet. A recent report from the American Heart Association suggests that 13% of deaths from CVD can be attributed to poor diet.2

Focus counseling on patients at risk
Primary care providers (PCPs) often struggle to effectively counsel patients on behavior change strategies, but face many barriers. Chief among them are the lack of time, training, and confidence in their counseling techniques, as well as a lack of patient motivation and readiness to change.3 In recognition of these barriers, the US Preventive Services Task Force recently recommended that PCPs focus behavioral counseling efforts on patients at high risk for heart disease.4

Large observational studies have found an association between trans fat and an increased risk of CVD, as well as a decreased risk of CVD in patients adhering to a Mediterranean diet.5-11 This type of diet typically includes a high intake of olive oil, fruit, nuts, vegetables, and cereals; moderate intake of fish and poultry; and low intake of dairy products, red meat, processed meats, and sweets. It also includes wine in moderation, consumed with meals.

Data on the physiologic properties of olive oil, including its antioxidant, vasodilating, and antiplatelet effects—as well as its effects on low-density lipoprotein cholesterol (LDL-C) that may inhibit atherogenesis—support the link between a Mediterranean diet and a decreased risk of CVD found in the observational studies.12,13 Until recently, however, no RCT had compared the effect of a Mediterranean diet with that of a low-fat diet for primary prevention of CVD.

STUDY SUMMARY: Mediterranean diet significantly lowers risk

Prevencion con Dieta Mediterranea (PREDIMED) was a large RCT (N=7447) comparing 2 variations of a Mediterranean diet with a low-fat diet for primary prevention of CVD. This Spanish study enrolled men 55 to 80 years of age and women ages 60 to 80 at high risk for developing CVD. The risk was based on either a diagnosis of type 2 diabetes or the presence of ≥3 major risk factors, including smoking, hypertension, elevated LDL-C, low high-density lipoprotein cholesterol, overweight or obese, and a family history of early heart disease.

A Mediterranean diet, with increased intake of either olive oil or mixed nuts, is more protective against CVD than a recommended low-fat diet.Participants were randomly assigned to one of 3 dietary groups: One group was assigned to a Mediterranean diet supplemented with ≥4 tablespoons per day of extra virgin olive oil; a second group was put on a Mediterranean diet supplemented by 30 grams (about 1/3 cup) of mixed nuts daily; a third group (the controls) was advised to follow a low-fat diet. The majority of baseline characteristics and medications taken throughout the study were similar among all 3 groups.

Those in both Mediterranean diet groups were followed for a median of 4.8 years, during which time they received quarterly dietary classes and individual and group counseling. The controls received baseline training, plus a leaflet about low-fat diets annually. In year 3, however, the researchers began giving the control group the same level of counseling as those in the Mediterranean diet groups to avoid confounding results.

Adherence to the diets was determined by a self-reported 14-item dietary screening questionnaire, plus urinary hydroxytyrosol and serum alpha-linoleic acid levels to assess for olive oil and mixed nut compliance. Self-reporting5 and biometric data indicated good compliance with the Mediterranean diets, and there was no difference found in levels of exercise among the groups.

After 5 years, those in the Mediterranean diet groups had consumed significantly more olive oil, nuts, vegetables, fruits, wine, legumes, fish, seafood, and sofrito sauce (a popular tomato-based sauce) than the control group. Participants in the low-fat diet group had decreased their fat intake by 2%, while those in the Mediterranean groups had increased fat intake (by 2.03% for the olive oil group and 2.1% for the nut group). Overall, 37% of energy intake by those in the low-fat diet group came from fat (exceeding the <30% of calories derived from fat intake that defines a low-fat diet) vs 39% fat intake for those in both Mediterranean diet groups.

 

 

 

The primary outcome was a composite of myocardial infarction (MI), stroke, and death from cardiovascular causes, and there were clinically meaningful and statistically significant differences between the Mediterranean diet groups and the controls. The primary outcome rate for the supplemental olive oil group was 3.8%; 3.4% for the extra nuts group; and 4.4% for the controls. This represents a 30% reduction in risk for combined stroke, MI, and death due to cardiovascular causes for the Mediterranean diet groups (hazard ratio [HR]=0.7; 95% confidence interval [CI], 0.53-0.91; P=.009; number needed to treat [NNT]=148 for the olive oil group and HR=0.7; 95% CI, 0.53-0.94; P=.02; NNT=100 for the group consuming extra nuts). Similar benefits were found in the multivariable adjusted analyses. The results correspond to 3 fewer events (stroke, MI, or cardiovascular death) per 1000 person-years for this high-risk population.

The only individual outcome that showed a significant decrease was stroke, with an NNT of 125 in both Mediterranean diet groups. Outcomes for the controls were similar before and after they began receiving quarterly counseling.

WHAT'S NEW?: Mediterranean diet is better than a lower-fat regimen

Rates of CVD are higher in the United States than in Spain, so implementing a Mediterranean diet on a large scale in this country has the potential to produce a greater response than that seen in this study. This study indicates that a Mediterranean diet, with increased intake of either olive oil or mixed nuts, is more protective against CVD than a recommended low-fat diet. It also shows that advising patients at high risk to follow a Mediterranean diet, providing dietary counseling, and monitoring them for adherence, rather than simply recommending a low-fat diet, can significantly decrease the risk of stroke.

Rates of CVD are higher in the United States than in Spain, so implementing a Mediterranean diet on a large scale in this country has the potential to produce a greater response than that seen in this study.

CAVEATS: Would a true low-fat diet be a better comparison?

Although the control group’s diet was meant to be low fat, the participants did not achieve this, possibly due to the relatively low level of dietary education and personalized counseling at the start of the study. Their inability to reach the <30% fat target could also reflect the difficulty patients have, in general, in decreasing fat content in their diet, which may mean the diet they maintained was a more realistic comparison.

This study used one brand of olive oil and a particular mixture of nuts (walnuts, hazelnuts, and almonds); it is possible that variations on either of these could affect the benefits of the diet.

CHALLENGES TO IMPLEMENTATION: Fitting a Mediterranean diet into an American lifestyle

The typical US diet is significantly different from that of most Spaniards. Americans may find it difficult to add either ≥4 tablespoons of olive oil or 30 g (1/3 cup) of nuts daily, for example, due to both cost and availability. Limited access to both individual and group counseling could be a barrier, as well.

On the other hand, this practice changer has the potential to simplify dietary counseling by allowing clinicians to focus on just one type of diet, for which there are many resources available both online and in print. We believe it makes sense to recommend a Mediterranean diet, while continuing to recommend increased exercise, smoking cessation, and improved control of chronic disease to lower patients’ risk of poor outcomes from CVD.

Acknowledgement

The PURLs Surveillance System was supported in part by Grant Number UL 1RR 024999 from the National Center for Research Resources, a Clinical Translational Science Award to the University of Chicago. The content is solely the responsibility of the authors and does not necessarily represent the official views of the National Center for Research Resources or the National Institutes of Health.

Files
References

1. Estruch R, Ros F, Salas-Salvado J, et al. Primary prevention of cardiovascular disease with a Mediterranean Diet. N Engl J Med. 2013;368:1279-1290.

2. Go AS, Mozaffarian D, Roger VL, et al; American Heart Association Statistics Committee and Stroke Statistics Subcommittee. Heart disease and stroke statistics—2013 update: a report from the American Heart Association. Circulation. 2013;127:e6-e245.

3. Kushner RF. Barriers to providing nutrition counseling by physicians: a survey of primary care practitioners. Prev Med. 1995;24:546-552.

4. Behavioral Counseling to Promote a Healthful Diet and Physical Activity for Cardiovascular Disease Prevention in Adults, topic page. US Preventive Services Task Force Web site. Available at: http://www.uspreventiveservicestaskforce.org/uspstf/usp- sphys.htm. Accessed August 1, 2013.

5. Hu FB, Stampfer MJ, Manson JE, et al. Dietary fat intake and the risk of coronary heart disease in women. N Engl J Med. 1997;337:1491-1499.

6. Oomen C, Ocké MC, Feskens JM, et al. Association between trans fatty acid intake and 10-year risk of coronary heart disease in the Zutphen Elderly Study: a prospective population-based study. Lancet. 2001;357:746-751.

7. de Lorgeril M, Salen P, Martin JL, et al. Mediterranean diet, traditional risk factors and the rate of cardiovascular complications after myocardial infarction. Final report of the Lyon Diet Heart Study. Circulation. 1999;99:779-785.

8. Knoops KT, de Groot LC, Kromhout D, et al. Mediterranean diet, lifestyle factors, and 10-year mortality in elderly European men and women: the HALE project. JAMA. 2004;292:1433-1439.

9. Kris-Etherton P, Eckel RH, Howard BV, et al; Nutrition Committee Population Science Committee and Clinical Science Committee of the American Heart Association. Lyon Diet Heart Study. Benefits of a Mediterranean-style, National Education Program/AHA Step 1 Dietary Pattern on cardiovascular disease. Circulation. 2001;103:1823-1825.

10. Panagiotakos DB, Chrysohoou C, Pitsavos C, et al. The association of Mediterranean diet with lower risk of acute coronary syndromes, in hypertensive subjects. Int J Cardiol. 2002;82:141-147.

11. Panagiotakos DB, Pitsavos C, Chrysohoou C, et al. The role of traditional Mediterranean-type of diet and lifestyle, in the devel- opment of acute coronary syndromes: preliminary results from CARDIO 2000 study. Centr Eur J Public Health. 2002;10:11-15.

12. Esposito K, Marfella R, Ciotola M, et al. Effect of a Mediterranean-style diet on endothelial dysfunction and markers of vascular inflammation in the metabolic syndrome: a randomized trial. JAMA. 2004;292:1440-1446.

13. Vincent-Baudry S, Defoort C, Gerber M, et al. The Medi-RIVAGE study: reduction of cardiovascular disease risk factors after a 3-mo intervention with a Mediterranean-type diet or a low-fat diet. Am J Clin Nutr. 2005;82:964-971.

Article PDF
Author and Disclosure Information

Mark Gwynne, DO
Anne Mounsey, MD

Department of Family Medicine, University of North Carolina at Chapel Hill

PURLs EDITORS
Bernard Ewigman, MD, MSPH
James Stevermer, MD, MSPH

The University of Chicago (Dr. Ewigman); University of Missouri, Columbia (Dr. Stevermer)

Issue
The Journal of Family Practice - 62(12)
Publications
Topics
Page Number
745-746, 748
Legacy Keywords
Mark Gwynne; DO; Anne Mounsey; MD; diet; nutrition; mediterranean diet; cardiovascular disease; CVD
Sections
Files
Files
Author and Disclosure Information

Mark Gwynne, DO
Anne Mounsey, MD

Department of Family Medicine, University of North Carolina at Chapel Hill

PURLs EDITORS
Bernard Ewigman, MD, MSPH
James Stevermer, MD, MSPH

The University of Chicago (Dr. Ewigman); University of Missouri, Columbia (Dr. Stevermer)

Author and Disclosure Information

Mark Gwynne, DO
Anne Mounsey, MD

Department of Family Medicine, University of North Carolina at Chapel Hill

PURLs EDITORS
Bernard Ewigman, MD, MSPH
James Stevermer, MD, MSPH

The University of Chicago (Dr. Ewigman); University of Missouri, Columbia (Dr. Stevermer)

Article PDF
Article PDF
Practice changer

Counsel patients at high risk for cardiovascular disease and stroke to follow a Mediterranean diet, which is associated with a 30% risk reduction.1

Strength of recommendation

A: Based on one well-design randomized controlled trial (RCT).

Estruch R, Ros F, Salas-Salvado J, et al. Primary prevention of cardiovascular disease with a Mediterranean diet. N Engl J Med. 2013;368:1279-1290.

 

Illustrative case

A 62-year-old patient with diabetes, obesity, and a family history of early onset coronary artery disease is motivated to make significant lifestyle changes. You recommend moderate aerobic exercise for 30 minutes 5 times a week, but wonder whether a low-fat diet or a Mediterranean diet would be more effective in lowering her risk.

Cardiovascular disease (CVD), including heart disease and stroke, is the leading cause of mortality in the United States. CVD accounts for one in every 3 deaths,2 and stroke is a leading cause of long-term disability.2 The direct cost of treating CVD is estimated at $312.6 billion annually.2

Many modifiable risk factors contribute to CVD, including smoking, sedentary lifestyle, obesity, alcohol consumption, and poorly controlled chronic disease, as well as an unhealthy diet. A recent report from the American Heart Association suggests that 13% of deaths from CVD can be attributed to poor diet.2

Focus counseling on patients at risk
Primary care providers (PCPs) often struggle to effectively counsel patients on behavior change strategies, but face many barriers. Chief among them are the lack of time, training, and confidence in their counseling techniques, as well as a lack of patient motivation and readiness to change.3 In recognition of these barriers, the US Preventive Services Task Force recently recommended that PCPs focus behavioral counseling efforts on patients at high risk for heart disease.4

Large observational studies have found an association between trans fat and an increased risk of CVD, as well as a decreased risk of CVD in patients adhering to a Mediterranean diet.5-11 This type of diet typically includes a high intake of olive oil, fruit, nuts, vegetables, and cereals; moderate intake of fish and poultry; and low intake of dairy products, red meat, processed meats, and sweets. It also includes wine in moderation, consumed with meals.

Data on the physiologic properties of olive oil, including its antioxidant, vasodilating, and antiplatelet effects—as well as its effects on low-density lipoprotein cholesterol (LDL-C) that may inhibit atherogenesis—support the link between a Mediterranean diet and a decreased risk of CVD found in the observational studies.12,13 Until recently, however, no RCT had compared the effect of a Mediterranean diet with that of a low-fat diet for primary prevention of CVD.

STUDY SUMMARY: Mediterranean diet significantly lowers risk

Prevencion con Dieta Mediterranea (PREDIMED) was a large RCT (N=7447) comparing 2 variations of a Mediterranean diet with a low-fat diet for primary prevention of CVD. This Spanish study enrolled men 55 to 80 years of age and women ages 60 to 80 at high risk for developing CVD. The risk was based on either a diagnosis of type 2 diabetes or the presence of ≥3 major risk factors, including smoking, hypertension, elevated LDL-C, low high-density lipoprotein cholesterol, overweight or obese, and a family history of early heart disease.

A Mediterranean diet, with increased intake of either olive oil or mixed nuts, is more protective against CVD than a recommended low-fat diet.Participants were randomly assigned to one of 3 dietary groups: One group was assigned to a Mediterranean diet supplemented with ≥4 tablespoons per day of extra virgin olive oil; a second group was put on a Mediterranean diet supplemented by 30 grams (about 1/3 cup) of mixed nuts daily; a third group (the controls) was advised to follow a low-fat diet. The majority of baseline characteristics and medications taken throughout the study were similar among all 3 groups.

Those in both Mediterranean diet groups were followed for a median of 4.8 years, during which time they received quarterly dietary classes and individual and group counseling. The controls received baseline training, plus a leaflet about low-fat diets annually. In year 3, however, the researchers began giving the control group the same level of counseling as those in the Mediterranean diet groups to avoid confounding results.

Adherence to the diets was determined by a self-reported 14-item dietary screening questionnaire, plus urinary hydroxytyrosol and serum alpha-linoleic acid levels to assess for olive oil and mixed nut compliance. Self-reporting5 and biometric data indicated good compliance with the Mediterranean diets, and there was no difference found in levels of exercise among the groups.

After 5 years, those in the Mediterranean diet groups had consumed significantly more olive oil, nuts, vegetables, fruits, wine, legumes, fish, seafood, and sofrito sauce (a popular tomato-based sauce) than the control group. Participants in the low-fat diet group had decreased their fat intake by 2%, while those in the Mediterranean groups had increased fat intake (by 2.03% for the olive oil group and 2.1% for the nut group). Overall, 37% of energy intake by those in the low-fat diet group came from fat (exceeding the <30% of calories derived from fat intake that defines a low-fat diet) vs 39% fat intake for those in both Mediterranean diet groups.

 

 

 

The primary outcome was a composite of myocardial infarction (MI), stroke, and death from cardiovascular causes, and there were clinically meaningful and statistically significant differences between the Mediterranean diet groups and the controls. The primary outcome rate for the supplemental olive oil group was 3.8%; 3.4% for the extra nuts group; and 4.4% for the controls. This represents a 30% reduction in risk for combined stroke, MI, and death due to cardiovascular causes for the Mediterranean diet groups (hazard ratio [HR]=0.7; 95% confidence interval [CI], 0.53-0.91; P=.009; number needed to treat [NNT]=148 for the olive oil group and HR=0.7; 95% CI, 0.53-0.94; P=.02; NNT=100 for the group consuming extra nuts). Similar benefits were found in the multivariable adjusted analyses. The results correspond to 3 fewer events (stroke, MI, or cardiovascular death) per 1000 person-years for this high-risk population.

The only individual outcome that showed a significant decrease was stroke, with an NNT of 125 in both Mediterranean diet groups. Outcomes for the controls were similar before and after they began receiving quarterly counseling.

WHAT'S NEW?: Mediterranean diet is better than a lower-fat regimen

Rates of CVD are higher in the United States than in Spain, so implementing a Mediterranean diet on a large scale in this country has the potential to produce a greater response than that seen in this study. This study indicates that a Mediterranean diet, with increased intake of either olive oil or mixed nuts, is more protective against CVD than a recommended low-fat diet. It also shows that advising patients at high risk to follow a Mediterranean diet, providing dietary counseling, and monitoring them for adherence, rather than simply recommending a low-fat diet, can significantly decrease the risk of stroke.

Rates of CVD are higher in the United States than in Spain, so implementing a Mediterranean diet on a large scale in this country has the potential to produce a greater response than that seen in this study.

CAVEATS: Would a true low-fat diet be a better comparison?

Although the control group’s diet was meant to be low fat, the participants did not achieve this, possibly due to the relatively low level of dietary education and personalized counseling at the start of the study. Their inability to reach the <30% fat target could also reflect the difficulty patients have, in general, in decreasing fat content in their diet, which may mean the diet they maintained was a more realistic comparison.

This study used one brand of olive oil and a particular mixture of nuts (walnuts, hazelnuts, and almonds); it is possible that variations on either of these could affect the benefits of the diet.

CHALLENGES TO IMPLEMENTATION: Fitting a Mediterranean diet into an American lifestyle

The typical US diet is significantly different from that of most Spaniards. Americans may find it difficult to add either ≥4 tablespoons of olive oil or 30 g (1/3 cup) of nuts daily, for example, due to both cost and availability. Limited access to both individual and group counseling could be a barrier, as well.

On the other hand, this practice changer has the potential to simplify dietary counseling by allowing clinicians to focus on just one type of diet, for which there are many resources available both online and in print. We believe it makes sense to recommend a Mediterranean diet, while continuing to recommend increased exercise, smoking cessation, and improved control of chronic disease to lower patients’ risk of poor outcomes from CVD.

Acknowledgement

The PURLs Surveillance System was supported in part by Grant Number UL 1RR 024999 from the National Center for Research Resources, a Clinical Translational Science Award to the University of Chicago. The content is solely the responsibility of the authors and does not necessarily represent the official views of the National Center for Research Resources or the National Institutes of Health.

Practice changer

Counsel patients at high risk for cardiovascular disease and stroke to follow a Mediterranean diet, which is associated with a 30% risk reduction.1

Strength of recommendation

A: Based on one well-design randomized controlled trial (RCT).

Estruch R, Ros F, Salas-Salvado J, et al. Primary prevention of cardiovascular disease with a Mediterranean diet. N Engl J Med. 2013;368:1279-1290.

 

Illustrative case

A 62-year-old patient with diabetes, obesity, and a family history of early onset coronary artery disease is motivated to make significant lifestyle changes. You recommend moderate aerobic exercise for 30 minutes 5 times a week, but wonder whether a low-fat diet or a Mediterranean diet would be more effective in lowering her risk.

Cardiovascular disease (CVD), including heart disease and stroke, is the leading cause of mortality in the United States. CVD accounts for one in every 3 deaths,2 and stroke is a leading cause of long-term disability.2 The direct cost of treating CVD is estimated at $312.6 billion annually.2

Many modifiable risk factors contribute to CVD, including smoking, sedentary lifestyle, obesity, alcohol consumption, and poorly controlled chronic disease, as well as an unhealthy diet. A recent report from the American Heart Association suggests that 13% of deaths from CVD can be attributed to poor diet.2

Focus counseling on patients at risk
Primary care providers (PCPs) often struggle to effectively counsel patients on behavior change strategies, but face many barriers. Chief among them are the lack of time, training, and confidence in their counseling techniques, as well as a lack of patient motivation and readiness to change.3 In recognition of these barriers, the US Preventive Services Task Force recently recommended that PCPs focus behavioral counseling efforts on patients at high risk for heart disease.4

Large observational studies have found an association between trans fat and an increased risk of CVD, as well as a decreased risk of CVD in patients adhering to a Mediterranean diet.5-11 This type of diet typically includes a high intake of olive oil, fruit, nuts, vegetables, and cereals; moderate intake of fish and poultry; and low intake of dairy products, red meat, processed meats, and sweets. It also includes wine in moderation, consumed with meals.

Data on the physiologic properties of olive oil, including its antioxidant, vasodilating, and antiplatelet effects—as well as its effects on low-density lipoprotein cholesterol (LDL-C) that may inhibit atherogenesis—support the link between a Mediterranean diet and a decreased risk of CVD found in the observational studies.12,13 Until recently, however, no RCT had compared the effect of a Mediterranean diet with that of a low-fat diet for primary prevention of CVD.

STUDY SUMMARY: Mediterranean diet significantly lowers risk

Prevencion con Dieta Mediterranea (PREDIMED) was a large RCT (N=7447) comparing 2 variations of a Mediterranean diet with a low-fat diet for primary prevention of CVD. This Spanish study enrolled men 55 to 80 years of age and women ages 60 to 80 at high risk for developing CVD. The risk was based on either a diagnosis of type 2 diabetes or the presence of ≥3 major risk factors, including smoking, hypertension, elevated LDL-C, low high-density lipoprotein cholesterol, overweight or obese, and a family history of early heart disease.

A Mediterranean diet, with increased intake of either olive oil or mixed nuts, is more protective against CVD than a recommended low-fat diet.Participants were randomly assigned to one of 3 dietary groups: One group was assigned to a Mediterranean diet supplemented with ≥4 tablespoons per day of extra virgin olive oil; a second group was put on a Mediterranean diet supplemented by 30 grams (about 1/3 cup) of mixed nuts daily; a third group (the controls) was advised to follow a low-fat diet. The majority of baseline characteristics and medications taken throughout the study were similar among all 3 groups.

Those in both Mediterranean diet groups were followed for a median of 4.8 years, during which time they received quarterly dietary classes and individual and group counseling. The controls received baseline training, plus a leaflet about low-fat diets annually. In year 3, however, the researchers began giving the control group the same level of counseling as those in the Mediterranean diet groups to avoid confounding results.

Adherence to the diets was determined by a self-reported 14-item dietary screening questionnaire, plus urinary hydroxytyrosol and serum alpha-linoleic acid levels to assess for olive oil and mixed nut compliance. Self-reporting5 and biometric data indicated good compliance with the Mediterranean diets, and there was no difference found in levels of exercise among the groups.

After 5 years, those in the Mediterranean diet groups had consumed significantly more olive oil, nuts, vegetables, fruits, wine, legumes, fish, seafood, and sofrito sauce (a popular tomato-based sauce) than the control group. Participants in the low-fat diet group had decreased their fat intake by 2%, while those in the Mediterranean groups had increased fat intake (by 2.03% for the olive oil group and 2.1% for the nut group). Overall, 37% of energy intake by those in the low-fat diet group came from fat (exceeding the <30% of calories derived from fat intake that defines a low-fat diet) vs 39% fat intake for those in both Mediterranean diet groups.

 

 

 

The primary outcome was a composite of myocardial infarction (MI), stroke, and death from cardiovascular causes, and there were clinically meaningful and statistically significant differences between the Mediterranean diet groups and the controls. The primary outcome rate for the supplemental olive oil group was 3.8%; 3.4% for the extra nuts group; and 4.4% for the controls. This represents a 30% reduction in risk for combined stroke, MI, and death due to cardiovascular causes for the Mediterranean diet groups (hazard ratio [HR]=0.7; 95% confidence interval [CI], 0.53-0.91; P=.009; number needed to treat [NNT]=148 for the olive oil group and HR=0.7; 95% CI, 0.53-0.94; P=.02; NNT=100 for the group consuming extra nuts). Similar benefits were found in the multivariable adjusted analyses. The results correspond to 3 fewer events (stroke, MI, or cardiovascular death) per 1000 person-years for this high-risk population.

The only individual outcome that showed a significant decrease was stroke, with an NNT of 125 in both Mediterranean diet groups. Outcomes for the controls were similar before and after they began receiving quarterly counseling.

WHAT'S NEW?: Mediterranean diet is better than a lower-fat regimen

Rates of CVD are higher in the United States than in Spain, so implementing a Mediterranean diet on a large scale in this country has the potential to produce a greater response than that seen in this study. This study indicates that a Mediterranean diet, with increased intake of either olive oil or mixed nuts, is more protective against CVD than a recommended low-fat diet. It also shows that advising patients at high risk to follow a Mediterranean diet, providing dietary counseling, and monitoring them for adherence, rather than simply recommending a low-fat diet, can significantly decrease the risk of stroke.

Rates of CVD are higher in the United States than in Spain, so implementing a Mediterranean diet on a large scale in this country has the potential to produce a greater response than that seen in this study.

CAVEATS: Would a true low-fat diet be a better comparison?

Although the control group’s diet was meant to be low fat, the participants did not achieve this, possibly due to the relatively low level of dietary education and personalized counseling at the start of the study. Their inability to reach the <30% fat target could also reflect the difficulty patients have, in general, in decreasing fat content in their diet, which may mean the diet they maintained was a more realistic comparison.

This study used one brand of olive oil and a particular mixture of nuts (walnuts, hazelnuts, and almonds); it is possible that variations on either of these could affect the benefits of the diet.

CHALLENGES TO IMPLEMENTATION: Fitting a Mediterranean diet into an American lifestyle

The typical US diet is significantly different from that of most Spaniards. Americans may find it difficult to add either ≥4 tablespoons of olive oil or 30 g (1/3 cup) of nuts daily, for example, due to both cost and availability. Limited access to both individual and group counseling could be a barrier, as well.

On the other hand, this practice changer has the potential to simplify dietary counseling by allowing clinicians to focus on just one type of diet, for which there are many resources available both online and in print. We believe it makes sense to recommend a Mediterranean diet, while continuing to recommend increased exercise, smoking cessation, and improved control of chronic disease to lower patients’ risk of poor outcomes from CVD.

Acknowledgement

The PURLs Surveillance System was supported in part by Grant Number UL 1RR 024999 from the National Center for Research Resources, a Clinical Translational Science Award to the University of Chicago. The content is solely the responsibility of the authors and does not necessarily represent the official views of the National Center for Research Resources or the National Institutes of Health.

References

1. Estruch R, Ros F, Salas-Salvado J, et al. Primary prevention of cardiovascular disease with a Mediterranean Diet. N Engl J Med. 2013;368:1279-1290.

2. Go AS, Mozaffarian D, Roger VL, et al; American Heart Association Statistics Committee and Stroke Statistics Subcommittee. Heart disease and stroke statistics—2013 update: a report from the American Heart Association. Circulation. 2013;127:e6-e245.

3. Kushner RF. Barriers to providing nutrition counseling by physicians: a survey of primary care practitioners. Prev Med. 1995;24:546-552.

4. Behavioral Counseling to Promote a Healthful Diet and Physical Activity for Cardiovascular Disease Prevention in Adults, topic page. US Preventive Services Task Force Web site. Available at: http://www.uspreventiveservicestaskforce.org/uspstf/usp- sphys.htm. Accessed August 1, 2013.

5. Hu FB, Stampfer MJ, Manson JE, et al. Dietary fat intake and the risk of coronary heart disease in women. N Engl J Med. 1997;337:1491-1499.

6. Oomen C, Ocké MC, Feskens JM, et al. Association between trans fatty acid intake and 10-year risk of coronary heart disease in the Zutphen Elderly Study: a prospective population-based study. Lancet. 2001;357:746-751.

7. de Lorgeril M, Salen P, Martin JL, et al. Mediterranean diet, traditional risk factors and the rate of cardiovascular complications after myocardial infarction. Final report of the Lyon Diet Heart Study. Circulation. 1999;99:779-785.

8. Knoops KT, de Groot LC, Kromhout D, et al. Mediterranean diet, lifestyle factors, and 10-year mortality in elderly European men and women: the HALE project. JAMA. 2004;292:1433-1439.

9. Kris-Etherton P, Eckel RH, Howard BV, et al; Nutrition Committee Population Science Committee and Clinical Science Committee of the American Heart Association. Lyon Diet Heart Study. Benefits of a Mediterranean-style, National Education Program/AHA Step 1 Dietary Pattern on cardiovascular disease. Circulation. 2001;103:1823-1825.

10. Panagiotakos DB, Chrysohoou C, Pitsavos C, et al. The association of Mediterranean diet with lower risk of acute coronary syndromes, in hypertensive subjects. Int J Cardiol. 2002;82:141-147.

11. Panagiotakos DB, Pitsavos C, Chrysohoou C, et al. The role of traditional Mediterranean-type of diet and lifestyle, in the devel- opment of acute coronary syndromes: preliminary results from CARDIO 2000 study. Centr Eur J Public Health. 2002;10:11-15.

12. Esposito K, Marfella R, Ciotola M, et al. Effect of a Mediterranean-style diet on endothelial dysfunction and markers of vascular inflammation in the metabolic syndrome: a randomized trial. JAMA. 2004;292:1440-1446.

13. Vincent-Baudry S, Defoort C, Gerber M, et al. The Medi-RIVAGE study: reduction of cardiovascular disease risk factors after a 3-mo intervention with a Mediterranean-type diet or a low-fat diet. Am J Clin Nutr. 2005;82:964-971.

References

1. Estruch R, Ros F, Salas-Salvado J, et al. Primary prevention of cardiovascular disease with a Mediterranean Diet. N Engl J Med. 2013;368:1279-1290.

2. Go AS, Mozaffarian D, Roger VL, et al; American Heart Association Statistics Committee and Stroke Statistics Subcommittee. Heart disease and stroke statistics—2013 update: a report from the American Heart Association. Circulation. 2013;127:e6-e245.

3. Kushner RF. Barriers to providing nutrition counseling by physicians: a survey of primary care practitioners. Prev Med. 1995;24:546-552.

4. Behavioral Counseling to Promote a Healthful Diet and Physical Activity for Cardiovascular Disease Prevention in Adults, topic page. US Preventive Services Task Force Web site. Available at: http://www.uspreventiveservicestaskforce.org/uspstf/usp- sphys.htm. Accessed August 1, 2013.

5. Hu FB, Stampfer MJ, Manson JE, et al. Dietary fat intake and the risk of coronary heart disease in women. N Engl J Med. 1997;337:1491-1499.

6. Oomen C, Ocké MC, Feskens JM, et al. Association between trans fatty acid intake and 10-year risk of coronary heart disease in the Zutphen Elderly Study: a prospective population-based study. Lancet. 2001;357:746-751.

7. de Lorgeril M, Salen P, Martin JL, et al. Mediterranean diet, traditional risk factors and the rate of cardiovascular complications after myocardial infarction. Final report of the Lyon Diet Heart Study. Circulation. 1999;99:779-785.

8. Knoops KT, de Groot LC, Kromhout D, et al. Mediterranean diet, lifestyle factors, and 10-year mortality in elderly European men and women: the HALE project. JAMA. 2004;292:1433-1439.

9. Kris-Etherton P, Eckel RH, Howard BV, et al; Nutrition Committee Population Science Committee and Clinical Science Committee of the American Heart Association. Lyon Diet Heart Study. Benefits of a Mediterranean-style, National Education Program/AHA Step 1 Dietary Pattern on cardiovascular disease. Circulation. 2001;103:1823-1825.

10. Panagiotakos DB, Chrysohoou C, Pitsavos C, et al. The association of Mediterranean diet with lower risk of acute coronary syndromes, in hypertensive subjects. Int J Cardiol. 2002;82:141-147.

11. Panagiotakos DB, Pitsavos C, Chrysohoou C, et al. The role of traditional Mediterranean-type of diet and lifestyle, in the devel- opment of acute coronary syndromes: preliminary results from CARDIO 2000 study. Centr Eur J Public Health. 2002;10:11-15.

12. Esposito K, Marfella R, Ciotola M, et al. Effect of a Mediterranean-style diet on endothelial dysfunction and markers of vascular inflammation in the metabolic syndrome: a randomized trial. JAMA. 2004;292:1440-1446.

13. Vincent-Baudry S, Defoort C, Gerber M, et al. The Medi-RIVAGE study: reduction of cardiovascular disease risk factors after a 3-mo intervention with a Mediterranean-type diet or a low-fat diet. Am J Clin Nutr. 2005;82:964-971.

Issue
The Journal of Family Practice - 62(12)
Issue
The Journal of Family Practice - 62(12)
Page Number
745-746, 748
Page Number
745-746, 748
Publications
Publications
Topics
Article Type
Display Headline
Mediterranean diet: Higher fat but lower risk
Display Headline
Mediterranean diet: Higher fat but lower risk
Legacy Keywords
Mark Gwynne; DO; Anne Mounsey; MD; diet; nutrition; mediterranean diet; cardiovascular disease; CVD
Legacy Keywords
Mark Gwynne; DO; Anne Mounsey; MD; diet; nutrition; mediterranean diet; cardiovascular disease; CVD
Sections
PURLs Copyright

Copyright © 2013 Family Physicians Inquiries Network. All rights reserved.

Disallow All Ads
Alternative CME
Use ProPublica
Hide sidebar & use full width
render the right sidebar.
Conference Recap Checkbox
Not Conference Recap
Clinical Edge
Display the Slideshow in this Article
Article PDF Media
Media Files

Do oral contraceptives carry a significant risk of stroke for women with migraines?

Article Type
Changed
Mon, 01/14/2019 - 13:43
Display Headline
Do oral contraceptives carry a significant risk of stroke for women with migraines?
EVIDENCE-BASED ANSWER:

Perhaps. Estrogen-containing oral contraceptives may raise the risk of ischemic stroke in women with migraine, particularly migraine with aura (strength of recommendation [SOR]: C, small case-control studies with methodological flaws and conflicting results).

 

EVIDENCE SUMMARY

Women with probable migraine with visual aura (PMVA) have an increased risk for ischemic stroke (odds ratio [OR]=2.1) but not hemorrhagic stroke.1 Women with >12 PMVA episodes per year are at greatest risk (OR=2.2, compared with <12 PMVA episodes per year, OR=1.1).2 Women taking oral contraceptive pills (OCPs) also have an increased risk for stroke, depending on the estrogen dose (OR=4.8 for 50 mcg; OR=2.7 for 30-40 mcg; OR=1.7 for 20 mcg; and OR=1.0 for progestin-only pills).3

Women with migraines who smoke and take OCPs have the highest risk

Four case-control studies evaluated the risk of ischemic stroke in women with migraines who take OCPs. The first study compared the OR among 135 women 15 to 49 years of age with PMVA and a first ischemic stroke with 614 controls (no history of stroke, matched for age and ethnicity).2 Although women with PMVA overall had an increased risk of ischemic stroke (OR=1.5; 95% confidence interval [CI], 1.1-2.0), a subgroup of women with PMVA who also were taking OCPs didn’t have a significantly greater stroke risk than women with PMVA who were not taking OCPs (OR=1.6; 95% CI, not given but reported as not significant; P=.87). Investigators didn’t specify the type of OCPs.

Women with PMVA who smoked had a greater risk of ischemic stroke (OR=1.5; 95% CI, 1.1-2.3), and women with PMVA who both smoked and took OCPs had the highest risk of ischemic stroke (OR =7.0; 95% CI, 1.4-22.8).

In women younger than 45 years, OCPs are associated with higher stroke risk

The second study compared the odds ratio for ischemic stroke among 47 women younger than 45 years with PMVA who were taking combined OCPs with 63 controls.3 Most OCPs contained 30 to 40 mcg estrogen. Women with PMVA taking a combined OCP had a higher risk of ischemic stroke (OR=13.9; 95% CI, 5.5-35.1) than women with PMVA who didn’t take OCPs (OR=3.7; 95% CI, 1.5-9.1). Investigators didn’t report the number of PMVA episodes per year among the women.

The third study compared the odds ratio for ischemic stroke among 10 women 20 to 44 years of age with migraines who were taking combined OCPs with 23 controls.4 Investigators didn’t specify the type of migraine, although classic migraine was approximately twice as common as simple migraine among the women in the larger study population. Women with migraine taking OCPs were more likely to have an ischemic stroke overall (OR=16.9; 95% CI, 2.7-106), with the exception of those taking OCPs with <50 mcg estrogen (4 patients) (OR=0.59; 95 % CI, 0.79-54.8).

The fourth study compared the OR for ischemic stroke among 4 women 18 to 44 years old who had a history of migraine (type not specified) and used low-dose OCPs with 14 controls. Women with migraines taking OCPs had a higher risk of ischemic stroke (OR=2.08; 95% CI, 1.19-3.65).5

RECOMMENDATIONS

The American College of Obstetricians and Gynecologists says that combined OCPs are contraindicated for women with migraine with focal neurologic symptoms such as aura.6 Although strokes are rare in women with migraine taking OCPs, the impact of a stroke is so devastating that clinicians should consider progestin-only, intrauterine, or barrier contraceptives for these women. However, physicians may consider combined OCPs for women younger than 35 years with migraine if they don’t have focal neurologic signs, don’t smoke, and are otherwise healthy.

The World Health Organization and Centers for Disease Control and Prevention state that women with a history of migraine who use combined OCPs are 2 to 4 times more likely to have an ischemic stroke than nonusers and conclude that combined OCP use in women older than 35 years with migraine, or migraine with aura at any age, represents an unacceptable health risk. However, the advantages of using combined OCPs in women younger than 35 years with migraine generally outweigh the theoretical or proven risks.7,8

References

1. Schürks M, Rist PM, Bigal ME, et al. Migraine and cardiovascular disease: systemic review and meta-analysis. BMJ. 2009;339:b3914.

2. MacClellan LR, Giles W, Cole J, et al. Probable migraine with visual aura and risk of ischemic stroke: the stroke prevention in young women study. Stroke. 2007;38:2438-2445.

3. Tzourio C, Tehindrazanarivelo A, Iglésias S, et al. Case-control study of migraine and risk of ischaemic stroke in young women. BMJ. 1995;310:830-833.

4. Chang C, Donaghy M, Poulter N, et al. Migraine and stroke in young women: case-control study. The World Health Organization Collaborative Study of Cardiovascular Disease and Steroid Hormone Contraception. BMJ. 1999;318:13-18.

5. Schwartz SM, Petitti DB, Siscovick DS, et al. Stroke and use of low-dose oral contraceptives in young women: a pooled analysis of two US studies. Stroke. 1998;29:2277-2284.

6. ACOG Committee on Practice Bulletins-Gynecology. ACOG Practice Bulletin No. 73. Use of hormonal contraception in women with coexisting medical conditions. Obstet Gynecol. 2006;107:1453-1472.

7. US Medical Eligibility Criteria for Contraceptive Use, 2010. Adapted from the World Health Organization Medical Eligibility Criteria for Contraceptive Use, 4th edition. Centers for Disease Control and Prevention Web site. Available at: http://www.cdc.gov/mmwr/preview/mmwrhtml/rr5904a1.htm. Accessed July 15, 2012.

8. Appendix B: Classifications for combined hormonal contraceptives. Centers for Disease Control and Prevention Web site. Available at: http://www.cdc.gov/mmwr/preview/mmwrhtml/rr5904a3.htm. Accessed July 15, 2012.

Article PDF
Author and Disclosure Information

Mallory McClester, MD
Anne Mounsey, MD

University of North Carolina, Chapel Hill

Leslie Mackler, MSLS
Director, Moses Cone Health System Library, Greensboro, NC

ASSISTANT EDITOR
E. Chris Vincent, MD
Swedish Family Medicine Residency—First Hill, Seattle, Wash

Issue
The Journal of Family Practice - 62(11)
Publications
Topics
Page Number
662-663
Legacy Keywords
Mallory McClester; MD; Anne Mounsey; MD; Leslie Mackler; MSLS; oral contraceptive; OC; migraines; ischemic stroke; probable migraine with visual aura; PMVA
Sections
Author and Disclosure Information

Mallory McClester, MD
Anne Mounsey, MD

University of North Carolina, Chapel Hill

Leslie Mackler, MSLS
Director, Moses Cone Health System Library, Greensboro, NC

ASSISTANT EDITOR
E. Chris Vincent, MD
Swedish Family Medicine Residency—First Hill, Seattle, Wash

Author and Disclosure Information

Mallory McClester, MD
Anne Mounsey, MD

University of North Carolina, Chapel Hill

Leslie Mackler, MSLS
Director, Moses Cone Health System Library, Greensboro, NC

ASSISTANT EDITOR
E. Chris Vincent, MD
Swedish Family Medicine Residency—First Hill, Seattle, Wash

Article PDF
Article PDF
EVIDENCE-BASED ANSWER:

Perhaps. Estrogen-containing oral contraceptives may raise the risk of ischemic stroke in women with migraine, particularly migraine with aura (strength of recommendation [SOR]: C, small case-control studies with methodological flaws and conflicting results).

 

EVIDENCE SUMMARY

Women with probable migraine with visual aura (PMVA) have an increased risk for ischemic stroke (odds ratio [OR]=2.1) but not hemorrhagic stroke.1 Women with >12 PMVA episodes per year are at greatest risk (OR=2.2, compared with <12 PMVA episodes per year, OR=1.1).2 Women taking oral contraceptive pills (OCPs) also have an increased risk for stroke, depending on the estrogen dose (OR=4.8 for 50 mcg; OR=2.7 for 30-40 mcg; OR=1.7 for 20 mcg; and OR=1.0 for progestin-only pills).3

Women with migraines who smoke and take OCPs have the highest risk

Four case-control studies evaluated the risk of ischemic stroke in women with migraines who take OCPs. The first study compared the OR among 135 women 15 to 49 years of age with PMVA and a first ischemic stroke with 614 controls (no history of stroke, matched for age and ethnicity).2 Although women with PMVA overall had an increased risk of ischemic stroke (OR=1.5; 95% confidence interval [CI], 1.1-2.0), a subgroup of women with PMVA who also were taking OCPs didn’t have a significantly greater stroke risk than women with PMVA who were not taking OCPs (OR=1.6; 95% CI, not given but reported as not significant; P=.87). Investigators didn’t specify the type of OCPs.

Women with PMVA who smoked had a greater risk of ischemic stroke (OR=1.5; 95% CI, 1.1-2.3), and women with PMVA who both smoked and took OCPs had the highest risk of ischemic stroke (OR =7.0; 95% CI, 1.4-22.8).

In women younger than 45 years, OCPs are associated with higher stroke risk

The second study compared the odds ratio for ischemic stroke among 47 women younger than 45 years with PMVA who were taking combined OCPs with 63 controls.3 Most OCPs contained 30 to 40 mcg estrogen. Women with PMVA taking a combined OCP had a higher risk of ischemic stroke (OR=13.9; 95% CI, 5.5-35.1) than women with PMVA who didn’t take OCPs (OR=3.7; 95% CI, 1.5-9.1). Investigators didn’t report the number of PMVA episodes per year among the women.

The third study compared the odds ratio for ischemic stroke among 10 women 20 to 44 years of age with migraines who were taking combined OCPs with 23 controls.4 Investigators didn’t specify the type of migraine, although classic migraine was approximately twice as common as simple migraine among the women in the larger study population. Women with migraine taking OCPs were more likely to have an ischemic stroke overall (OR=16.9; 95% CI, 2.7-106), with the exception of those taking OCPs with <50 mcg estrogen (4 patients) (OR=0.59; 95 % CI, 0.79-54.8).

The fourth study compared the OR for ischemic stroke among 4 women 18 to 44 years old who had a history of migraine (type not specified) and used low-dose OCPs with 14 controls. Women with migraines taking OCPs had a higher risk of ischemic stroke (OR=2.08; 95% CI, 1.19-3.65).5

RECOMMENDATIONS

The American College of Obstetricians and Gynecologists says that combined OCPs are contraindicated for women with migraine with focal neurologic symptoms such as aura.6 Although strokes are rare in women with migraine taking OCPs, the impact of a stroke is so devastating that clinicians should consider progestin-only, intrauterine, or barrier contraceptives for these women. However, physicians may consider combined OCPs for women younger than 35 years with migraine if they don’t have focal neurologic signs, don’t smoke, and are otherwise healthy.

The World Health Organization and Centers for Disease Control and Prevention state that women with a history of migraine who use combined OCPs are 2 to 4 times more likely to have an ischemic stroke than nonusers and conclude that combined OCP use in women older than 35 years with migraine, or migraine with aura at any age, represents an unacceptable health risk. However, the advantages of using combined OCPs in women younger than 35 years with migraine generally outweigh the theoretical or proven risks.7,8

EVIDENCE-BASED ANSWER:

Perhaps. Estrogen-containing oral contraceptives may raise the risk of ischemic stroke in women with migraine, particularly migraine with aura (strength of recommendation [SOR]: C, small case-control studies with methodological flaws and conflicting results).

 

EVIDENCE SUMMARY

Women with probable migraine with visual aura (PMVA) have an increased risk for ischemic stroke (odds ratio [OR]=2.1) but not hemorrhagic stroke.1 Women with >12 PMVA episodes per year are at greatest risk (OR=2.2, compared with <12 PMVA episodes per year, OR=1.1).2 Women taking oral contraceptive pills (OCPs) also have an increased risk for stroke, depending on the estrogen dose (OR=4.8 for 50 mcg; OR=2.7 for 30-40 mcg; OR=1.7 for 20 mcg; and OR=1.0 for progestin-only pills).3

Women with migraines who smoke and take OCPs have the highest risk

Four case-control studies evaluated the risk of ischemic stroke in women with migraines who take OCPs. The first study compared the OR among 135 women 15 to 49 years of age with PMVA and a first ischemic stroke with 614 controls (no history of stroke, matched for age and ethnicity).2 Although women with PMVA overall had an increased risk of ischemic stroke (OR=1.5; 95% confidence interval [CI], 1.1-2.0), a subgroup of women with PMVA who also were taking OCPs didn’t have a significantly greater stroke risk than women with PMVA who were not taking OCPs (OR=1.6; 95% CI, not given but reported as not significant; P=.87). Investigators didn’t specify the type of OCPs.

Women with PMVA who smoked had a greater risk of ischemic stroke (OR=1.5; 95% CI, 1.1-2.3), and women with PMVA who both smoked and took OCPs had the highest risk of ischemic stroke (OR =7.0; 95% CI, 1.4-22.8).

In women younger than 45 years, OCPs are associated with higher stroke risk

The second study compared the odds ratio for ischemic stroke among 47 women younger than 45 years with PMVA who were taking combined OCPs with 63 controls.3 Most OCPs contained 30 to 40 mcg estrogen. Women with PMVA taking a combined OCP had a higher risk of ischemic stroke (OR=13.9; 95% CI, 5.5-35.1) than women with PMVA who didn’t take OCPs (OR=3.7; 95% CI, 1.5-9.1). Investigators didn’t report the number of PMVA episodes per year among the women.

The third study compared the odds ratio for ischemic stroke among 10 women 20 to 44 years of age with migraines who were taking combined OCPs with 23 controls.4 Investigators didn’t specify the type of migraine, although classic migraine was approximately twice as common as simple migraine among the women in the larger study population. Women with migraine taking OCPs were more likely to have an ischemic stroke overall (OR=16.9; 95% CI, 2.7-106), with the exception of those taking OCPs with <50 mcg estrogen (4 patients) (OR=0.59; 95 % CI, 0.79-54.8).

The fourth study compared the OR for ischemic stroke among 4 women 18 to 44 years old who had a history of migraine (type not specified) and used low-dose OCPs with 14 controls. Women with migraines taking OCPs had a higher risk of ischemic stroke (OR=2.08; 95% CI, 1.19-3.65).5

RECOMMENDATIONS

The American College of Obstetricians and Gynecologists says that combined OCPs are contraindicated for women with migraine with focal neurologic symptoms such as aura.6 Although strokes are rare in women with migraine taking OCPs, the impact of a stroke is so devastating that clinicians should consider progestin-only, intrauterine, or barrier contraceptives for these women. However, physicians may consider combined OCPs for women younger than 35 years with migraine if they don’t have focal neurologic signs, don’t smoke, and are otherwise healthy.

The World Health Organization and Centers for Disease Control and Prevention state that women with a history of migraine who use combined OCPs are 2 to 4 times more likely to have an ischemic stroke than nonusers and conclude that combined OCP use in women older than 35 years with migraine, or migraine with aura at any age, represents an unacceptable health risk. However, the advantages of using combined OCPs in women younger than 35 years with migraine generally outweigh the theoretical or proven risks.7,8

References

1. Schürks M, Rist PM, Bigal ME, et al. Migraine and cardiovascular disease: systemic review and meta-analysis. BMJ. 2009;339:b3914.

2. MacClellan LR, Giles W, Cole J, et al. Probable migraine with visual aura and risk of ischemic stroke: the stroke prevention in young women study. Stroke. 2007;38:2438-2445.

3. Tzourio C, Tehindrazanarivelo A, Iglésias S, et al. Case-control study of migraine and risk of ischaemic stroke in young women. BMJ. 1995;310:830-833.

4. Chang C, Donaghy M, Poulter N, et al. Migraine and stroke in young women: case-control study. The World Health Organization Collaborative Study of Cardiovascular Disease and Steroid Hormone Contraception. BMJ. 1999;318:13-18.

5. Schwartz SM, Petitti DB, Siscovick DS, et al. Stroke and use of low-dose oral contraceptives in young women: a pooled analysis of two US studies. Stroke. 1998;29:2277-2284.

6. ACOG Committee on Practice Bulletins-Gynecology. ACOG Practice Bulletin No. 73. Use of hormonal contraception in women with coexisting medical conditions. Obstet Gynecol. 2006;107:1453-1472.

7. US Medical Eligibility Criteria for Contraceptive Use, 2010. Adapted from the World Health Organization Medical Eligibility Criteria for Contraceptive Use, 4th edition. Centers for Disease Control and Prevention Web site. Available at: http://www.cdc.gov/mmwr/preview/mmwrhtml/rr5904a1.htm. Accessed July 15, 2012.

8. Appendix B: Classifications for combined hormonal contraceptives. Centers for Disease Control and Prevention Web site. Available at: http://www.cdc.gov/mmwr/preview/mmwrhtml/rr5904a3.htm. Accessed July 15, 2012.

References

1. Schürks M, Rist PM, Bigal ME, et al. Migraine and cardiovascular disease: systemic review and meta-analysis. BMJ. 2009;339:b3914.

2. MacClellan LR, Giles W, Cole J, et al. Probable migraine with visual aura and risk of ischemic stroke: the stroke prevention in young women study. Stroke. 2007;38:2438-2445.

3. Tzourio C, Tehindrazanarivelo A, Iglésias S, et al. Case-control study of migraine and risk of ischaemic stroke in young women. BMJ. 1995;310:830-833.

4. Chang C, Donaghy M, Poulter N, et al. Migraine and stroke in young women: case-control study. The World Health Organization Collaborative Study of Cardiovascular Disease and Steroid Hormone Contraception. BMJ. 1999;318:13-18.

5. Schwartz SM, Petitti DB, Siscovick DS, et al. Stroke and use of low-dose oral contraceptives in young women: a pooled analysis of two US studies. Stroke. 1998;29:2277-2284.

6. ACOG Committee on Practice Bulletins-Gynecology. ACOG Practice Bulletin No. 73. Use of hormonal contraception in women with coexisting medical conditions. Obstet Gynecol. 2006;107:1453-1472.

7. US Medical Eligibility Criteria for Contraceptive Use, 2010. Adapted from the World Health Organization Medical Eligibility Criteria for Contraceptive Use, 4th edition. Centers for Disease Control and Prevention Web site. Available at: http://www.cdc.gov/mmwr/preview/mmwrhtml/rr5904a1.htm. Accessed July 15, 2012.

8. Appendix B: Classifications for combined hormonal contraceptives. Centers for Disease Control and Prevention Web site. Available at: http://www.cdc.gov/mmwr/preview/mmwrhtml/rr5904a3.htm. Accessed July 15, 2012.

Issue
The Journal of Family Practice - 62(11)
Issue
The Journal of Family Practice - 62(11)
Page Number
662-663
Page Number
662-663
Publications
Publications
Topics
Article Type
Display Headline
Do oral contraceptives carry a significant risk of stroke for women with migraines?
Display Headline
Do oral contraceptives carry a significant risk of stroke for women with migraines?
Legacy Keywords
Mallory McClester; MD; Anne Mounsey; MD; Leslie Mackler; MSLS; oral contraceptive; OC; migraines; ischemic stroke; probable migraine with visual aura; PMVA
Legacy Keywords
Mallory McClester; MD; Anne Mounsey; MD; Leslie Mackler; MSLS; oral contraceptive; OC; migraines; ischemic stroke; probable migraine with visual aura; PMVA
Sections
PURLs Copyright

Evidence-based answers from the Family Physicians Inquiries Network

Disallow All Ads
Article PDF Media

Is self-swabbing for STIs a good idea?

Article Type
Changed
Mon, 01/14/2019 - 13:44
Display Headline
Is self-swabbing for STIs a good idea?
Practice changer

Ask women who are at risk for sexually transmitted infections (STIs) to self-swab for chlamydia and gonorrhea testing; self-collection of vulvovaginal swabs with nucleic acid amplification testing (NAAT) has excellent sensitivity in women with and without symptoms.1,2

Strength of recommendation

B: Based on prospective diagnostic cohort study.

Schoeman SA, Stewart CM, Booth RA, et al. Assessment of best single sample for finding chlamydia in women with and without symptoms: a diagnostic test study. BMJ. 2012;345:e8013.

Stewart CM, Schoeman SA, Booth RA, et al. Assessment of self taken swabs versus clinician taken swab cultures for diagnosing gonorrhoea in women: single centre, diagnostic accuracy study. BMJ. 2012;345:e8107.

 

Illustrative case

An 18-year-old woman comes to your office requesting testing for STIs. She has no symptoms. What is the best way to collect samples for chlamydia and gonorrhea testing?

Despite public health efforts, chlamydia and gonorrhea remain significant health problems, with more than 1.4 million cases of chlamydia and 321,849 cases of gonorrhea reported in the United States in 2011.3 Both can have devastating effects on reproduction, even in women who are asymptomatic.

Annual testing is recommended for women at risk
According to the Centers for Disease Control and Prevention (CDC), most reported cases of chlamydia (70%) and gonorrhea (62%) occur in men and women between the ages of 15 and 24 years.3 Both the CDC and the US Preventive Services Task Force recommend annual chlamydia screening for all sexually active women younger than 25, and for older women with risk factors, including having multiple sex partners and living in communities with a high burden of disease.4,5 Annual gonorrhea screening is recommended for sexually active women with risk factors, as well.4,5

How best to test? A number of unknowns
NAAT is the most sensitive test for detection of chlamydia and gonorrhea, but other questions about how best to screen for STIs remain.1,6 It has not been clear whether self-collected vulvovaginal swabs are equivalent to clinician-collected urethral or endocervical swabs for the detection of gonorrhea, or whether NAAT testing of the self-collected swabs or culture of the clinician-collected swabs is a more sensitive test for gonorrhea.

While some studies have found self-collected vulvovaginal samples to be as sensitive as clinician-collected endocervical samples for the diagnosis of chlamydia and gonorrhea, samples are still often collected by clinicians.7,8 Collecting endocervical swabs is uncomfortable for patients and time consuming for clinicians, and evidence suggests that patients prefer noninvasive sampling.9

STUDY SUMMARY: Self-collected samples are highly sensitive

This study was designed to compare the sensitivity and specificity of self-collected vulvovaginal swabs vs clinician-collected swabs for chlamydia and gonorrhea, both in asymptomatic women and women with symptoms of an STI. Test methods were also assessed for gonorrhea, comparing detection rates of self-swabs tested with NAAT vs the culture of clinician-collected urethral and endocervical samples.

The researchers evaluated a total of 3973 women, ages 16 to 59 years, who sought care at a single sexual health center in the United Kingdom. The average age was 25 years; 37% of the participants reported a prior STI, and 42% had at least one symptom suggestive of an STI. Exclusion criteria included having taken an antibiotic in the preceding 28 days and being unable or unwilling to take a vulvovaginal swab or undergo clinician examination and sample collection.

Self-collected vulvovaginal swabs are the sample of choice for both chlamydia and gonorrhea testing in women, regardless of whether patients have symptoms.The women performed vulvovaginal swabs for NAAT (Aptima Combo-2, Hologic GenProbe, San Diego, Calif) prior to a speculum exam; endocervical swab for both NAAT and culture and a urethral swab for culture were collected by the clinician. All the swabs sent for NAAT were tested for chlamydia and gonorrhea, and cultures were performed to detect gonorrhea.

Chlamydia: Vulvovaginal swabs have higher detection rates

Of the 3867 participants with complete results, 10.2% were infected with chlamydia. Self-collected vulvovaginal swabs were significantly more sensitive than endocervical swabs (97% vs 88%; P<.00001) and had equal specificity (99.9% vs 100%). In women with symptoms of an STI, the sensitivity was 97% vs 88% (P<.0008); in those with no symptoms, the sensitivity was 97% vs 89% (P<.002).

 

 

 

Gonorrhea: Self-collection, NAAT yield better results

Gonorrhea was found in 2.5% of the 3859 women with complete results for testing of this STI. Self-collected swabs and physician-collected swabs analyzed by NAAT both had excellent sensitivity (99% vs 96%; P=.375). But self-collected samples that underwent NAAT were significantly more sensitive than clinician-collected urethral and endocervical samples that were cultured (99% vs 81%; P<.001). The number needed to test by self-collection for NAAT (compared with clinician-collected culture) to detect one additional case of gonorrhea was 5.

In women with symptoms suggestive of infection, the NAAT assays—both physician- and self-collected—were equivalent and were more sensitive than gonorrhea culture (P=.004). In asymptomatic women, 1.8% of whom had gonorrhea, the vulvovaginal swab sent for NAAT was more sensitive than culture (98% vs 78%; P=0.008) and equivalent to the endocervical swab for NAAT (90%).

The bottom line: Self-collected vulvovaginal swabs are the sample of choice for both chlamydia and gonorrhea testing in women, regardless of whether they have symptoms. When a clinical examination is needed, either the clinician or the patient can collect a vulvovaginal swab.

WHAT'S NEW: Endocervical samples, cultures have lower detection rates

In this study, endocervical samples collected by the physician rather than self-collected vulvovaginal samples would have missed 9% (one in 11) of chlamydial infections in women with symptoms of an STI. Vulvovaginal swabs and endocervical swabs have equal sensitivity for the diagnosis of gonorrhea when NAAT is used, but culture would have missed one in 5 gonorrhea infections (in women with and without symptoms).

CAVEATS: NAAT is costly, and does not test for drug sensitivity

Although NAAT has replaced cell culture methodology as the gold standard for gonorrhea and chlamydia diagnosis, it is potentially costly if not readily available in your practice setting. What’s more, NAAT does not allow testing for antibiotic sensitivity, which is particularly relevant with increasing resistance of gonorrhea to multiple antibiotics. In addition, it’s unclear whether these results would apply to all NAAT assays or just the one used in this study.

These studies examine sensitivity and specificity of gonorrhea and chlamydia testing in a high-risk population—women who were seeking care in a sexual health center. Your patient population may be lower risk, which will lower the prevalence of STIs and lower the positive predictive value of NAAT. A positive NAAT test for an STI should be followed by a confirmation NAAT in low-risk populations.

CHALLENGES TO IMPLEMENTATION: Reconsidering the way we practice

Most family physicians are accustomed to performing a full examination on patients with a suspected STI, and changing the flow of the office visit may be difficult. And, to implement this practice changer properly, it would be necessary to provide patient instruction in self-collection technique.

Also, making this change could be costly if you do not have this particular NAAT available. Once implemented, however, self-collection with NAAT will likely save time and be more comfortable for your patients. It will also provide a higher sensitivity in detecting chlamydia infections and equal sensitivity in detecting gonorrhea compared with physician-collected NAAT testing.

ACKNOWLEDGEMENT
The PURLs surveillance system was supported in part by Grant Number UL1RR024999 from the National center for Research Resources, a clinical Translational science Award to the University of Chicago. The content is solely the responsibility of the authors and does not necessarily represent the official views of the National center for Research Resources or the National Institutes of Health.

Click here to view PURL METHODOLOGY 1

Click here to view PURL METHODOLOGY 2

References

1. Schoeman SA, Stewart CM, Booth RA, et al. Assessment of best single sample for finding chlamydia in women with and without symptoms: a diagnostic test study. BMJ. 2012;345:e8013.

2. Stewart CM, Schoeman SA, Booth RA, et al. Assessment of self taken swabs versus clinician taken swab for cultures for diagnosing gonorrhea in women: single centre, diagnostic accuracy study. BMJ. 2012;345:e8107.

3. Centers for Disease Control and Prevention. Fact sheet: STD trends in the United States, 2011 national data for chlamydia, gonorrhea, and syphilis. Atlanta, GA: US Department of Health and Human Services, Centers for Disease Control and Prevention, 2012. http://www.cdc.gov/std/stats11/trends-2011.pdf. Accessed July 17, 2013.

4. Centers for Disease Control and Prevention. Fact sheet: Incidence, prevalence, and cost of sexually transmitted infections in the United States. Atlanta, GA: US Department of Health and Human Services, Centers for Disease Control and Prevention, 2013. http://www.cdc.gov/std/stats/STI-Estimates-Fact-Sheet-Feb-2013.pdf. Accessed July 17, 2013.

5. USPSTF Recommendations for STI screening. United States Preventive
Services Task Force Web site. http://www.uspreventiveservicestaskforce.org/uspstf08/methods/stinfections.htm Accessed August 29, 2013.

6. 2010 STD Treatment Guidelines. Centers for Disease Control and Prevention Web site. http://www.cdc.gov/std/treatment/2010/default.htm. Accessed October 23, 2013.

7. Cook RL, Hutchison SL, Østergaard L, Braithwaite RS, Ness RB. Systematic review: Non-invasive testing for Chlamydia trachomatis and Neisseria gonorrhoeae. Ann Intern Med. 2005:142:914-925.

8. Moss S, Mallinson H. The contribution of APTIMA Combo 2 assay to the diagnosis of gonorrhoea in genitourinary medicine setting. Int J STD AIDS. 2007;18:551-554.

9. Chernesky MA, Hook EW 3rd, Martin DH, et al. Women find it easy and prefer to collect their own vaginal swabs to diagnose chlamydia trachomatis or Neisseria gonorrhaea infections. Sex Transm Dis. 2005;32:729-733.

Article PDF
Author and Disclosure Information

Cristy Page, MD, MPH
Anne Mounsey, MD
Kate Rowland, MD, MS

Department of Family Medicine, University of North Carolina at Chapel Hill (Drs. Page and Mounsey); Department of Family Medicine, University of Chicago (Dr. Rowland)

PURLs EDITOR
James Stevermer, MD, MPH
Department of Family Medicine, University of Missouri

Issue
The Journal of Family Practice - 62(11)
Publications
Topics
Page Number
651-653
Legacy Keywords
Cristy Page; MD; MPH; Anne Mounsey; MD; Kate Rowland; MD; MS; STIs; self-swabbing; gonorrhea; NAAT; nucleic acid amplification testing; sexually transmitted infections
Sections
Author and Disclosure Information

Cristy Page, MD, MPH
Anne Mounsey, MD
Kate Rowland, MD, MS

Department of Family Medicine, University of North Carolina at Chapel Hill (Drs. Page and Mounsey); Department of Family Medicine, University of Chicago (Dr. Rowland)

PURLs EDITOR
James Stevermer, MD, MPH
Department of Family Medicine, University of Missouri

Author and Disclosure Information

Cristy Page, MD, MPH
Anne Mounsey, MD
Kate Rowland, MD, MS

Department of Family Medicine, University of North Carolina at Chapel Hill (Drs. Page and Mounsey); Department of Family Medicine, University of Chicago (Dr. Rowland)

PURLs EDITOR
James Stevermer, MD, MPH
Department of Family Medicine, University of Missouri

Article PDF
Article PDF
Practice changer

Ask women who are at risk for sexually transmitted infections (STIs) to self-swab for chlamydia and gonorrhea testing; self-collection of vulvovaginal swabs with nucleic acid amplification testing (NAAT) has excellent sensitivity in women with and without symptoms.1,2

Strength of recommendation

B: Based on prospective diagnostic cohort study.

Schoeman SA, Stewart CM, Booth RA, et al. Assessment of best single sample for finding chlamydia in women with and without symptoms: a diagnostic test study. BMJ. 2012;345:e8013.

Stewart CM, Schoeman SA, Booth RA, et al. Assessment of self taken swabs versus clinician taken swab cultures for diagnosing gonorrhoea in women: single centre, diagnostic accuracy study. BMJ. 2012;345:e8107.

 

Illustrative case

An 18-year-old woman comes to your office requesting testing for STIs. She has no symptoms. What is the best way to collect samples for chlamydia and gonorrhea testing?

Despite public health efforts, chlamydia and gonorrhea remain significant health problems, with more than 1.4 million cases of chlamydia and 321,849 cases of gonorrhea reported in the United States in 2011.3 Both can have devastating effects on reproduction, even in women who are asymptomatic.

Annual testing is recommended for women at risk
According to the Centers for Disease Control and Prevention (CDC), most reported cases of chlamydia (70%) and gonorrhea (62%) occur in men and women between the ages of 15 and 24 years.3 Both the CDC and the US Preventive Services Task Force recommend annual chlamydia screening for all sexually active women younger than 25, and for older women with risk factors, including having multiple sex partners and living in communities with a high burden of disease.4,5 Annual gonorrhea screening is recommended for sexually active women with risk factors, as well.4,5

How best to test? A number of unknowns
NAAT is the most sensitive test for detection of chlamydia and gonorrhea, but other questions about how best to screen for STIs remain.1,6 It has not been clear whether self-collected vulvovaginal swabs are equivalent to clinician-collected urethral or endocervical swabs for the detection of gonorrhea, or whether NAAT testing of the self-collected swabs or culture of the clinician-collected swabs is a more sensitive test for gonorrhea.

While some studies have found self-collected vulvovaginal samples to be as sensitive as clinician-collected endocervical samples for the diagnosis of chlamydia and gonorrhea, samples are still often collected by clinicians.7,8 Collecting endocervical swabs is uncomfortable for patients and time consuming for clinicians, and evidence suggests that patients prefer noninvasive sampling.9

STUDY SUMMARY: Self-collected samples are highly sensitive

This study was designed to compare the sensitivity and specificity of self-collected vulvovaginal swabs vs clinician-collected swabs for chlamydia and gonorrhea, both in asymptomatic women and women with symptoms of an STI. Test methods were also assessed for gonorrhea, comparing detection rates of self-swabs tested with NAAT vs the culture of clinician-collected urethral and endocervical samples.

The researchers evaluated a total of 3973 women, ages 16 to 59 years, who sought care at a single sexual health center in the United Kingdom. The average age was 25 years; 37% of the participants reported a prior STI, and 42% had at least one symptom suggestive of an STI. Exclusion criteria included having taken an antibiotic in the preceding 28 days and being unable or unwilling to take a vulvovaginal swab or undergo clinician examination and sample collection.

Self-collected vulvovaginal swabs are the sample of choice for both chlamydia and gonorrhea testing in women, regardless of whether patients have symptoms.The women performed vulvovaginal swabs for NAAT (Aptima Combo-2, Hologic GenProbe, San Diego, Calif) prior to a speculum exam; endocervical swab for both NAAT and culture and a urethral swab for culture were collected by the clinician. All the swabs sent for NAAT were tested for chlamydia and gonorrhea, and cultures were performed to detect gonorrhea.

Chlamydia: Vulvovaginal swabs have higher detection rates

Of the 3867 participants with complete results, 10.2% were infected with chlamydia. Self-collected vulvovaginal swabs were significantly more sensitive than endocervical swabs (97% vs 88%; P<.00001) and had equal specificity (99.9% vs 100%). In women with symptoms of an STI, the sensitivity was 97% vs 88% (P<.0008); in those with no symptoms, the sensitivity was 97% vs 89% (P<.002).

 

 

 

Gonorrhea: Self-collection, NAAT yield better results

Gonorrhea was found in 2.5% of the 3859 women with complete results for testing of this STI. Self-collected swabs and physician-collected swabs analyzed by NAAT both had excellent sensitivity (99% vs 96%; P=.375). But self-collected samples that underwent NAAT were significantly more sensitive than clinician-collected urethral and endocervical samples that were cultured (99% vs 81%; P<.001). The number needed to test by self-collection for NAAT (compared with clinician-collected culture) to detect one additional case of gonorrhea was 5.

In women with symptoms suggestive of infection, the NAAT assays—both physician- and self-collected—were equivalent and were more sensitive than gonorrhea culture (P=.004). In asymptomatic women, 1.8% of whom had gonorrhea, the vulvovaginal swab sent for NAAT was more sensitive than culture (98% vs 78%; P=0.008) and equivalent to the endocervical swab for NAAT (90%).

The bottom line: Self-collected vulvovaginal swabs are the sample of choice for both chlamydia and gonorrhea testing in women, regardless of whether they have symptoms. When a clinical examination is needed, either the clinician or the patient can collect a vulvovaginal swab.

WHAT'S NEW: Endocervical samples, cultures have lower detection rates

In this study, endocervical samples collected by the physician rather than self-collected vulvovaginal samples would have missed 9% (one in 11) of chlamydial infections in women with symptoms of an STI. Vulvovaginal swabs and endocervical swabs have equal sensitivity for the diagnosis of gonorrhea when NAAT is used, but culture would have missed one in 5 gonorrhea infections (in women with and without symptoms).

CAVEATS: NAAT is costly, and does not test for drug sensitivity

Although NAAT has replaced cell culture methodology as the gold standard for gonorrhea and chlamydia diagnosis, it is potentially costly if not readily available in your practice setting. What’s more, NAAT does not allow testing for antibiotic sensitivity, which is particularly relevant with increasing resistance of gonorrhea to multiple antibiotics. In addition, it’s unclear whether these results would apply to all NAAT assays or just the one used in this study.

These studies examine sensitivity and specificity of gonorrhea and chlamydia testing in a high-risk population—women who were seeking care in a sexual health center. Your patient population may be lower risk, which will lower the prevalence of STIs and lower the positive predictive value of NAAT. A positive NAAT test for an STI should be followed by a confirmation NAAT in low-risk populations.

CHALLENGES TO IMPLEMENTATION: Reconsidering the way we practice

Most family physicians are accustomed to performing a full examination on patients with a suspected STI, and changing the flow of the office visit may be difficult. And, to implement this practice changer properly, it would be necessary to provide patient instruction in self-collection technique.

Also, making this change could be costly if you do not have this particular NAAT available. Once implemented, however, self-collection with NAAT will likely save time and be more comfortable for your patients. It will also provide a higher sensitivity in detecting chlamydia infections and equal sensitivity in detecting gonorrhea compared with physician-collected NAAT testing.

ACKNOWLEDGEMENT
The PURLs surveillance system was supported in part by Grant Number UL1RR024999 from the National center for Research Resources, a clinical Translational science Award to the University of Chicago. The content is solely the responsibility of the authors and does not necessarily represent the official views of the National center for Research Resources or the National Institutes of Health.

Click here to view PURL METHODOLOGY 1

Click here to view PURL METHODOLOGY 2

Practice changer

Ask women who are at risk for sexually transmitted infections (STIs) to self-swab for chlamydia and gonorrhea testing; self-collection of vulvovaginal swabs with nucleic acid amplification testing (NAAT) has excellent sensitivity in women with and without symptoms.1,2

Strength of recommendation

B: Based on prospective diagnostic cohort study.

Schoeman SA, Stewart CM, Booth RA, et al. Assessment of best single sample for finding chlamydia in women with and without symptoms: a diagnostic test study. BMJ. 2012;345:e8013.

Stewart CM, Schoeman SA, Booth RA, et al. Assessment of self taken swabs versus clinician taken swab cultures for diagnosing gonorrhoea in women: single centre, diagnostic accuracy study. BMJ. 2012;345:e8107.

 

Illustrative case

An 18-year-old woman comes to your office requesting testing for STIs. She has no symptoms. What is the best way to collect samples for chlamydia and gonorrhea testing?

Despite public health efforts, chlamydia and gonorrhea remain significant health problems, with more than 1.4 million cases of chlamydia and 321,849 cases of gonorrhea reported in the United States in 2011.3 Both can have devastating effects on reproduction, even in women who are asymptomatic.

Annual testing is recommended for women at risk
According to the Centers for Disease Control and Prevention (CDC), most reported cases of chlamydia (70%) and gonorrhea (62%) occur in men and women between the ages of 15 and 24 years.3 Both the CDC and the US Preventive Services Task Force recommend annual chlamydia screening for all sexually active women younger than 25, and for older women with risk factors, including having multiple sex partners and living in communities with a high burden of disease.4,5 Annual gonorrhea screening is recommended for sexually active women with risk factors, as well.4,5

How best to test? A number of unknowns
NAAT is the most sensitive test for detection of chlamydia and gonorrhea, but other questions about how best to screen for STIs remain.1,6 It has not been clear whether self-collected vulvovaginal swabs are equivalent to clinician-collected urethral or endocervical swabs for the detection of gonorrhea, or whether NAAT testing of the self-collected swabs or culture of the clinician-collected swabs is a more sensitive test for gonorrhea.

While some studies have found self-collected vulvovaginal samples to be as sensitive as clinician-collected endocervical samples for the diagnosis of chlamydia and gonorrhea, samples are still often collected by clinicians.7,8 Collecting endocervical swabs is uncomfortable for patients and time consuming for clinicians, and evidence suggests that patients prefer noninvasive sampling.9

STUDY SUMMARY: Self-collected samples are highly sensitive

This study was designed to compare the sensitivity and specificity of self-collected vulvovaginal swabs vs clinician-collected swabs for chlamydia and gonorrhea, both in asymptomatic women and women with symptoms of an STI. Test methods were also assessed for gonorrhea, comparing detection rates of self-swabs tested with NAAT vs the culture of clinician-collected urethral and endocervical samples.

The researchers evaluated a total of 3973 women, ages 16 to 59 years, who sought care at a single sexual health center in the United Kingdom. The average age was 25 years; 37% of the participants reported a prior STI, and 42% had at least one symptom suggestive of an STI. Exclusion criteria included having taken an antibiotic in the preceding 28 days and being unable or unwilling to take a vulvovaginal swab or undergo clinician examination and sample collection.

Self-collected vulvovaginal swabs are the sample of choice for both chlamydia and gonorrhea testing in women, regardless of whether patients have symptoms.The women performed vulvovaginal swabs for NAAT (Aptima Combo-2, Hologic GenProbe, San Diego, Calif) prior to a speculum exam; endocervical swab for both NAAT and culture and a urethral swab for culture were collected by the clinician. All the swabs sent for NAAT were tested for chlamydia and gonorrhea, and cultures were performed to detect gonorrhea.

Chlamydia: Vulvovaginal swabs have higher detection rates

Of the 3867 participants with complete results, 10.2% were infected with chlamydia. Self-collected vulvovaginal swabs were significantly more sensitive than endocervical swabs (97% vs 88%; P<.00001) and had equal specificity (99.9% vs 100%). In women with symptoms of an STI, the sensitivity was 97% vs 88% (P<.0008); in those with no symptoms, the sensitivity was 97% vs 89% (P<.002).

 

 

 

Gonorrhea: Self-collection, NAAT yield better results

Gonorrhea was found in 2.5% of the 3859 women with complete results for testing of this STI. Self-collected swabs and physician-collected swabs analyzed by NAAT both had excellent sensitivity (99% vs 96%; P=.375). But self-collected samples that underwent NAAT were significantly more sensitive than clinician-collected urethral and endocervical samples that were cultured (99% vs 81%; P<.001). The number needed to test by self-collection for NAAT (compared with clinician-collected culture) to detect one additional case of gonorrhea was 5.

In women with symptoms suggestive of infection, the NAAT assays—both physician- and self-collected—were equivalent and were more sensitive than gonorrhea culture (P=.004). In asymptomatic women, 1.8% of whom had gonorrhea, the vulvovaginal swab sent for NAAT was more sensitive than culture (98% vs 78%; P=0.008) and equivalent to the endocervical swab for NAAT (90%).

The bottom line: Self-collected vulvovaginal swabs are the sample of choice for both chlamydia and gonorrhea testing in women, regardless of whether they have symptoms. When a clinical examination is needed, either the clinician or the patient can collect a vulvovaginal swab.

WHAT'S NEW: Endocervical samples, cultures have lower detection rates

In this study, endocervical samples collected by the physician rather than self-collected vulvovaginal samples would have missed 9% (one in 11) of chlamydial infections in women with symptoms of an STI. Vulvovaginal swabs and endocervical swabs have equal sensitivity for the diagnosis of gonorrhea when NAAT is used, but culture would have missed one in 5 gonorrhea infections (in women with and without symptoms).

CAVEATS: NAAT is costly, and does not test for drug sensitivity

Although NAAT has replaced cell culture methodology as the gold standard for gonorrhea and chlamydia diagnosis, it is potentially costly if not readily available in your practice setting. What’s more, NAAT does not allow testing for antibiotic sensitivity, which is particularly relevant with increasing resistance of gonorrhea to multiple antibiotics. In addition, it’s unclear whether these results would apply to all NAAT assays or just the one used in this study.

These studies examine sensitivity and specificity of gonorrhea and chlamydia testing in a high-risk population—women who were seeking care in a sexual health center. Your patient population may be lower risk, which will lower the prevalence of STIs and lower the positive predictive value of NAAT. A positive NAAT test for an STI should be followed by a confirmation NAAT in low-risk populations.

CHALLENGES TO IMPLEMENTATION: Reconsidering the way we practice

Most family physicians are accustomed to performing a full examination on patients with a suspected STI, and changing the flow of the office visit may be difficult. And, to implement this practice changer properly, it would be necessary to provide patient instruction in self-collection technique.

Also, making this change could be costly if you do not have this particular NAAT available. Once implemented, however, self-collection with NAAT will likely save time and be more comfortable for your patients. It will also provide a higher sensitivity in detecting chlamydia infections and equal sensitivity in detecting gonorrhea compared with physician-collected NAAT testing.

ACKNOWLEDGEMENT
The PURLs surveillance system was supported in part by Grant Number UL1RR024999 from the National center for Research Resources, a clinical Translational science Award to the University of Chicago. The content is solely the responsibility of the authors and does not necessarily represent the official views of the National center for Research Resources or the National Institutes of Health.

Click here to view PURL METHODOLOGY 1

Click here to view PURL METHODOLOGY 2

References

1. Schoeman SA, Stewart CM, Booth RA, et al. Assessment of best single sample for finding chlamydia in women with and without symptoms: a diagnostic test study. BMJ. 2012;345:e8013.

2. Stewart CM, Schoeman SA, Booth RA, et al. Assessment of self taken swabs versus clinician taken swab for cultures for diagnosing gonorrhea in women: single centre, diagnostic accuracy study. BMJ. 2012;345:e8107.

3. Centers for Disease Control and Prevention. Fact sheet: STD trends in the United States, 2011 national data for chlamydia, gonorrhea, and syphilis. Atlanta, GA: US Department of Health and Human Services, Centers for Disease Control and Prevention, 2012. http://www.cdc.gov/std/stats11/trends-2011.pdf. Accessed July 17, 2013.

4. Centers for Disease Control and Prevention. Fact sheet: Incidence, prevalence, and cost of sexually transmitted infections in the United States. Atlanta, GA: US Department of Health and Human Services, Centers for Disease Control and Prevention, 2013. http://www.cdc.gov/std/stats/STI-Estimates-Fact-Sheet-Feb-2013.pdf. Accessed July 17, 2013.

5. USPSTF Recommendations for STI screening. United States Preventive
Services Task Force Web site. http://www.uspreventiveservicestaskforce.org/uspstf08/methods/stinfections.htm Accessed August 29, 2013.

6. 2010 STD Treatment Guidelines. Centers for Disease Control and Prevention Web site. http://www.cdc.gov/std/treatment/2010/default.htm. Accessed October 23, 2013.

7. Cook RL, Hutchison SL, Østergaard L, Braithwaite RS, Ness RB. Systematic review: Non-invasive testing for Chlamydia trachomatis and Neisseria gonorrhoeae. Ann Intern Med. 2005:142:914-925.

8. Moss S, Mallinson H. The contribution of APTIMA Combo 2 assay to the diagnosis of gonorrhoea in genitourinary medicine setting. Int J STD AIDS. 2007;18:551-554.

9. Chernesky MA, Hook EW 3rd, Martin DH, et al. Women find it easy and prefer to collect their own vaginal swabs to diagnose chlamydia trachomatis or Neisseria gonorrhaea infections. Sex Transm Dis. 2005;32:729-733.

References

1. Schoeman SA, Stewart CM, Booth RA, et al. Assessment of best single sample for finding chlamydia in women with and without symptoms: a diagnostic test study. BMJ. 2012;345:e8013.

2. Stewart CM, Schoeman SA, Booth RA, et al. Assessment of self taken swabs versus clinician taken swab for cultures for diagnosing gonorrhea in women: single centre, diagnostic accuracy study. BMJ. 2012;345:e8107.

3. Centers for Disease Control and Prevention. Fact sheet: STD trends in the United States, 2011 national data for chlamydia, gonorrhea, and syphilis. Atlanta, GA: US Department of Health and Human Services, Centers for Disease Control and Prevention, 2012. http://www.cdc.gov/std/stats11/trends-2011.pdf. Accessed July 17, 2013.

4. Centers for Disease Control and Prevention. Fact sheet: Incidence, prevalence, and cost of sexually transmitted infections in the United States. Atlanta, GA: US Department of Health and Human Services, Centers for Disease Control and Prevention, 2013. http://www.cdc.gov/std/stats/STI-Estimates-Fact-Sheet-Feb-2013.pdf. Accessed July 17, 2013.

5. USPSTF Recommendations for STI screening. United States Preventive
Services Task Force Web site. http://www.uspreventiveservicestaskforce.org/uspstf08/methods/stinfections.htm Accessed August 29, 2013.

6. 2010 STD Treatment Guidelines. Centers for Disease Control and Prevention Web site. http://www.cdc.gov/std/treatment/2010/default.htm. Accessed October 23, 2013.

7. Cook RL, Hutchison SL, Østergaard L, Braithwaite RS, Ness RB. Systematic review: Non-invasive testing for Chlamydia trachomatis and Neisseria gonorrhoeae. Ann Intern Med. 2005:142:914-925.

8. Moss S, Mallinson H. The contribution of APTIMA Combo 2 assay to the diagnosis of gonorrhoea in genitourinary medicine setting. Int J STD AIDS. 2007;18:551-554.

9. Chernesky MA, Hook EW 3rd, Martin DH, et al. Women find it easy and prefer to collect their own vaginal swabs to diagnose chlamydia trachomatis or Neisseria gonorrhaea infections. Sex Transm Dis. 2005;32:729-733.

Issue
The Journal of Family Practice - 62(11)
Issue
The Journal of Family Practice - 62(11)
Page Number
651-653
Page Number
651-653
Publications
Publications
Topics
Article Type
Display Headline
Is self-swabbing for STIs a good idea?
Display Headline
Is self-swabbing for STIs a good idea?
Legacy Keywords
Cristy Page; MD; MPH; Anne Mounsey; MD; Kate Rowland; MD; MS; STIs; self-swabbing; gonorrhea; NAAT; nucleic acid amplification testing; sexually transmitted infections
Legacy Keywords
Cristy Page; MD; MPH; Anne Mounsey; MD; Kate Rowland; MD; MS; STIs; self-swabbing; gonorrhea; NAAT; nucleic acid amplification testing; sexually transmitted infections
Sections
PURLs Copyright

Copyright © 2013 Family Physicians Inquiries Network. All rights reserved.

Disallow All Ads
Alternative CME
Article PDF Media

Should You Still Recommend Omega-3 Supplements?

Article Type
Changed
Tue, 12/13/2016 - 12:08
Display Headline
Should You Still Recommend Omega-3 Supplements?
Probably not, a new meta-analysis suggests. Read the details here.

PRACTICE CHANGER
Stop recommending omega-3 fatty acid supplements for cardiovascular protection. They have no significant impact on all-cause mortality, acute myocardial infarction (MI), sudden death, or stroke.1

STRENGTH OF RECOMMENDATION
A: Based on a meta-analysis of randomized controlled trials (RCTs).

ILLUSTRATIVE CASE
A 59-year-old patient who had an MI three years ago is taking an ACE inhibitor, a statin, and a -blocker. He asks you whether he should also take omega-3 fatty acid supplements to further decrease his risk for heart disease. What should you tell him?

Coronary artery disease (CAD) kills more than 500,000 Americans every year,2 and medical and dietary therapies for primary and secondary cardiovascular protection are paramount. Omega-3 polyunsaturated fatty acid (PUFA) supplementation is one such therapy.

Omega-3 PUFAs are precursors to certain prostaglandins that decrease the proinflammatory state in patients with CAD. They also lower triglyceride levels and produce an anti-arrhythmic effect by promoting electrical stability.

But do PUFA supplements ­provide cardioprotection?

The American Heart Association’s Nutrition Committee recommends either omega-3 PUFA supplementation with 250 to 500 mg of eicosapentaenoic acid (EPA) and docosahexaenoic acid (DHA) per day or two servings of oily fish per week for both primary and secondary prevention of CAD.3 The European Society of Cardiology also encourages increased consumption of oily fish.4

These recommendations are based on primary and secondary prevention studies performed between 1989 and 2007, which found a 15% to 29% decrease in all-cause mortality and nonfatal cardiovascular events associated with regular intake of omega-3 fatty acids.5-7 The systematic review and meta-analysis detailed below revisited the effect of omega-3 supplementation on major cardiovascular outcomes.1

STUDY SUMMARY
Omega-3 supplements don’t lower cardiovascular risk

This meta-analysis included 20 RCTs with a total of 68,680 patients. The median age was 68, with a range of 49 to 70.  Thirteen of the studies evaluated omega-3 PUFAs for secondary prevention of cardiovascular outcomes, four assessed both primary and secondary prevention, and three looked at outcomes in patients with implantable cardioverter defibrillators. All lasted longer than one year, and most were high quality, with a low risk for bias.

The median treatment duration was two years, with a maximum of 6.2 years. The mean omega-3 PUFA dose evaluated in the studies was 1.5 g/d, with the exception of two studies in which patients received omega-3 PUFAs through dietary sources. Twelve studies used a dose of 1 g or more per day. Half of the included trials were performed during the period when statins were routinely prescribed for cardiovascular risk modification (1998 or later).

Outcomes included all-cause mortality (17 studies), cardiac death (13 studies), sudden death (seven studies), MI (13 studies), and stroke (nine studies).

This meta-analysis found trends toward a decrease in all-cause mortality, cardiac death, sudden death, and MI in patients taking omega-3 PUFAs, but no statistically significant association between any of the outcomes and omega-3 PUFA supplementation. The relative risk for all-cause mortality was 0.96. Prespecified subgroup analysis found no association between treatment effect and omega-3 fatty acid dose.

Are dietary sources of omega-3s more effective?

In the two trials that involved dietary supplementation with omega-3 PUFAs, the results for all-cause mortality and cardiac death were conflicting, with one showing an increase in all-cause mortality and cardiac death and the other showing a decrease in both outcomes compared with the control group.

No harmful effects of omega-3 PUFAs were found in either the supplement- or diet-based studies.

WHAT’S NEW
More evidence of little benefit

The meta-analysis by Rizos et al is the most up-to-date, comprehensive look at the value of omega-3 fatty acids for primary and secondary prevention of cardiovascular events. It differs from previous reviews in that most of the included studies were well-done RCTs.

In addition, the studies were performed in both primary and secondary cardiovascular disease prevention settings and involved different forms of omega-3 PUFA supplementation, including dietary sources and supplements. The trials were predominantly larger than those included in previous systematic reviews, as well. The baseline risk for cardiovascular disease in the newer studies (seven of the 20 RCTs were completed after 2007) may be different from that of previous studies because of increased use of certain medications, such as statins.

In recent years, other studies of omega-3 PUFAs have had similar results. A meta-analysis of 14 RCTs found that omega-3 PUFA supplementation offered no benefit for the secondary prevention of cardiovascular disease.8 The FORWARD trial—published earlier this year—showed that omega-3 PUFAs did not decrease the recurrence of atrial fibrillation in patients with a history of confirmed paroxysmal atrial fibrillation.9 And an earlier (2006) analysis of RCTs and cohort studies found no benefit from omega-3 fatty acids for primary prevention of cardiovascular disease or cancer.10

 

 

CAVEATS
No significant help, and no harm

While this meta-analysis found no statistically significant benefits from omega-3 PUFAs, there is no evidence of harm from PUFA intake, whether from dietary sources or supplements. There is no need to tell patients who wish to take omega-3 supplements not to do so. But we should not promote their use for the sole purpose of cardiovascular disease prevention.

CHALLENGES TO IMPLEMENTATION
Changing minds won’t be easy

Despite recent findings indicating that omega-3 PUFAs provide little primary or secondary protection against cardiovascular events, advertising from supplement manufacturers may make it hard to change patients’ minds. Because diets and supplements containing these fatty acids do not cause apparent harm, patients and clinicians may decide that a small potential benefit is worth the expense.

References

1. Rizos E, Ntzani E, Bika E, et al. Association between omega-3 fatty acid supplementation and risk of major cardiovascular disease events: a systemic review and meta-analysis. JAMA. 2012; 308:1024-1033.

2. CDC. FastStats. Deaths and mortality. 2010 [updated April 5, 2013]. www.cdc.gov/nchs/fastats/deaths.htm. Accessed July 21, 2013.

3. Kris-Etherton PM, Harris WS, Appel LJ, et al. Fish consumption, fish oil, omega-3 fatty acids and cardiovascular disease. Circulation. 2002;106:2747-2757.

4. Van de Werf F, Bax J, Betriu A, et al. Management of acute myocardial infarction in patients presenting with persistent ST-segment elevation: the Task Force on the Management of ST-segment Elevation Acute Myocardial Infarction of the European Society of Cardiology. Eur Heart J. 2008;29:2909-2945.

5. Artham SM, Lavie CJ, Milani RV, et al. Fish oil in primary and secondary cardiovascular prevention. Ochsner J. 2008;8: 49-60.

6. Gruppo Italiano per lo Studio della Sopravvivenza nell’Infarto miocardico. Dietary supplementation with n-3 polyunsaturated fatty acids and vitamin E after myocardial infarction: results of GISSI-Prevenzione trial. Lancet. 1999; 354:447-455.

7. Yokoyama M, Origasa H, Matsuzaki M, et al. Effects of eicosapentaenoic acid on major coronary events in hypercholesteroleamic patients (JELIS): a randomized open-label, blinded endpoint analysis. Lancet. 2007;369:1090-1098.

8. Kwak SM, Myung SK, Lee YJ, et al. Efficacy of omega-3 fatty acid supplements (eicosapentaenoic acid and docosahexaenoic acid) in the secondary prevention of cardiovascular disease: a meta-analysis of randomized, double-blind, placebo-controlled trials. Arch Intern Med. 2012;172: 686-694.

9. Macchia A, Grancelli H, Varini S, et al. Omega-3 fatty acids for the prevention of recurrent symptomatic atrial fibrillation: results of the FORWARD (Randomized Trial to Assess Efficacy of PUFA for the Maintenance of Sinus Rhythm in Persistent Atrial Fibrillation) trial. J Am Coll Cardiol. 2013;61:463-468.

10. Hooper L, Thompson RL, Harrison RA, et al. Risks and benefits of omega 3 fats for mortality, cardiovascular disease, and cancer: systemic review. BMJ. 2006;332:752-760.

Acknowledgement

The PURLs Surveillance System was supported in part by Grant Number UL1RR024999 from the National Center for Research Resources, a Clinical Translational Science Award to the University of Chicago. The content is solely the responsibility of the authors and does not necessarily represent the official views of the National Center for Research Resources or the National Institutes of Health.

Copyright © 2013. The Family Physicians Inquiries Network. All rights reserved.

Reprinted with permission from the Family Physicians Inquiries Network and The Journal of Family Practice. 2013;62(7):372-374.

Author and Disclosure Information

Julie Monaco, MD, Anne Mounsey, MD, Jennifer Bello Kottenstette, MD

Issue
Clinician Reviews - 23(9)
Publications
Topics
Page Number
20
Legacy Keywords
purls, omega-3, supplements, cardiovascular, protection, practice change, fatty acid
Sections
Author and Disclosure Information

Julie Monaco, MD, Anne Mounsey, MD, Jennifer Bello Kottenstette, MD

Author and Disclosure Information

Julie Monaco, MD, Anne Mounsey, MD, Jennifer Bello Kottenstette, MD

Probably not, a new meta-analysis suggests. Read the details here.
Probably not, a new meta-analysis suggests. Read the details here.

PRACTICE CHANGER
Stop recommending omega-3 fatty acid supplements for cardiovascular protection. They have no significant impact on all-cause mortality, acute myocardial infarction (MI), sudden death, or stroke.1

STRENGTH OF RECOMMENDATION
A: Based on a meta-analysis of randomized controlled trials (RCTs).

ILLUSTRATIVE CASE
A 59-year-old patient who had an MI three years ago is taking an ACE inhibitor, a statin, and a -blocker. He asks you whether he should also take omega-3 fatty acid supplements to further decrease his risk for heart disease. What should you tell him?

Coronary artery disease (CAD) kills more than 500,000 Americans every year,2 and medical and dietary therapies for primary and secondary cardiovascular protection are paramount. Omega-3 polyunsaturated fatty acid (PUFA) supplementation is one such therapy.

Omega-3 PUFAs are precursors to certain prostaglandins that decrease the proinflammatory state in patients with CAD. They also lower triglyceride levels and produce an anti-arrhythmic effect by promoting electrical stability.

But do PUFA supplements ­provide cardioprotection?

The American Heart Association’s Nutrition Committee recommends either omega-3 PUFA supplementation with 250 to 500 mg of eicosapentaenoic acid (EPA) and docosahexaenoic acid (DHA) per day or two servings of oily fish per week for both primary and secondary prevention of CAD.3 The European Society of Cardiology also encourages increased consumption of oily fish.4

These recommendations are based on primary and secondary prevention studies performed between 1989 and 2007, which found a 15% to 29% decrease in all-cause mortality and nonfatal cardiovascular events associated with regular intake of omega-3 fatty acids.5-7 The systematic review and meta-analysis detailed below revisited the effect of omega-3 supplementation on major cardiovascular outcomes.1

STUDY SUMMARY
Omega-3 supplements don’t lower cardiovascular risk

This meta-analysis included 20 RCTs with a total of 68,680 patients. The median age was 68, with a range of 49 to 70.  Thirteen of the studies evaluated omega-3 PUFAs for secondary prevention of cardiovascular outcomes, four assessed both primary and secondary prevention, and three looked at outcomes in patients with implantable cardioverter defibrillators. All lasted longer than one year, and most were high quality, with a low risk for bias.

The median treatment duration was two years, with a maximum of 6.2 years. The mean omega-3 PUFA dose evaluated in the studies was 1.5 g/d, with the exception of two studies in which patients received omega-3 PUFAs through dietary sources. Twelve studies used a dose of 1 g or more per day. Half of the included trials were performed during the period when statins were routinely prescribed for cardiovascular risk modification (1998 or later).

Outcomes included all-cause mortality (17 studies), cardiac death (13 studies), sudden death (seven studies), MI (13 studies), and stroke (nine studies).

This meta-analysis found trends toward a decrease in all-cause mortality, cardiac death, sudden death, and MI in patients taking omega-3 PUFAs, but no statistically significant association between any of the outcomes and omega-3 PUFA supplementation. The relative risk for all-cause mortality was 0.96. Prespecified subgroup analysis found no association between treatment effect and omega-3 fatty acid dose.

Are dietary sources of omega-3s more effective?

In the two trials that involved dietary supplementation with omega-3 PUFAs, the results for all-cause mortality and cardiac death were conflicting, with one showing an increase in all-cause mortality and cardiac death and the other showing a decrease in both outcomes compared with the control group.

No harmful effects of omega-3 PUFAs were found in either the supplement- or diet-based studies.

WHAT’S NEW
More evidence of little benefit

The meta-analysis by Rizos et al is the most up-to-date, comprehensive look at the value of omega-3 fatty acids for primary and secondary prevention of cardiovascular events. It differs from previous reviews in that most of the included studies were well-done RCTs.

In addition, the studies were performed in both primary and secondary cardiovascular disease prevention settings and involved different forms of omega-3 PUFA supplementation, including dietary sources and supplements. The trials were predominantly larger than those included in previous systematic reviews, as well. The baseline risk for cardiovascular disease in the newer studies (seven of the 20 RCTs were completed after 2007) may be different from that of previous studies because of increased use of certain medications, such as statins.

In recent years, other studies of omega-3 PUFAs have had similar results. A meta-analysis of 14 RCTs found that omega-3 PUFA supplementation offered no benefit for the secondary prevention of cardiovascular disease.8 The FORWARD trial—published earlier this year—showed that omega-3 PUFAs did not decrease the recurrence of atrial fibrillation in patients with a history of confirmed paroxysmal atrial fibrillation.9 And an earlier (2006) analysis of RCTs and cohort studies found no benefit from omega-3 fatty acids for primary prevention of cardiovascular disease or cancer.10

 

 

CAVEATS
No significant help, and no harm

While this meta-analysis found no statistically significant benefits from omega-3 PUFAs, there is no evidence of harm from PUFA intake, whether from dietary sources or supplements. There is no need to tell patients who wish to take omega-3 supplements not to do so. But we should not promote their use for the sole purpose of cardiovascular disease prevention.

CHALLENGES TO IMPLEMENTATION
Changing minds won’t be easy

Despite recent findings indicating that omega-3 PUFAs provide little primary or secondary protection against cardiovascular events, advertising from supplement manufacturers may make it hard to change patients’ minds. Because diets and supplements containing these fatty acids do not cause apparent harm, patients and clinicians may decide that a small potential benefit is worth the expense.

References

1. Rizos E, Ntzani E, Bika E, et al. Association between omega-3 fatty acid supplementation and risk of major cardiovascular disease events: a systemic review and meta-analysis. JAMA. 2012; 308:1024-1033.

2. CDC. FastStats. Deaths and mortality. 2010 [updated April 5, 2013]. www.cdc.gov/nchs/fastats/deaths.htm. Accessed July 21, 2013.

3. Kris-Etherton PM, Harris WS, Appel LJ, et al. Fish consumption, fish oil, omega-3 fatty acids and cardiovascular disease. Circulation. 2002;106:2747-2757.

4. Van de Werf F, Bax J, Betriu A, et al. Management of acute myocardial infarction in patients presenting with persistent ST-segment elevation: the Task Force on the Management of ST-segment Elevation Acute Myocardial Infarction of the European Society of Cardiology. Eur Heart J. 2008;29:2909-2945.

5. Artham SM, Lavie CJ, Milani RV, et al. Fish oil in primary and secondary cardiovascular prevention. Ochsner J. 2008;8: 49-60.

6. Gruppo Italiano per lo Studio della Sopravvivenza nell’Infarto miocardico. Dietary supplementation with n-3 polyunsaturated fatty acids and vitamin E after myocardial infarction: results of GISSI-Prevenzione trial. Lancet. 1999; 354:447-455.

7. Yokoyama M, Origasa H, Matsuzaki M, et al. Effects of eicosapentaenoic acid on major coronary events in hypercholesteroleamic patients (JELIS): a randomized open-label, blinded endpoint analysis. Lancet. 2007;369:1090-1098.

8. Kwak SM, Myung SK, Lee YJ, et al. Efficacy of omega-3 fatty acid supplements (eicosapentaenoic acid and docosahexaenoic acid) in the secondary prevention of cardiovascular disease: a meta-analysis of randomized, double-blind, placebo-controlled trials. Arch Intern Med. 2012;172: 686-694.

9. Macchia A, Grancelli H, Varini S, et al. Omega-3 fatty acids for the prevention of recurrent symptomatic atrial fibrillation: results of the FORWARD (Randomized Trial to Assess Efficacy of PUFA for the Maintenance of Sinus Rhythm in Persistent Atrial Fibrillation) trial. J Am Coll Cardiol. 2013;61:463-468.

10. Hooper L, Thompson RL, Harrison RA, et al. Risks and benefits of omega 3 fats for mortality, cardiovascular disease, and cancer: systemic review. BMJ. 2006;332:752-760.

Acknowledgement

The PURLs Surveillance System was supported in part by Grant Number UL1RR024999 from the National Center for Research Resources, a Clinical Translational Science Award to the University of Chicago. The content is solely the responsibility of the authors and does not necessarily represent the official views of the National Center for Research Resources or the National Institutes of Health.

Copyright © 2013. The Family Physicians Inquiries Network. All rights reserved.

Reprinted with permission from the Family Physicians Inquiries Network and The Journal of Family Practice. 2013;62(7):372-374.

PRACTICE CHANGER
Stop recommending omega-3 fatty acid supplements for cardiovascular protection. They have no significant impact on all-cause mortality, acute myocardial infarction (MI), sudden death, or stroke.1

STRENGTH OF RECOMMENDATION
A: Based on a meta-analysis of randomized controlled trials (RCTs).

ILLUSTRATIVE CASE
A 59-year-old patient who had an MI three years ago is taking an ACE inhibitor, a statin, and a -blocker. He asks you whether he should also take omega-3 fatty acid supplements to further decrease his risk for heart disease. What should you tell him?

Coronary artery disease (CAD) kills more than 500,000 Americans every year,2 and medical and dietary therapies for primary and secondary cardiovascular protection are paramount. Omega-3 polyunsaturated fatty acid (PUFA) supplementation is one such therapy.

Omega-3 PUFAs are precursors to certain prostaglandins that decrease the proinflammatory state in patients with CAD. They also lower triglyceride levels and produce an anti-arrhythmic effect by promoting electrical stability.

But do PUFA supplements ­provide cardioprotection?

The American Heart Association’s Nutrition Committee recommends either omega-3 PUFA supplementation with 250 to 500 mg of eicosapentaenoic acid (EPA) and docosahexaenoic acid (DHA) per day or two servings of oily fish per week for both primary and secondary prevention of CAD.3 The European Society of Cardiology also encourages increased consumption of oily fish.4

These recommendations are based on primary and secondary prevention studies performed between 1989 and 2007, which found a 15% to 29% decrease in all-cause mortality and nonfatal cardiovascular events associated with regular intake of omega-3 fatty acids.5-7 The systematic review and meta-analysis detailed below revisited the effect of omega-3 supplementation on major cardiovascular outcomes.1

STUDY SUMMARY
Omega-3 supplements don’t lower cardiovascular risk

This meta-analysis included 20 RCTs with a total of 68,680 patients. The median age was 68, with a range of 49 to 70.  Thirteen of the studies evaluated omega-3 PUFAs for secondary prevention of cardiovascular outcomes, four assessed both primary and secondary prevention, and three looked at outcomes in patients with implantable cardioverter defibrillators. All lasted longer than one year, and most were high quality, with a low risk for bias.

The median treatment duration was two years, with a maximum of 6.2 years. The mean omega-3 PUFA dose evaluated in the studies was 1.5 g/d, with the exception of two studies in which patients received omega-3 PUFAs through dietary sources. Twelve studies used a dose of 1 g or more per day. Half of the included trials were performed during the period when statins were routinely prescribed for cardiovascular risk modification (1998 or later).

Outcomes included all-cause mortality (17 studies), cardiac death (13 studies), sudden death (seven studies), MI (13 studies), and stroke (nine studies).

This meta-analysis found trends toward a decrease in all-cause mortality, cardiac death, sudden death, and MI in patients taking omega-3 PUFAs, but no statistically significant association between any of the outcomes and omega-3 PUFA supplementation. The relative risk for all-cause mortality was 0.96. Prespecified subgroup analysis found no association between treatment effect and omega-3 fatty acid dose.

Are dietary sources of omega-3s more effective?

In the two trials that involved dietary supplementation with omega-3 PUFAs, the results for all-cause mortality and cardiac death were conflicting, with one showing an increase in all-cause mortality and cardiac death and the other showing a decrease in both outcomes compared with the control group.

No harmful effects of omega-3 PUFAs were found in either the supplement- or diet-based studies.

WHAT’S NEW
More evidence of little benefit

The meta-analysis by Rizos et al is the most up-to-date, comprehensive look at the value of omega-3 fatty acids for primary and secondary prevention of cardiovascular events. It differs from previous reviews in that most of the included studies were well-done RCTs.

In addition, the studies were performed in both primary and secondary cardiovascular disease prevention settings and involved different forms of omega-3 PUFA supplementation, including dietary sources and supplements. The trials were predominantly larger than those included in previous systematic reviews, as well. The baseline risk for cardiovascular disease in the newer studies (seven of the 20 RCTs were completed after 2007) may be different from that of previous studies because of increased use of certain medications, such as statins.

In recent years, other studies of omega-3 PUFAs have had similar results. A meta-analysis of 14 RCTs found that omega-3 PUFA supplementation offered no benefit for the secondary prevention of cardiovascular disease.8 The FORWARD trial—published earlier this year—showed that omega-3 PUFAs did not decrease the recurrence of atrial fibrillation in patients with a history of confirmed paroxysmal atrial fibrillation.9 And an earlier (2006) analysis of RCTs and cohort studies found no benefit from omega-3 fatty acids for primary prevention of cardiovascular disease or cancer.10

 

 

CAVEATS
No significant help, and no harm

While this meta-analysis found no statistically significant benefits from omega-3 PUFAs, there is no evidence of harm from PUFA intake, whether from dietary sources or supplements. There is no need to tell patients who wish to take omega-3 supplements not to do so. But we should not promote their use for the sole purpose of cardiovascular disease prevention.

CHALLENGES TO IMPLEMENTATION
Changing minds won’t be easy

Despite recent findings indicating that omega-3 PUFAs provide little primary or secondary protection against cardiovascular events, advertising from supplement manufacturers may make it hard to change patients’ minds. Because diets and supplements containing these fatty acids do not cause apparent harm, patients and clinicians may decide that a small potential benefit is worth the expense.

References

1. Rizos E, Ntzani E, Bika E, et al. Association between omega-3 fatty acid supplementation and risk of major cardiovascular disease events: a systemic review and meta-analysis. JAMA. 2012; 308:1024-1033.

2. CDC. FastStats. Deaths and mortality. 2010 [updated April 5, 2013]. www.cdc.gov/nchs/fastats/deaths.htm. Accessed July 21, 2013.

3. Kris-Etherton PM, Harris WS, Appel LJ, et al. Fish consumption, fish oil, omega-3 fatty acids and cardiovascular disease. Circulation. 2002;106:2747-2757.

4. Van de Werf F, Bax J, Betriu A, et al. Management of acute myocardial infarction in patients presenting with persistent ST-segment elevation: the Task Force on the Management of ST-segment Elevation Acute Myocardial Infarction of the European Society of Cardiology. Eur Heart J. 2008;29:2909-2945.

5. Artham SM, Lavie CJ, Milani RV, et al. Fish oil in primary and secondary cardiovascular prevention. Ochsner J. 2008;8: 49-60.

6. Gruppo Italiano per lo Studio della Sopravvivenza nell’Infarto miocardico. Dietary supplementation with n-3 polyunsaturated fatty acids and vitamin E after myocardial infarction: results of GISSI-Prevenzione trial. Lancet. 1999; 354:447-455.

7. Yokoyama M, Origasa H, Matsuzaki M, et al. Effects of eicosapentaenoic acid on major coronary events in hypercholesteroleamic patients (JELIS): a randomized open-label, blinded endpoint analysis. Lancet. 2007;369:1090-1098.

8. Kwak SM, Myung SK, Lee YJ, et al. Efficacy of omega-3 fatty acid supplements (eicosapentaenoic acid and docosahexaenoic acid) in the secondary prevention of cardiovascular disease: a meta-analysis of randomized, double-blind, placebo-controlled trials. Arch Intern Med. 2012;172: 686-694.

9. Macchia A, Grancelli H, Varini S, et al. Omega-3 fatty acids for the prevention of recurrent symptomatic atrial fibrillation: results of the FORWARD (Randomized Trial to Assess Efficacy of PUFA for the Maintenance of Sinus Rhythm in Persistent Atrial Fibrillation) trial. J Am Coll Cardiol. 2013;61:463-468.

10. Hooper L, Thompson RL, Harrison RA, et al. Risks and benefits of omega 3 fats for mortality, cardiovascular disease, and cancer: systemic review. BMJ. 2006;332:752-760.

Acknowledgement

The PURLs Surveillance System was supported in part by Grant Number UL1RR024999 from the National Center for Research Resources, a Clinical Translational Science Award to the University of Chicago. The content is solely the responsibility of the authors and does not necessarily represent the official views of the National Center for Research Resources or the National Institutes of Health.

Copyright © 2013. The Family Physicians Inquiries Network. All rights reserved.

Reprinted with permission from the Family Physicians Inquiries Network and The Journal of Family Practice. 2013;62(7):372-374.

Issue
Clinician Reviews - 23(9)
Issue
Clinician Reviews - 23(9)
Page Number
20
Page Number
20
Publications
Publications
Topics
Article Type
Display Headline
Should You Still Recommend Omega-3 Supplements?
Display Headline
Should You Still Recommend Omega-3 Supplements?
Legacy Keywords
purls, omega-3, supplements, cardiovascular, protection, practice change, fatty acid
Legacy Keywords
purls, omega-3, supplements, cardiovascular, protection, practice change, fatty acid
Sections
Article Source

PURLs Copyright

Inside the Article

Should you still recommend omega-3 supplements?

Article Type
Changed
Tue, 07/14/2020 - 15:06
Display Headline
Should you still recommend omega-3 supplements?
PRACTICE CHANGER

Stop recommending omega-3 fatty acid supplements for cardiovascular protection. They have no significant impact on all-cause mortality, acute myocardial infarction, sudden death, or stroke.1

Strength of Recommendation

A: Based on a meta-analysis of randomized controlled trials (RCTs).

Rizos E, Ntzani E, Bika E, et al. Association between omega-3 fatty acid supplementation and risk of major cardiovascular disease events: a systematic review and meta-analysis. JAMA. 2012;308:1024-1033.

 

ILLUSTRATIVE CASE

A 59–year-old patient who had a myocardial infarction (mi) 3 years ago is taking an ace inhibitor, a statin, and a b-blocker. He asks you whether he should also take omega-3 fatty acid supplements to further decrease his risk of heart disease. What should you tell him?

Coronary artery disease (CAD) kills more than 500,000 Americans every year2, and medical and dietary therapies for primary and secondary cardiovascular protection are paramount. Omega-3 polyunsaturated fatty acid (PUFA) supplementation is one such therapy. Omega-3 PUFAs are precursors to certain prostaglandins that decrease the proinflammatory state in patients with CAD. They also lower triglyceride levels and produce an antiarrhythmic effect by promoting electrical stability.

But do PUFA supplements provide cardioprotection?
The American Heart Association’s Nutrition Committee recommends either omega-3 PUFA supplementation with 250 to 500 mg eicosapentaenoic acid (EPA) and docosahexaenoic acid (DHA) per day or 2 servings of oily fish per week for both primary and secondary prevention of CAD.3 The European Society of Cardiology also encourages increased consumption of oily fish.4

These recommendations are based on primary and secondary prevention studies, performed between 1989 and 2007, that found a 15% to 29% decrease in all-cause mortality and nonfatal cardiovascular events associated with regular intake of omega-3 fatty acids.5-7 The systematic review and meta-analysis detailed below revisited the effect of omega-3 supplementation on major cardiovascular outcomes.1

STUDY SUMMARY: Omega-3 supplements don’t lower cardiovascular risk

This meta-analysis included 20 RCTs with a total of 68,680 patients. The median age was 68 years, with a range from 49 to 70 years. Thirteen of the studies evaluated omega-3 PUFAs for secondary prevention of cardiovascular outcomes, 4 assessed both primary and secondary prevention, and 3 looked at outcomes in patients with implantable cardioverter defibrillators. All lasted longer than one year, and most were high quality, with a low risk of bias.

The median treatment duration was 2 years, with a maximum of 6.2 years. The mean omega-3 PUFA dose evaluated in the studies was 1.5 g per day, with the exception of 2 studies in which patients received omega-3 PUFAs through dietary sources. Twelve studies used a dose of 1 g or more per day. Half of the included trials were performed during the period when statins were routinely prescribed for cardiovascular risk modification (1998 or later).

Outcomes included all-cause mortality (17 studies), cardiac death (13 studies), sudden death (7 studies), MI (13 studies), and stroke (9 studies).

This meta-analysis found trends toward a decrease in all-cause mortality, cardiac death, sudden death, and MI in patients taking omega-3 PUFAs, but no statistically significant association between any of the outcomes and omega-3 PUFA supplementation. The relative risk for all-cause mortality was 0.96 (95% confidence interval, 0.91-1.02; P=.17). Prespecified subgroup analysis found no association between treatment effect and omega-3 fatty acid dose.

Are dietary sources of omega-3s more effective?
In the 2 trials involving dietary supplementation with omega-3 PUFAs, the results for all-cause mortality and cardiac death were conflicting, with one showing an increase in all-cause mortality and cardiac death and the other showing a decrease in both outcomes compared with the control group. No harmful effects of omega-3 PUFAs were found in either the supplement- or diet-based studies. 

 

 

 

WHAT’S NEW: More evidence of little benefit 

The meta-analysis by Rizos et al is the most up-to-date, comprehensive look at the value of omega-3 fatty acids for primary and secondary prevention of cardiovascular events.It differs from previous reviews in that most included studies were well-done RCTs. In addition, the studies were performed in both primary and secondary cardiovascular disease prevention settings and involved different forms of omega-3 PUFA supplementation, including dietary sources and supplements. The trials were predominantly larger than those included in previous systematic reviews, as well. The baseline risk for cardiovascular disease in the newer studies (7 of the 20 RCTs were completed after 2007) may be different from that of previous studies because of increased use of certain medications,such as statins. In recent years, other studies of omega-3 PUFAs have had similar results. A metaanalysis of 14 RCTs found that omega-3 PUFA supplementation offered no benefit for the secondary prevention of cardiovascular disease.8 The FORWARD trial—published earlier this year—showed that omega-3 PUFAs did not decrease the recurrence of atrial fibrillationin patients with a history of confirmed paroxysmal atrial fibrillation.9 And an earlier(2006) analysis of RCTs and cohort studies found no benefit from omega-3 fatty acids for primary prevention of cardiovascular disease or cancer.10

CAVEATS: No significant help, and no harm

There’s no need to tell patients who wish to take omega-3 supplements not to do so, but we should not promote their use for the sole purpose of cardiovascular disease protection.

While this meta-analysis found no statistically significant benefits from omega-3 PUFAs, there is no evidence of harm from PUFA intake, whether from dietary sources or supplements. There is no need to tell patients who wish to take omega-3 supplements not to do so. But we should not promote their use for the sole purpose of cardiovascular disease prevention.

CHALLENGES TO IMPLEMENTATION: Changing minds won’t be easy

Despite recent findings indicating that omega-3 PUFAs provide little primary or secondary protection against cardiovascular events, advertising from supplement manufacturers may make it hard to change patients’ minds. Because diets and supplements containing these fatty acids do not cause apparent harm, patients and physicians may decide that a small potential benefit is worth the expense.

Acknowledgement

The PURLs Surveillance System was supported in part by Grant Number UL1RR024999 from the National Center for Research Resources, a Clinical Translational Science Award to the University of Chicago. The content is solely the responsibility of the authors and does not necessarily represent the official views of the National Center for Research Resources or the National Institutes of Health.

Files
References

1. Rizos E, Ntzani E, Bika E, et al. Association between omega-3 fatty acid supplementation and risk of major cardiovascular disease events: a systemic review and meta-analysis. JAMA.  2012;308:1024-1033.

2. Centers for Disease Control and Prevention. FastStats. Deaths and mortality. 2010. Available at: http://www.cdc.gov/nchs/fastats/deaths.htm. Updated April 5, 2013. Accessed July 21, 2013.

3. Kris-Etherton PM, Harris WS, Appel LJ, et al. Fish consumption, fish oil, omega-3 fatty acids and cardiovascular disease. Circulation.  2002;106:2747-2757.

4. Van de Werf F, Bax J, Betriu A, et al. Management of acute myocardial infarction in patients presenting with persistent ST-segment elevation: the Task Force on the management of ST-segment elevation acute myocardial infarction of the European Society of Cardiology. Eur Heart J. 2008;29:2909-2945.

5. Artham SM, Lavie CJ, Milani RV, et al. Fish oil in primary and secondary cardiovascular prevention. Ochsner J. 2008;8:49-60.

6. Gruppo Italiano per lo Studio della Sopravvivenza nell’Infarto miocardico. Dietary supplementation with n-3 polyunsaturated fatty acids and vitamin E after myocardial infarction: results of GISSI-Prevenzione trial. Lancet. 1999;354:447-455.

7. Yokoyama M, Origasa H, Matsuzaki M, et al. Effects of eicosapentaenoic acid on major coronary events in hypercholesteroleamic patients (JELIS): a randomized open-label, blinded endpoint analysis. Lancet. 2007;369:1090-1098.

8. Kwak SM, Myung SK, Lee YJ, et al. Efficacy of omega-3 fatty acid supplements (eicosapentaenoic acid and docosahexaenoic acid) in the secondary prevention of cardiovascular disease: a metaanalysis of randomized, double-blind, placebo-controlled trials.  Arch Intern Med. 2012;172:686-694.

9. Macchia A, Grancelli H, Varini S, et al. Omega-3 fatty acids for the prevention of recurrent symptomatic atrial fibrillation: results of the FORWARD (Randomized Trial to Assess Efficacy of PUFA for the Maintenance of Sinus Rhythm in Persistent Atrial Fibrillation) trial. J Am Coll Cardiol. 2013;61:463-468.

10. Hooper L, Thompson RL, Harrison RA, et al. Risks and benefits of omega 3 fats for mortality, cardiovascular disease, and cancer: systemic review. BMJ. 2006;332:752-760.

Article PDF
Author and Disclosure Information

Julie Monaco, MD;
Anne Mounsey, MD;
Jennifer Bello Kottenstette, MD

Department of Family Medicine, University of North Carolina at Chapel Hill (Drs. Monaco and Mounsey); Department of Family Medicine, The University of Chicago (Dr. Kottenstette)

PURLs EDITOR
James Stevermer, MD, MSPH
Department of Family Medicine, University of Missouri-Columbia

Issue
The Journal of Family Practice - 62(8)
Publications
Topics
Page Number
422-424
Legacy Keywords
Julie Monaco; MD; Anne Mounsey; MD; Jennifer; Bello Kottenstette; MD; PURLs; omega-3 supplements; polyunsaturated fatty acid; PUFA; eicosapentaenoic acid; EPA; docosahexaenoic acid; DHA; PUFA supplements
Sections
Files
Files
Author and Disclosure Information

Julie Monaco, MD;
Anne Mounsey, MD;
Jennifer Bello Kottenstette, MD

Department of Family Medicine, University of North Carolina at Chapel Hill (Drs. Monaco and Mounsey); Department of Family Medicine, The University of Chicago (Dr. Kottenstette)

PURLs EDITOR
James Stevermer, MD, MSPH
Department of Family Medicine, University of Missouri-Columbia

Author and Disclosure Information

Julie Monaco, MD;
Anne Mounsey, MD;
Jennifer Bello Kottenstette, MD

Department of Family Medicine, University of North Carolina at Chapel Hill (Drs. Monaco and Mounsey); Department of Family Medicine, The University of Chicago (Dr. Kottenstette)

PURLs EDITOR
James Stevermer, MD, MSPH
Department of Family Medicine, University of Missouri-Columbia

Article PDF
Article PDF
PRACTICE CHANGER

Stop recommending omega-3 fatty acid supplements for cardiovascular protection. They have no significant impact on all-cause mortality, acute myocardial infarction, sudden death, or stroke.1

Strength of Recommendation

A: Based on a meta-analysis of randomized controlled trials (RCTs).

Rizos E, Ntzani E, Bika E, et al. Association between omega-3 fatty acid supplementation and risk of major cardiovascular disease events: a systematic review and meta-analysis. JAMA. 2012;308:1024-1033.

 

ILLUSTRATIVE CASE

A 59–year-old patient who had a myocardial infarction (mi) 3 years ago is taking an ace inhibitor, a statin, and a b-blocker. He asks you whether he should also take omega-3 fatty acid supplements to further decrease his risk of heart disease. What should you tell him?

Coronary artery disease (CAD) kills more than 500,000 Americans every year2, and medical and dietary therapies for primary and secondary cardiovascular protection are paramount. Omega-3 polyunsaturated fatty acid (PUFA) supplementation is one such therapy. Omega-3 PUFAs are precursors to certain prostaglandins that decrease the proinflammatory state in patients with CAD. They also lower triglyceride levels and produce an antiarrhythmic effect by promoting electrical stability.

But do PUFA supplements provide cardioprotection?
The American Heart Association’s Nutrition Committee recommends either omega-3 PUFA supplementation with 250 to 500 mg eicosapentaenoic acid (EPA) and docosahexaenoic acid (DHA) per day or 2 servings of oily fish per week for both primary and secondary prevention of CAD.3 The European Society of Cardiology also encourages increased consumption of oily fish.4

These recommendations are based on primary and secondary prevention studies, performed between 1989 and 2007, that found a 15% to 29% decrease in all-cause mortality and nonfatal cardiovascular events associated with regular intake of omega-3 fatty acids.5-7 The systematic review and meta-analysis detailed below revisited the effect of omega-3 supplementation on major cardiovascular outcomes.1

STUDY SUMMARY: Omega-3 supplements don’t lower cardiovascular risk

This meta-analysis included 20 RCTs with a total of 68,680 patients. The median age was 68 years, with a range from 49 to 70 years. Thirteen of the studies evaluated omega-3 PUFAs for secondary prevention of cardiovascular outcomes, 4 assessed both primary and secondary prevention, and 3 looked at outcomes in patients with implantable cardioverter defibrillators. All lasted longer than one year, and most were high quality, with a low risk of bias.

The median treatment duration was 2 years, with a maximum of 6.2 years. The mean omega-3 PUFA dose evaluated in the studies was 1.5 g per day, with the exception of 2 studies in which patients received omega-3 PUFAs through dietary sources. Twelve studies used a dose of 1 g or more per day. Half of the included trials were performed during the period when statins were routinely prescribed for cardiovascular risk modification (1998 or later).

Outcomes included all-cause mortality (17 studies), cardiac death (13 studies), sudden death (7 studies), MI (13 studies), and stroke (9 studies).

This meta-analysis found trends toward a decrease in all-cause mortality, cardiac death, sudden death, and MI in patients taking omega-3 PUFAs, but no statistically significant association between any of the outcomes and omega-3 PUFA supplementation. The relative risk for all-cause mortality was 0.96 (95% confidence interval, 0.91-1.02; P=.17). Prespecified subgroup analysis found no association between treatment effect and omega-3 fatty acid dose.

Are dietary sources of omega-3s more effective?
In the 2 trials involving dietary supplementation with omega-3 PUFAs, the results for all-cause mortality and cardiac death were conflicting, with one showing an increase in all-cause mortality and cardiac death and the other showing a decrease in both outcomes compared with the control group. No harmful effects of omega-3 PUFAs were found in either the supplement- or diet-based studies. 

 

 

 

WHAT’S NEW: More evidence of little benefit 

The meta-analysis by Rizos et al is the most up-to-date, comprehensive look at the value of omega-3 fatty acids for primary and secondary prevention of cardiovascular events.It differs from previous reviews in that most included studies were well-done RCTs. In addition, the studies were performed in both primary and secondary cardiovascular disease prevention settings and involved different forms of omega-3 PUFA supplementation, including dietary sources and supplements. The trials were predominantly larger than those included in previous systematic reviews, as well. The baseline risk for cardiovascular disease in the newer studies (7 of the 20 RCTs were completed after 2007) may be different from that of previous studies because of increased use of certain medications,such as statins. In recent years, other studies of omega-3 PUFAs have had similar results. A metaanalysis of 14 RCTs found that omega-3 PUFA supplementation offered no benefit for the secondary prevention of cardiovascular disease.8 The FORWARD trial—published earlier this year—showed that omega-3 PUFAs did not decrease the recurrence of atrial fibrillationin patients with a history of confirmed paroxysmal atrial fibrillation.9 And an earlier(2006) analysis of RCTs and cohort studies found no benefit from omega-3 fatty acids for primary prevention of cardiovascular disease or cancer.10

CAVEATS: No significant help, and no harm

There’s no need to tell patients who wish to take omega-3 supplements not to do so, but we should not promote their use for the sole purpose of cardiovascular disease protection.

While this meta-analysis found no statistically significant benefits from omega-3 PUFAs, there is no evidence of harm from PUFA intake, whether from dietary sources or supplements. There is no need to tell patients who wish to take omega-3 supplements not to do so. But we should not promote their use for the sole purpose of cardiovascular disease prevention.

CHALLENGES TO IMPLEMENTATION: Changing minds won’t be easy

Despite recent findings indicating that omega-3 PUFAs provide little primary or secondary protection against cardiovascular events, advertising from supplement manufacturers may make it hard to change patients’ minds. Because diets and supplements containing these fatty acids do not cause apparent harm, patients and physicians may decide that a small potential benefit is worth the expense.

Acknowledgement

The PURLs Surveillance System was supported in part by Grant Number UL1RR024999 from the National Center for Research Resources, a Clinical Translational Science Award to the University of Chicago. The content is solely the responsibility of the authors and does not necessarily represent the official views of the National Center for Research Resources or the National Institutes of Health.

PRACTICE CHANGER

Stop recommending omega-3 fatty acid supplements for cardiovascular protection. They have no significant impact on all-cause mortality, acute myocardial infarction, sudden death, or stroke.1

Strength of Recommendation

A: Based on a meta-analysis of randomized controlled trials (RCTs).

Rizos E, Ntzani E, Bika E, et al. Association between omega-3 fatty acid supplementation and risk of major cardiovascular disease events: a systematic review and meta-analysis. JAMA. 2012;308:1024-1033.

 

ILLUSTRATIVE CASE

A 59–year-old patient who had a myocardial infarction (mi) 3 years ago is taking an ace inhibitor, a statin, and a b-blocker. He asks you whether he should also take omega-3 fatty acid supplements to further decrease his risk of heart disease. What should you tell him?

Coronary artery disease (CAD) kills more than 500,000 Americans every year2, and medical and dietary therapies for primary and secondary cardiovascular protection are paramount. Omega-3 polyunsaturated fatty acid (PUFA) supplementation is one such therapy. Omega-3 PUFAs are precursors to certain prostaglandins that decrease the proinflammatory state in patients with CAD. They also lower triglyceride levels and produce an antiarrhythmic effect by promoting electrical stability.

But do PUFA supplements provide cardioprotection?
The American Heart Association’s Nutrition Committee recommends either omega-3 PUFA supplementation with 250 to 500 mg eicosapentaenoic acid (EPA) and docosahexaenoic acid (DHA) per day or 2 servings of oily fish per week for both primary and secondary prevention of CAD.3 The European Society of Cardiology also encourages increased consumption of oily fish.4

These recommendations are based on primary and secondary prevention studies, performed between 1989 and 2007, that found a 15% to 29% decrease in all-cause mortality and nonfatal cardiovascular events associated with regular intake of omega-3 fatty acids.5-7 The systematic review and meta-analysis detailed below revisited the effect of omega-3 supplementation on major cardiovascular outcomes.1

STUDY SUMMARY: Omega-3 supplements don’t lower cardiovascular risk

This meta-analysis included 20 RCTs with a total of 68,680 patients. The median age was 68 years, with a range from 49 to 70 years. Thirteen of the studies evaluated omega-3 PUFAs for secondary prevention of cardiovascular outcomes, 4 assessed both primary and secondary prevention, and 3 looked at outcomes in patients with implantable cardioverter defibrillators. All lasted longer than one year, and most were high quality, with a low risk of bias.

The median treatment duration was 2 years, with a maximum of 6.2 years. The mean omega-3 PUFA dose evaluated in the studies was 1.5 g per day, with the exception of 2 studies in which patients received omega-3 PUFAs through dietary sources. Twelve studies used a dose of 1 g or more per day. Half of the included trials were performed during the period when statins were routinely prescribed for cardiovascular risk modification (1998 or later).

Outcomes included all-cause mortality (17 studies), cardiac death (13 studies), sudden death (7 studies), MI (13 studies), and stroke (9 studies).

This meta-analysis found trends toward a decrease in all-cause mortality, cardiac death, sudden death, and MI in patients taking omega-3 PUFAs, but no statistically significant association between any of the outcomes and omega-3 PUFA supplementation. The relative risk for all-cause mortality was 0.96 (95% confidence interval, 0.91-1.02; P=.17). Prespecified subgroup analysis found no association between treatment effect and omega-3 fatty acid dose.

Are dietary sources of omega-3s more effective?
In the 2 trials involving dietary supplementation with omega-3 PUFAs, the results for all-cause mortality and cardiac death were conflicting, with one showing an increase in all-cause mortality and cardiac death and the other showing a decrease in both outcomes compared with the control group. No harmful effects of omega-3 PUFAs were found in either the supplement- or diet-based studies. 

 

 

 

WHAT’S NEW: More evidence of little benefit 

The meta-analysis by Rizos et al is the most up-to-date, comprehensive look at the value of omega-3 fatty acids for primary and secondary prevention of cardiovascular events.It differs from previous reviews in that most included studies were well-done RCTs. In addition, the studies were performed in both primary and secondary cardiovascular disease prevention settings and involved different forms of omega-3 PUFA supplementation, including dietary sources and supplements. The trials were predominantly larger than those included in previous systematic reviews, as well. The baseline risk for cardiovascular disease in the newer studies (7 of the 20 RCTs were completed after 2007) may be different from that of previous studies because of increased use of certain medications,such as statins. In recent years, other studies of omega-3 PUFAs have had similar results. A metaanalysis of 14 RCTs found that omega-3 PUFA supplementation offered no benefit for the secondary prevention of cardiovascular disease.8 The FORWARD trial—published earlier this year—showed that omega-3 PUFAs did not decrease the recurrence of atrial fibrillationin patients with a history of confirmed paroxysmal atrial fibrillation.9 And an earlier(2006) analysis of RCTs and cohort studies found no benefit from omega-3 fatty acids for primary prevention of cardiovascular disease or cancer.10

CAVEATS: No significant help, and no harm

There’s no need to tell patients who wish to take omega-3 supplements not to do so, but we should not promote their use for the sole purpose of cardiovascular disease protection.

While this meta-analysis found no statistically significant benefits from omega-3 PUFAs, there is no evidence of harm from PUFA intake, whether from dietary sources or supplements. There is no need to tell patients who wish to take omega-3 supplements not to do so. But we should not promote their use for the sole purpose of cardiovascular disease prevention.

CHALLENGES TO IMPLEMENTATION: Changing minds won’t be easy

Despite recent findings indicating that omega-3 PUFAs provide little primary or secondary protection against cardiovascular events, advertising from supplement manufacturers may make it hard to change patients’ minds. Because diets and supplements containing these fatty acids do not cause apparent harm, patients and physicians may decide that a small potential benefit is worth the expense.

Acknowledgement

The PURLs Surveillance System was supported in part by Grant Number UL1RR024999 from the National Center for Research Resources, a Clinical Translational Science Award to the University of Chicago. The content is solely the responsibility of the authors and does not necessarily represent the official views of the National Center for Research Resources or the National Institutes of Health.

References

1. Rizos E, Ntzani E, Bika E, et al. Association between omega-3 fatty acid supplementation and risk of major cardiovascular disease events: a systemic review and meta-analysis. JAMA.  2012;308:1024-1033.

2. Centers for Disease Control and Prevention. FastStats. Deaths and mortality. 2010. Available at: http://www.cdc.gov/nchs/fastats/deaths.htm. Updated April 5, 2013. Accessed July 21, 2013.

3. Kris-Etherton PM, Harris WS, Appel LJ, et al. Fish consumption, fish oil, omega-3 fatty acids and cardiovascular disease. Circulation.  2002;106:2747-2757.

4. Van de Werf F, Bax J, Betriu A, et al. Management of acute myocardial infarction in patients presenting with persistent ST-segment elevation: the Task Force on the management of ST-segment elevation acute myocardial infarction of the European Society of Cardiology. Eur Heart J. 2008;29:2909-2945.

5. Artham SM, Lavie CJ, Milani RV, et al. Fish oil in primary and secondary cardiovascular prevention. Ochsner J. 2008;8:49-60.

6. Gruppo Italiano per lo Studio della Sopravvivenza nell’Infarto miocardico. Dietary supplementation with n-3 polyunsaturated fatty acids and vitamin E after myocardial infarction: results of GISSI-Prevenzione trial. Lancet. 1999;354:447-455.

7. Yokoyama M, Origasa H, Matsuzaki M, et al. Effects of eicosapentaenoic acid on major coronary events in hypercholesteroleamic patients (JELIS): a randomized open-label, blinded endpoint analysis. Lancet. 2007;369:1090-1098.

8. Kwak SM, Myung SK, Lee YJ, et al. Efficacy of omega-3 fatty acid supplements (eicosapentaenoic acid and docosahexaenoic acid) in the secondary prevention of cardiovascular disease: a metaanalysis of randomized, double-blind, placebo-controlled trials.  Arch Intern Med. 2012;172:686-694.

9. Macchia A, Grancelli H, Varini S, et al. Omega-3 fatty acids for the prevention of recurrent symptomatic atrial fibrillation: results of the FORWARD (Randomized Trial to Assess Efficacy of PUFA for the Maintenance of Sinus Rhythm in Persistent Atrial Fibrillation) trial. J Am Coll Cardiol. 2013;61:463-468.

10. Hooper L, Thompson RL, Harrison RA, et al. Risks and benefits of omega 3 fats for mortality, cardiovascular disease, and cancer: systemic review. BMJ. 2006;332:752-760.

References

1. Rizos E, Ntzani E, Bika E, et al. Association between omega-3 fatty acid supplementation and risk of major cardiovascular disease events: a systemic review and meta-analysis. JAMA.  2012;308:1024-1033.

2. Centers for Disease Control and Prevention. FastStats. Deaths and mortality. 2010. Available at: http://www.cdc.gov/nchs/fastats/deaths.htm. Updated April 5, 2013. Accessed July 21, 2013.

3. Kris-Etherton PM, Harris WS, Appel LJ, et al. Fish consumption, fish oil, omega-3 fatty acids and cardiovascular disease. Circulation.  2002;106:2747-2757.

4. Van de Werf F, Bax J, Betriu A, et al. Management of acute myocardial infarction in patients presenting with persistent ST-segment elevation: the Task Force on the management of ST-segment elevation acute myocardial infarction of the European Society of Cardiology. Eur Heart J. 2008;29:2909-2945.

5. Artham SM, Lavie CJ, Milani RV, et al. Fish oil in primary and secondary cardiovascular prevention. Ochsner J. 2008;8:49-60.

6. Gruppo Italiano per lo Studio della Sopravvivenza nell’Infarto miocardico. Dietary supplementation with n-3 polyunsaturated fatty acids and vitamin E after myocardial infarction: results of GISSI-Prevenzione trial. Lancet. 1999;354:447-455.

7. Yokoyama M, Origasa H, Matsuzaki M, et al. Effects of eicosapentaenoic acid on major coronary events in hypercholesteroleamic patients (JELIS): a randomized open-label, blinded endpoint analysis. Lancet. 2007;369:1090-1098.

8. Kwak SM, Myung SK, Lee YJ, et al. Efficacy of omega-3 fatty acid supplements (eicosapentaenoic acid and docosahexaenoic acid) in the secondary prevention of cardiovascular disease: a metaanalysis of randomized, double-blind, placebo-controlled trials.  Arch Intern Med. 2012;172:686-694.

9. Macchia A, Grancelli H, Varini S, et al. Omega-3 fatty acids for the prevention of recurrent symptomatic atrial fibrillation: results of the FORWARD (Randomized Trial to Assess Efficacy of PUFA for the Maintenance of Sinus Rhythm in Persistent Atrial Fibrillation) trial. J Am Coll Cardiol. 2013;61:463-468.

10. Hooper L, Thompson RL, Harrison RA, et al. Risks and benefits of omega 3 fats for mortality, cardiovascular disease, and cancer: systemic review. BMJ. 2006;332:752-760.

Issue
The Journal of Family Practice - 62(8)
Issue
The Journal of Family Practice - 62(8)
Page Number
422-424
Page Number
422-424
Publications
Publications
Topics
Article Type
Display Headline
Should you still recommend omega-3 supplements?
Display Headline
Should you still recommend omega-3 supplements?
Legacy Keywords
Julie Monaco; MD; Anne Mounsey; MD; Jennifer; Bello Kottenstette; MD; PURLs; omega-3 supplements; polyunsaturated fatty acid; PUFA; eicosapentaenoic acid; EPA; docosahexaenoic acid; DHA; PUFA supplements
Legacy Keywords
Julie Monaco; MD; Anne Mounsey; MD; Jennifer; Bello Kottenstette; MD; PURLs; omega-3 supplements; polyunsaturated fatty acid; PUFA; eicosapentaenoic acid; EPA; docosahexaenoic acid; DHA; PUFA supplements
Sections
PURLs Copyright

Copyright © 2013. The Family Physicians Inquiries Network. All rights reserved.

Disallow All Ads
Alternative CME
Use ProPublica
Hide sidebar & use full width
render the right sidebar.
Conference Recap Checkbox
Not Conference Recap
Clinical Edge
Display the Slideshow in this Article
Article PDF Media
Media Files